You are on page 1of 133

Pediatrics

1) Peeling of the palms of the hand and the soles of the feet is a characteristic feature of: A Chickenpox. B Scarlet fever. C Kawasaki disease. D Tinea infection. E Papular urticaria. (False) (True) (True) (True) (False)

Comments: Both chicken pox and popular urticaria are associated with encrusted vesicles. Kawasaki disease is associated with high temperature, mucosal inflammation and desquamation.

2) A term newborn infant is noticed to be hypotonic. The following are possible causes:

A Turner's syndrome. B Kugelberg-Welander disease. C Cephalhaematoma. D Prader-Willi syndrome. E Galactosaemia.

(False) (False) (False) (True) (True)

Comments: Causes of neonatal hypotonia can be classified anatomically: Systemic: sepsis, trisomies, metabolic disease, e.g. galactosaemia, drugs, etc Brain: hypoxic ischaemic encephalopathy, intracranial haemorrhage, malformations Spinal cord: birth trauma, spina bifida, tumour Anterior horn cell: spinal muscular atrophy Neuromuscular junction: neonatal myasthenia gravis Muscle: congenital myopathies, benign congenital hypotonia.

3) Respiration is stimulated by the following: A Hypercarbia. B Hypoxia. C Acidosis. D Increased barometric pressure. E Carbon monoxide inhalation. (True) (True) (True) (False) (False)

Comments: Respiration is stimulated primarily by medullary chemoreceptors which respond to

Pediatrics

hypercarbia. The presence of acidosis exaggerates this response. Oxygen only stimulates respiration if levels are very low, e.g. at high altitude (low barometric pressure). This also becomes more important in chronic lung disease when carbon dioxide levels are chronically raised. From Hannam et al. MRCP (Paediatrics) Part 1 MCQs. page 67 WB Saunders. Reproduced with permission.

4) Which of the following features supports a diagnosis of carditis in a child suspected of having rheumatic fever?

A Pan-systolic murmur in the mitral area. B Elevated sleeping pulse. C Sinus arrhythmia. D Ejection systolic murmur at the left sternal border. E Pericardial rub.

(True) (False) (False) (False) (True)

Comments: Pan systolic murmur indicates mitral incompetence. Elevated sleeping pulse though a feature, is not diagnostic. The classical valve lesions are aortic and mitral incompetence. Pericarditis indicates cardiac involvement and almost always occurs with valvulitis.

5) Early cardiac failure during the neonatal period is often associated with:

A Peripheral oedema B Failure to thrive C Feeding difficulties D Episodes of apnoea E A raised pulse rate but normal respiratory rate

(False) (True) (True) (True) (False)

Comments: Oedema occurs uncommonly in neonates. Failure to thrive, feeding difficulties and apnoeic episodes are common. Tachycardia and tachypnoea with respiratory rate of above 60 per minute are common. 6) A normal 4 year old would be expected to be able to: A Catch a ball. B Draw a circle. C Count to 10. D Tell you his/her name and address. (True) (True) (True) (True)

Pediatrics

E Get dressed alone.

(True)

Comments: It is essential to have some knowledge of each of the four areas of child development (i.e. gross motor, fine motor and vision, hearing and speech and social behaviour) for several age groups. We suggest learning two to three normal milestones for each area of development for the following ages: 6 weeks, 6-8 months, 12 months, 2.5 years, 3 years, 4 years and 5 years. From Hannam et al. MRCP (Paediatrics) Part 1 MCQs. page 169 WB Saunders. Reproduced with permission.

7) The following may cause deafness in children A meningococcal septicaemia B maternal rubella C congenital syphilis D hereditary nephritis E Pendreds syndrome (True) (True) (True) (True) (True)

Comments: 1.Deafness is an important complication of meningitis in children 2. Deafness occurs in up to 80% of children affected by congenital rubella 3. Nerve deafness can be a late manifestation of congenital syphilis 4. Alports syndrome is the combination of nerve deafness and hereditary nephritis 5.Pendreds syndrome nerve deafness and hypothyroidism due to an inborn defect of organification of trapped iodine

8) A 6-week-old male infant is admitted to hospital with 10-15% dehydration. The serum sodium is 160 mmol/l and the serum potassium is 3.3 mmol/l:

A Congenital adrenal hyperplasia is a likely diagnosis. B Loss of skin turgor is likely to be present. C The dehydration should be corrected over the next 2-4 hours. D Bloody diarrhoea suggests a bacterial cause. E Initial fluid replacement should be with 0.45% saline.

(False) (False) (False) (True) (True)

Comments: The most likely cause of hypernatraemic dehydration in infants and children is infective diarrhoea. This is likely to be bacterial if the stool contains blood. The skin may have a doughy consistency in hypernatraemic dehydration. Rapid correction of the increased serum osmolarity results in intracellular fluid accumulation, which may lead to cerebral oedema. This can be avoided by slowly correcting the fluid depletion (over 24-48 hours) and ensuring that the serum sodium is lowered at a rate of no greater than 10 mmol/l/24 hours.

Pediatrics

9) The following are associated with neonatal hypoglycaemia:

A Down's syndrome. B Reye syndrome. C nesidioblastosis. D Galactosaemia. E Polycythaemia.

(False) (False) (True) (True) (True)

Comments: Causes of neonatal hypoglycaemia can be classified as follows: 1. Hyperinsulinism (maternal diabetes - islet cell hyperplasia - Beckwith-Wiedemann syndrome Nesidioblastosis - islet cell adenoma). 2. Decreased production (prematurity - ntrauterine growth retardation - glycogen storage disease glactosaemia). 3. Increased utilization (hypothermia polycythaemia asphyxia sepsis). 4. Inborn errors (maple syrup urine disease (MSUD), - proprionic acidaemia, tyrosinaemia). 5. Endocrine causes (panhypopituitarism, - adrenal insufficiency). 6. Miscellaneous (maternal drugs (beta agonists/blockers, thiazides), - post-exchange transfusion).

10) The following statements regarding an atrioventricular septal defect (AVSD) are correct:

A Most defects occur in children with Down's syndrome. B Congestive heart failure usually occurs by 4-8 weeks of age. C Without surgical treatment most infants will die between the ages of 2 and 3 years. D An ECG rarely shows right ventricular hypertrophy (RVH) or right bundle branch block (RBBB). E A loud 3-4/6 pan systolic murmur is audible along the left sternal edge (LSE).

(False) (True) (True) (False) (True)

Comments: AVSDs account for about 2% of all congenital heart disease. Children with Down's syndrome only account for about 30% of all AVSDs. Heart failure occurs early and requires aggressive medical treatment. Recurrent chest infections are common. Surgical intervention is essential for long-term survival. An ECG shows left anterior hemi block (superior QRS axis), a QRS axis between -40 degrees and -150 degrees. RVH or RBBB is present in all patients and left ventricular hypertrophy may occur. First-degree atrioventricular block with a prolonged PR interval is also common.

Pediatrics

11) Which of the following statements regarding Napkin rashes are correct?

A Always caused by ammonia. B Less common in babies fed a soy protein milk. C Often successfully treated by exposure to air. D Evidence of early infantile eczema. E More common in boys.

(False) (False) (True) (False) (False)

Comments: Causes of napkin rash include contact dermatitis, which may produce ammonia and this may burn the skin. Infection with bacteria and candida yeasts may cause nappy rash, as can psoriasis and atopic dermatitis affecting the nappy area. Nappy rash is not an indicator of infantile eczema, and it is not more common in boys, nor is it less common in soy fed infants. Treatment is best achieved by prevention by frequent (disposable absorbent) nappy changing, and fluid feeding early in the day to lessen night time urination. Anti-fungal and Lotrimin lotions may also be useful.

12) Cyanosis in the first few days of life is likely with: A Transposition of the great vessels B Tetralogy of Fallot C Tricuspid artresia D Atrial septal defect E Coarctation of the aorta (True) (False) (True) (False) (False)

Comments: Tricuspid atresia and Transposition of the great arteries are both cyanotic congenital heart diseases and present in the immediate newborn period. Tetralogy of Fallot can manifest at any time and sometimes in early childhood. ASD is a left to right shunt and presents a few months after birth. Cyanosis develops if there is Eisenmengers syndrome. Coarctation is a narrowing of the aortic arch.

13) The following statements relating to the development of the human brain are correct:

A The primitive brain vesicle appears at the end of the 4th week. B The cerebral hemispheres result from evagination of the prosencephalon. C The diencephalon is adjacent to the IIIrd ventricle. D The cerebellum starts as a projection into the IVth ventricle.

(True) (True) (True) (True)

Pediatrics

E The nervous system develops from the ventral ectoderm at about the 16th day.

(False)

Comments: Development of the brain begins with the appearance of the brain vesicles in the early part of the embryonic period. Evagination of the prosencephalon gives rise to the cerebral hemispheres. The primitive hemispheres and lamina terminalis constitute the telencephalon (endbrain). The diencephalon (epithalamus, thalamus and hypothalamic apparatus) is adjacent to the IIIrd ventricle. The cerebellum develops from a projection into the IVth ventricle. The nervous system dedelops from Dorsal ectoderm at the 16th day. The neural tube begins to fuse at the end of the 3rd week. Rostral closure occurs on the 24th and caudal closure on the 26th day.

14) A small ventricular septal defect is associated with: A A pansystolic murmur heard best at the end of the sternal edge B A high risk of infective endocarditis C Fixed splitting of the second sound D Cyanosis E A mid-diastolic murmur at the apex (True) (True) (False) (False) (False)

Comments: A small VSD is classically associated with the holosysolic murmur at the left sternal edge; The risk of infective endocarditis is greater in haemodynamically insignificant defects. The second heart sound may be obscured by the murmur, but is normally split. Cyanosis occurs if there is Eisenmengers syndrome. A mid diastolic murmur at the apex is associated with a moderate sized shunt and increased flow through the left atrium.

15) Regarding achondroplasia: A Usually a hereditary condition B Have proportionate trunk and limb lengths C The individual usually has decreased mental development D Could lead to lumbar lordosis E X-ray shows short dense bones with flaring of the ends (True) (False) (False) (True) (True)

Comments: Achondroplasia is usually a hereditary condition but may also occur sporadically. It particularly affects the pre-bone cartilage of long bones at birth resulting in Dwarfism. The affected individuals have a normal trunk length but short stumpy limbs all of the same length. They also have a large head with a saddle nose. Adult dwarfs have increasing lumbar lordosis, bow legs, and shortened proximal arms and legs. Intelligence level is usually normal. X-rays show short dense bones with flared ends with wide epiphysis.

Pediatrics

16) In children with Fallots tetralogy A Cyanosis is always present shortly after birth B Heart failure is a common complication C Auscultation reveals a systolic ejection murmur in the pulmonary area and a single second heart sound D Squatting relieves cyanosis after exercise E Functional anaemia may be present in spite of a haemoglobin concentration which is normal to age. (False) (False) (True) (True) (True)

Comments: Cyanosis usually occurs 3-6 months after birth and may present in the first few years of life. Heart failure is not generally a feature of this condition. Systolic ejection murmur is present and the intensity is inversely proportional to the degree of Rt ventricular outflow obstruction. Pulmonic valve closure is not heard and hence single S2. Squatting is uniquely characteristic of a right to left shunt, and relieves cyanosis in exercise by decreasing systemic venous return. Prolonged cyanosis causes secondary polycythaemia which increases oxygen carrying capacity, therefore normal Hb concentration may cause symptomatic anaemia.

17) Galactosaemia A is an autosomal recessive condition B cataract does not occur C can cause jaundice D can cause hypoglycaemia in the neonate E can be improved by excluding milk from the diet (True) (False) (True) (True) (True)

Comments: 2. cataracts do occur, usually bilaterally, and are reversible. 3. An important cause of prolonged jaundice in the neonate 4. A well recognized cause of hypoglycaemia 5. galactose is ingested as lactose in milk

18) The following conditions cause an increased anion gap:

A Diabetic ketoacidosis. B Proximal renal tubular acidosis. C Aspirin poisoning. D Lactic acidosis. E Acute renal failure.

(True) (False) (True) (True) (True)

Pediatrics

Comments: The anion gap is normally about 12 mmol/l. It is the difference between the cations (principally sodium and potassium) and the anions (principally consisting of chloride and bicarbonate ions). A rise in the gap shows the presence of an unmeasured cation or anion. In diabetic ketoacidosis this is beta-hydroxybutyrate and acetoacetate and in renal failure retained phosphate and sulphate. In proximal renal tubular acidosis the defect is one of bicarbonate loss and the anion gap remains unchanged.

18) A strawberry naevus is: A Usually present at birth. B Best treated immediately it is recognized. C Pulsatile. D Often present at the nape of the neck. E A capillary haemangioma. (False) (False) (False) (False) (True)

Comments: A strawberry naevus is commonly present at birth but may not present till later on. Treatment is by laser or tattooing at a much later date. The majority occur in the regions supplied by the fifth cranial nerve. It is non pulsatile The other names are naevus flammeus and capillary haemangioma.

19) The following statements regarding classical phenylketonuria (PKU) are correct:

A Eczema is characteristically present in young children with the disease. B Phenylalanine is not detectable in cord blood in newborn infants with PKU. C Dietary restrictions are stopped after the age of 10 years. D A pregnant woman with PKU will not put her baby at risk if she is on an unrestricted diet. E Vomiting is an early presenting symptom of the disease.

(True) (True) (False) (False) (True)

Comments: Classical PKU is the result of a deficiency of phenylalanine hydroxylase. Eczema is often present in younger children, which disappears as the child grows older. Phenylalanine levels may rise to levels that are detectable by the Guthrie test as soon as 4 hours after birth, even before the ingestion of protein containing milk. It is recommended that a diet low in phenylalanine is continued for life. It has been shown that changes in white matter on stopping dietary restrictions are reversible on resumption of a low phenylalanine diet. It is essential that women with PKU should be on a diet low in phenylalanine in the preconception period to avoid damaging a normal fetus.

Pediatrics

20) Which of the following is/are recognized causes of alopecia in children:

A Alopecia areatea. B Ectodermal dysplasia. C Naevus flammeus. D Ringworm. E Trichotillomania.

(True) (True) (False) (True) (True)

Comments: Alopecia areata is autoimmune destruction of hair follicles. Ectodermal dysplasia is characterized by absent or deficient function of at least 2 derivatives of the ectoderm eg teeth and hair. Trichotillomania is an impulsive need to pull out hair from scalp and eyelashes. Ringworm is a fungal infection which results in hair loss. Naevus flammeus is another name given to a capillary haemangioma or port-wine stain.

21) The following infections are caused by gram-positive organisms: A Whooping cough. B Typhoid. C Diphtheria. D Ophthalmia neonatorum. E Tetanus. (False) (False) (True) (False) (True)

Comments: Causative organisms are as follows: Whooping cough - Bordetella pertussis Typhoid Salmonella typhi Diphtheria - Corynebacterium diphtheriae Ophthalmia neonatorum Neisseria gonorrhoeae Tetanus - Clostridium tetani N.B: TTT of gram +ve organisms need 10 day, gram ve organisms need 14 days.

Gram +ve cocci Staph. aureus Strep. pyogenes Strep. pneumoniae Strep. viridans

Gram +ve bacilli Clostridium sp. Corynebacterium diphtheriae Listeria sp. Bacillus anthracis

Gram -ve cocci N. meningitidis N. gonorrhoeae

Gram -ve bacilli E. coli Salmonella sp. Shigella sp. Proteus sp. Klebsiella sp. Haemophilus sp. Bordetella pertussis Pseudomonas sp.

Pediatrics

22) A 5-year child presents with progressive weakness. The following make a diagnosis of Guillain-Barre syndrome more likely:

A Isolation of Campylobacter jejuni from the stool. B leg pain C dilated pupils D The presence of over 50 lymphocytes/ml in the cerebrospinal fluid (CSF). E Asymmetric weakness of the lower limbs.

(True) (True) (False) (False) (False)

Comments: Guillain-Barre syndrome (GBS) has been shown to be associated with C. jejuni infection. Pain is often a predominant feature of GBS and is the presenting symptom in up to 20% of cases. Children who present with pain often have significant emotional lability associated with vomiting and a headache. Pupillary abnormalities are not seen in GBS and when present suggest diptheria or botulism. The presence of over 50 lymphocytes/ml of CSF suggests polio or central nervous system lymphoma. In GBS the protein level in the CSF is typically 80-200 mg/dl. Paralysis of the lower limbs is usually symmetrical and is frequently associated with distal paresthesias and numbness but in poliomyelitis asymmetrical paralysis (in polio. No sensory involvement).

23) Parents of autistic children: A Commonly belong to higher social class. B Have an increased incidence of schizophrenia. C Should be warned about 1 in 4 risk of autism in future children. D Can look forward to their child attending normal schooling. E Commonly blame themselves unrealistically for their childs behaviour. (True) (False) (False) (False) (True)

Comments: A bias towards higher occupational class has been found in some studies. There is no increased prevalence of schizophrenia in the parents Between one third and two thirds of children are severely mentally retarded. The cause of autism is possibly related to complex genetic and environmental factors, specific risk factors for autism have not been identified.

Pediatrics

10

24) The following occur in childhood asthma. A cough as the predominant symptom B stridor C increased airways resistance D decreased lung compliance E decreased functional residual capacity (True) (False) (True) (False) (False)

Comments: Persistent cough, especially at night, is often a presenting feature of childhood asthma. Stridor is a feature of upper airways obstruction and therefore does not occur in asthma. This leads to air trapping and increased Functional Residual Capacity (FRC). Compliance is normal.

25) The following viral infections are paired with their appropriate incubation periods:

A Measles - 10-12 days. B Rubella - 7-14 days. C Varicella zoster - 7-10days. D Mumps - 14-24 days. E Hepatitis B - 60-110 days.

(True) (False) (False) (True) (True)

Comments: The incubation period of rubella is 14-21 days. Varicella zoster virus has an incubation period of 13-17 days.

26) The following statements are correct: A Insulin is derived from C-peptide. B Glucagon stimulates insulin release. C C-peptide levels are increased in endogenous hyperinsulinaemia. D Insulin decreases absorption of glucose from the gut. E Catecholamines antagonize the actions of insulin. (False) (True) (True) (False) (True)

Comments: Insulin is derived from proinsulin, which is cleaved to form insulin and C-peptide. C-peptide can be measured to distinguish between endogenous hyperinsulinaemia and exogenously administered insulin. Insulin inhibits glucagon release, whereas glucagon stimulates insulin release. Insulin has no effect on the absorption of glucose from the gut. Catecholamines inhibit the action of insulin.

Pediatrics

11

27) The following statements apply to a cephalhaematoma in the newborn.

A It usually manifests immediately after birth. B The haematoma is formed beneath the periosteum. C A common site is over the parietal bone. D The treatment of choice is aspiration. E Mother can be told it will disappear by the age of four weeks.

(False) (True) (True) (False) (False)

Comments: It usually manifests several hours after delivery. The haematoma occurs as a result of damage to capillaries beneath the periostium.They usually require no treatment, but may take up to 3 months to resolve. (beneath = under) 28) Regarding congenital dislocation of the hip: A The incidence is 2.5 to 20 per 1000 live births. B The Ortolani-Barlow manoeuvre is highly sensitive and specific. C It may be associated with polyhydraminos. D The male: female ratio is 2:1. E A family history is frequently found. Comments: The incidence of unstable hips at birth is approximately 2%, of which 90% are dislocatable, and 10% dislocated. There is a 6:1 female to male ratio, left side > right side, and there may be a family history. It is commoner in first children, breech presentations (50% of infants with CDH), oligohydraminos, and may be associated with other deformities such as (talipes = club foot) The Ortolani manoeuvre: detects a dislocated hip, as the hip is 'clunked' back into position. Barlow's test: detects a dislocatable hip by pressing posteriorly and internally rotating it, the hip can be dislocated. (True) (False) (False) (False) (False)

The introduction of routine testing of the hips has not decreased the incidence of established dislocation. This may be because of poor examination technique, a hip that becomes unstable only when weight bearing begins, or an acquired dislocation with age. Late presentation of CDH may include asymmetry of skin folds, limited abduction of hips, shortening of the thigh, with knees at different level when the patient is supine and the legs are flexed (Galeazzi sign), and limping.

Pediatrics

12

29)The following are side-effects of sulphasalazine: A Reversible neutropenia. B Acute pancreatitis. C Methaemoglobinaemia. D Permanent oligospermia. E Folate deficiency. (True) (True) (True) (False) (True)

Comments: Sulphasalazine is a chemical combination of sulpha-pyridine and 5-aminosalicylic acid (5-ASA) and is useful in the treatment of ulcerative colitis. 5-ASA is the active moiety and sulphapyridine acts as a carrier to the colonic site of action. Common side-effects are nausea, vomiting, epigastric pain, headache and rash, occasionally fever, anaemia, reversible neutropenia, folate deficiency and reversible oligospermia. Rarer side-effects include methaemoglobinaemia, pancreatitis, hepatitis, thrombocytopenia, agranulocytosis, aplastic anaemia and Stevens-Johnson syndrome.

30) Pulmonary hypertension: A Causes a diminished pulmonary second sound B Causes left ventricular hypertrophy C Commonly complicates large ventricular septal defects D Results in a diminished main pulmonary artery shadow on chest x-ray E Is a feature of the Eisenmengers syndrome (False) (False) (True) (False) (True)

Comments: Pulmonary hypertension causes a loud P2 which may be palpable and causes right ventricular hypertrophy. Large ventricular defects are associated with increased pulmonary vascular resistance and clinical signs of pulmonary hypertension. The main pulmonary artery segment is prominent on the chest x-rays and the vascularity in the outer one third of the lung fields is reduced. Eisenmengers syndrome is characterized by pulmonary hypertension following reversal of a left to right shunt. 31) Vesicles are characteristic in A Exfoliative dermatitis B Erythema nodosum C Dermatitis herpetiformis D porphyria cutanea tarda E Steven's-Johnson syndrome (False) (False) (True) (True) (True)

Pediatrics

13

Comments: Vesicles occur in erythema multiforme (Steven's Johnson), chicken pox, herpes zoster, Porphyria cutanea tarda, dermatitis herpetiformis and pemphigus / pemphigoid. Not erythema nodosum - raised tender nodules usually on the shins nor exfoliative dermatitis.

32) Regarding Vitamin K: A Modern formula feeds contain sufficient Vitamin K, but babies who are entirely or mainly breast fed should receive supplements. B There is unequivocal evidence to suggest that intramuscular Vitamin K at birth increases the risk of childhood cancer. C The ideal oral Vitamin K regimen is 3 doses given at birth, 7days and 28 days. D Newborn infants have very low levels of Vitamin K which is needed for normal clotting. E Deficiency most often presents at 3 months of age.

(True) (False) (True) (False) (True)

Comments: Deficiency of Vitamin K can lead to haemorrhage at a variety of sites. This may occur during the first few days (early haemorrhagic disease) or within the first 3 months of life (late haemorrhagic disease). The latter is much more serious because of the potential for intracranial bleeds, leaving 30% dead and 40% seriously handicapped. Modern formula feeds are supplemented with Vitamin K, but babies who are breast fed should definitely receive supplements. There is no definite link of intramuscular Vitamin K with childhood cancer, and the larger studies suggest that there is no link at all. Oral regimens of prophylaxis are likely to be suboptimal for compliance particularly from the third dose (in one study measured there was only 40%).

33) In children: A Moles are usually present at birth. B Giant hairy moles may undergo malignant change. C Malignant melanoma is very rare under the age of 10 years. D All moles on the palms and soles should be excised prophylactically. E Moles can appear at any time after birth. (False) (True) (True) (False) (True)

Comments: Only 1% of moles are present at birth and termed congenital naevi. 3-10% of giant congenital moles (9cm on head and 6cm on the body) can give rise to melanoma. Small moles occurring on the palms and soles may be entirely benign and should be observed for signs of change. Moles can appear throughout life.

Pediatrics

14

34) Neuroblastoma A is an embryonic tumour B usually has a familial incidence C metastasises most commonly to the lungs D is easily eradicated by radiotherapy E is a secreting tumour (True) (False) (False) (False) (True)

Comments: Neuroblastoma is a tumour derived from neural crest tissue and so, is properly regarded as an APUD tumour. The most common sites are: 50% adrenal medulla, 25% abdominal sympathetic ganglia, 20% chest, 5% pelvis, 5% neck. The tumour may secrete catecholamines hence urine VMA concentrations are elevated Spontaneous remission is well recognized in this condition. Extra abdominal sites generally have a better prognosis.

35) Hyponatraemia can occur in: A Congenital adrenal hyperplasia B Gastroenteritis C Diabetes insipidus D Diabetes mellitus E Severe pneumonia (True) (True) (False) (True) (True)

Comments: Hyponatraemia occurs in congenital adrenal hyperplasia due to failure to synthesise cortisol and mineralocorticoid. Gastroenteritis may cause loss of sodium and water through and hence hyponatraemic dehydration. Diabetes insipidus causes hypernatraemia. Diabetes mellitus may cause osmotic diuresis and associated tubular sodium loss. Pneumonia may cause inappropriate ADH secretion and hyponatraemia results.

36) The following conditions may be detectable by growth monitoring: A Hyperthyroidism B Hypothyroidism C Pseudohypoparathyroidism D XYY Syndrome E Insulin dependent diabetes mellitus Comments: (False) (True) (False) (False) (False)

Pediatrics

15

Benefits of growth monitoring include: Early detection of conditions such as: Hypothyroidism. Growth hormone insufficiency. Syndromes: Turners, Russell-Silver, Noonan's, skeletal dysplasias. Growth impairment e.g. coeliac disease, inflammatory bowel disease or chronic renal failure. Intracranial tumours. Short normal children. Children with short stature. Health promotion: impaired growth may be associated with child abuse or neglect for example. Focus of interest for parents.

Public health aspects: Secular trend of increasing growth. Linking growth patterns in fetal life and early infancy with adult patterns of disease. Link between height and social circumstances.

37) The following are recognized features of Turner's syndrome: A Low birth weight. B Ovarian cysts. C Low plasma luteinizing hormone (LH) level. D High plasma follicle-stimulating hormone (FSH) level. E Lymphoedema. (True) (False) (False) (True) (True)

Comments: The features of Turner's syndrome are: low birth weight short stature short, webbed neck with low hair line and loose skin folds typical facies (prominent ears, small jaw) high arched palate hypoplastic nails oedema of hands and feet shield-shaped chest cubitus valgus pigmented naevi low IQ hearing impairment coarctation of the aorta renal tract abnormalities streak ovaries high basal LH and FSH.

Pediatrics

16

38) The following conditions present with cyanosis in the newborn: A Tricuspid atresia. B Ebstein's anomaly. C Atrioventricular septal defect (AVSD). D Tetralogy of Fallot. E Transient myocardial ischaemia. (True) (True) (False) (True) (False)

Comments: Cardiac conditions that present with cyanosis in neonates include: 1. Transposition of the great arteries (TGA) 2. Pulmonary atresia 3. Severe pulmonary stenosis 4. Severe tetralogy of Fallot 5. Tricuspid atresia 6. Ebstein's anomaly 7. Total anomalous pulmonary venous drainage (TAPVD) 8. Hypoplastic left heart 9. Truncus arteriosus Babies with AVSD usually become symptomatic after the first 2 weeks of life with symptoms and signs of congestive cardiac failure. Transient myocardial ischaemia is often a feature of perinatal asphyxia. It may cause varying degrees of congestive cardiac failure or may only be evident as temporary ECG changes.

39) Concerning congenital heart disease: A ASD is the commonest malformation at birth B Osteogenesis imperfecta is associated with aortic stenosis C Ebstein's anomaly is associated with maternal exposure to lithium carbonate D congenital complete heart block is usually associated with Anti-Ro antibodies in the mother E hypoplastic left heart syndrome is characterised by a large, dilated left ventricle (False) (False) (True) (False) (False)

Comments: A- VSD is the commonest at 30%, ASD is 10%. B- Aortic regurgitation. C- If exposed in the first trimester. D- In the vast majority of cases the cause is not known but rarely it may be caused by autoimmune disease, particularly Anti-Ro antibodies, in the mother. E- The left sided chambers fail to develop and blood enters the systemic circulation from the right ventricle via the pulmonary artery and a patent ductus arteriosus.

Pediatrics

17

40) Which of the following is true regarding rheumatoid arthritis. A Primarily affects the articular cartilage. B Is associated with the HLA antigens DR4 and DW4. C Occurs more often in women. D The cervical spine is commonly involved. E Extra-articular manifestations occur in 20% of patients. (True) (False) (True) (False) (True)

Comments: Rheumatoid arthritis is a disease of the synovial membrane. It results in joint destruction, pannus formation and periarticular erosion. DR4 and DW4 are associated with diabetes. The female to male ratio is 3:1. The hand, elbows and knees are the commonest joints involved. Extra-articular manifestations occur in 20% and include keratoconjuctivitis sicca, episcleritis, pulmonary nodules, pleural effusions, pericarditis and subcutaneous rheumatoid nodules.

41) Club-foot (talipes equino-varus-tev) deformity in a newborn: A Is most commonly postural in origin. B Has a higher incidence in babies born by the breech than the vertex. C Needs open reduction in most instances. D Is more common in Chinese than other races. E Occurs in association with spina bifida cystica only if there is accompanying hydrocephalus. (True) (True) (False) (False) (False)

Comments: Postural talipes is most common and can be passively corrected. Spina bifida cystica refers to meningocele and myelomeningocele; these are associated with other skeletal deformities but not hydrocephalus. It is more common in Polynesians rather than Caucasians and not in Chinese.

42) In examination of the cardiovascular system in childhood: A A venous hum is best heard with the child lying flat B Ventricular septal defect may not be clinically detectable in the first few days of life C Intercostals recession is useful evidence for a left to right shunt D Normal femoral pulses at birth exclude co-arctation of aorta E Peripheral oedema is a valuable sign of heart failure in infancy (False) (True) (True) (False) (False)

Comments: Venous hum is best heard above the Rt clavicle with the child sitting up. VSD not usually detected till after one month of age when there is normal decrease in pulmonary vascular

Pediatrics

18

resistance allowing left to right shunting to take place. Preductal coarctation of the aorta is associated with other cardiac defects and if there is an associated PDA then pulses in the lower extremities will be preserved. Post ductal coarctation is associated with diminished or absent femoral pulses. Right sided venous congestion characteristically causes hepatosplenomegaly in infants, oedema and ascites occur less frequently.

43) A presumed male child of 6 years has apparent hypospadias and no palpable testes. The following would be in favour of the child being female:A buccal mucosa chromatin -ve B high urinary 17-ketosteroid excretion C pubic hair present D abnormally tall stature E retarded bone maturation (False) (True) (True) (True) (False)

Comments: a - females have a barr body b,c,d - suggest CAH and virilization e - not a feature at this age, would expect advanced bone age in a female.

44) Haematological consequences of splenectomy for traumatic rupture of the spleen in a 2 year old include A An increased peripheral white cell count B Heinz bodies on the blood film C Howell-Jolly bodies on the blood film D the development of target cells E increased susceptibility to overwhelming septicaemia Comments: + siderocytes, inc. plt initially (True) (True) (True) (True) (True)

Pediatrics

19

45) In the management of 2-year child with burns: A Once damp QRS dressings are placed over the burns, they should not be removed till the child reaches the burns unit B Lund and Browder chart accurately estimates the percentage of burns C A urine output of 0.5 ml/kg/hr is an indicator of adequate hydration D Ionotropes may be required in burns > 10% total body surface area E Once the burn wounds are dressed, they should be left undisturbed for five days

(False) (False) (False) (True) (False)

Comments: Damp dressings can cause hypothermia in children; therefore, continuous assessment of the child is essential. Lund and Browder chart (rule-of-nines) is useful in adult burns; in children, head makes up 14% of the total body surface area and the legs make up only 14%. Urine output of at least 1ml/kg/hr is required in this age group; 0.5 ml/kg/hr is adequate in adults. Ionotropes such as dopamine may be required if the urine output is not satisfactory. The wound depth should be estimated again after 24-48 hrs, since a partial thickness burn may progress to a full-thickness burn, thus altering the management plan.

46) Which of the following statements is correct regarding osteomalacia?

A Is due to vitamin A deficiency B Can cause a distal myopathy C May present with pseudo-fractures D Serum calcium is increased E A bone biopsy would show an increase in mineralized osteoid

(False) (True) (True) (False) (False)

Comments: Osteomalacia is due to vitamin D deficiency T It can result from malabsorption, renal disease (familial hypophosphataemic rickets), chronic renal failure and anticonvulsant therapy (e.g. phenytoin). It often presents with bone pain and a proximal myopathy but a distal myopathy may feature. Serum calcium is low. A bone biopsy would show an increase in the amount of unmineralised osteoid.

47) Which of the following is/are true regarding osteoid osteoma A It is a benign bone tumour B The iliac crest is the commonest site involved C Presents with severe pain that is typically relieved by aspirin D Radiologically appears as an osteosclerotic lesion E Local excision or currettage is curative (True) (False) (True) (False) (True)

Pediatrics

20

Comments: Osteoid osteoma is a benign tumour that mainly affects young adults. The femur and tibia are the commonest bones involved. Pain is often the presenting feature and relief by asprin is almost diagnostic. Local surgery is curative.

48) A complete unilateral facial hemiparesis may be caused by: A An intracranial tumour (False) (True) (False) (False) (False)

B Birth injury C Phenothiazine toxicity D Myasthenia gravis E Cerebellar atrophy

Comments: The facial nerve consists of 2 parts. The larger motor components supplies all the muscles of facial expression, while the smaller part (nervous intermedias) comprises the sensory and parasympathetic branches of taste from the anterior two thirds of the tongue, with efferent fibres to the lacrimal, submaixillary, and sublinguinal salivary glands. Unilateral upper motor neurone lesions (above the level of the ponds) cause weakness more in the lower than in the upper part of the face, since upper facial structures receive bilateral innovation. A unilateral lower motor neurone lesion such as Bell's Palsy, affects the eyes as well. An intracranial tumour can, therefore, cause complete weakness only when both sides are affected. A forceps injury may compress the facial nerve.
49) Which of the following is/are true regarding osteoid osteoma A It is a benign bone tumour B The femur and tibia are the commonest bones involved C Presents with severe pain that is typically relieved by aspirin D Radiologically appears as a radiolucent lesion surrounded by dense bone E Local excision or currettage is curative (True) (True) (True) (True) (True)

Comments: Osteoid osteoma is a benign tumour that mainly affects young adults. The femur and tibia are the commonest bones involved. Pain is often the presenting feature and relief by asprin is almost diagnostic. Xray reveals a radiolucent nidus with a dense rim. Local surgery is curative.

Pediatrics

21

Osteoid osteoma Rare, 2:1 M:F Prox phalanx or carpus Pain relieved by aspirin or NSAIDs Xray - eccentric cortical sclerosis & radiolucent nidus Rx curettage using image intensifier 13% recurrence

50) Which of the following is true concerning Meningococcal meningitis?

A if it is suspected, benzyl penicillin should be administered prior to hospital admission B vaccination is available against all strains of meningococcus C sensorineural deafness has been shown to be the most common permanent sequela D it has been shown to be more common in those children in contact with cigarette smoke E Rifampicin has been shown to be the treatment of choice for close contacts

(True) (False) (True) (True) (True)

Comments: a - This may be life saving. b - Only against groups A and C. Most patients make a full recovery from meningococcal meningitis but deafness is one of the commoner sequelae. Risk factors include overcrowding, parental smoking. Rifampicin should be administered to close contacts as prophylaxis. (Cornwall Trainers)

60) The following applies to congenital heart defects: A Most deaths occur in the first year of life. B There is an increased incidence of heart disease in children who have had rubella. C A ventricular septal defect may close spontaneously as a child grows. D Downs syndrome is frequently associated with Fallots tetralogy. (True) (False) (True) (False)

E Tricuspid atresia is the commonest cause of cyanotic heart disease in the first month (False) of life.

Pediatrics

22

Comments: Approximately 51% of deaths occur in infants aged under 1 year and only 7% in age group 1-4. Maternal rubella and not childhood disease is associated with cardiac defects eg pulmonary alular disease, PDA and VSD. Downs syndrome is frequently associated with endocardial cushion defects, which occur in 25% of children and not tetralogy of fallot. In the first month of life transposition of the great arteries is the commonest cause of cyanotic congenital heart disease, but overall tetralogy of fallot is the commonest. 61) The following are common signs of heart failure in infancy: A Breathlessness. B Basal crepitations. C Tachycardia. D Raised JVP. E Enlarged liver. (True) (False) (True) (False) (True)

Comments: Tachypnoea and dyspnoea, tachycardia and hepatomegaly are common clinical signs of heart failure. Cough and wheeze may occur and basal crepitations are uncommon. Raised jugular venous pressure is an unreliable clinical sign in infancy.

62) A chest x-ray showings signs of increased pulmonary blood flow (increased pulmonary plethora) is compatible with a diagnosis of:

A Ventricular septal defect B Patent ductus arteriosis C Pulmonary tetralogy D Fallots tetralogy E Coarctation of the aorta

(True) (True) (False) (False) (False)

Comments: VSD will cause increased blood flow through the Rt ventricle and pulmonary trunk, as will PDA which connects the left pulmonary artery and descending aorta. In fallots and pulmonary tetralogy there will be decreased flow due to infundibular stenosis. Coarctation, a narrowing of the aorta near the left subclavian vein is associated with normal pulmonary vasculature.

Pediatrics

23

63) Which of the following statements applies to Cleft Palate? A Carries an increased risk of middle ear infections. B Has a recognized association with micrognathia. C May be associated with maternal anticonvulsant therapy. D Is usually repaired before the age of three months. E Is associated with subsequent speech disorders. (True) (True) (True) (False) (True)

Comments: Hearing impairment and repeated ear infections may occur, as well as other craniofacial abnormalities. Anti-convulsants have been associated with cleft lip & palate, congenital heart disease, CNS & skeletal abnormalities. Surgical methods such as bone grafting one are carried out in childhood and are most successful in patients under 10 years of age and as early as 5 to 6 years as the front incisor teeth are erupting. Delayed language and articulation development is common. 64) In children with Down's syndrome: A The infant is usually hypertonic. B Asthma is more common. C Most mothers are over 35 years of age. D Karyotyping is always indicated. E Brachycephaly is characteristic. (False) (False) (False) (True) (True)

Comments: Infants with Down's syndrome (trisomy 21) are commonly hypotonic at birth. Brachycephaly a consistent clinical feature. Owing to cases of Down's syndrome where a translocation has occurred, karyotyping should always be carried out in order to assess the recurrence risk in parents and other family members.

65) A 3-year-old child presents to hospital with a 10-day history of abdominal pain and bloody diarrhoea. On examination the child appears anaemic and jaundiced. The following laboratory findings are consistent with a diagnosis of haemolytic uraemic syndrome (HUS):

A Positive Coombs' test. B Normal osmotic fragility test. C Low reticulocyte count and anaemia. D Increased fibrin degradation products. E Raised lactate dehydrogenase levels in the serum.

(False) (True) (False) (True) (True)

Pediatrics

24

Comments: HUS is the most common cause of acute renal failure in an otherwise healthy child. It presents with bloody diarrhoea that may have been present for 1-3 weeks. E. coli 0157:H7 is the most commonly implicated organism. Anaemia results from haemolysis of red blood cells (RBC) in the microvasculature. Laboratory findings include: reticulocytosis, unconjugated hyperbilirubinaemia, increased lactate dehydrogenase (released from RBC), decreased haptoglobulins, a negative Coombs' test and normal red cell enzymes and osmotic fragility. A neutrophil leucocytosis usually occurs. If the leucocytosis is > 20 x 10E9/l, there is a poorer prognosis.

66) In bulimia nervosa: A Male cases do not occur. B Weight is usually normal. C Dental problems are a recognized association. D Diabetes mellitus is a recognized association. E The presence of amenorrhoea indicates a worse prognosis. (False) (True) (True) (True) (True)

Comments: Bulimia nervosa is an eating disorder (more common than anorexia nervosa) in which an intense preoccupation with food and weight is accompanied by episodes of binge eating and self induced vomiting. Over 90% of cases are female. Recurrent vomiting may cause erosion of dental enamel and a worse prognosis occurs in patients with menstrual irregularities, biochemical disturbances and concurrent depressive.

67) Oral rehydration therapy: A works by passive absorption of glucose; B Is effective in 50% of moderately dehydrated infants. C Should not be used in hypernatraemic dehydration. D Contains 5 mmol/l of potassium. E Can be made up safely at home using salt and sugar. (False) (False) (False) (False) (False)

Comments: Oral rehydration therapy works as glucose is actively absorbed across the lumen of the intestine in a cotransport system with sodium. It is effective in up to 90% of infants in the UK with moderate dehydration. Oral rehydration therapy is the treatment of choice for hypernatraemic dehydration as there is a more gradual correction of electrolyte imbalances than using intravenous therapy. All oral rehydration solutions contain potassium concentrations of 20 mmol/l. Solutions made up using home ingredients should be discouraged because of the possibilities of making hypertonic solutions.

Pediatrics

25

68) A five year-old child presents with a high swinging fever. A diagnosis of systemic onset juvenile chronic arthritis (JCA) is more likely if the following features are present: A The child is female. B HLA studies show that the child is HLA-B27. C The child is rheumatoid factor positive. D Iridocyclitis is present. E Hepatospenomegaly is present. (False) (False) (False) (False) (True)

Comments: Systemic onset JCA is typified by a persistent, high swinging fever, a 'rheumatoid' rash, hepatosplenomegaly, lymphadenopathy and pleuritis. It has an equal sex distribution. HLA associations with the condition are not yet know. HLA-B27 is seen in about 90% of children seen with juvenile ankylosing spondylitis. Children with systemic onset JCA are rheumatoid factor negative and do not develop iridocyclitis. In pauciarticular JCA up to 50% of those who are antinuclear factor positive will develop chronic iridocyclitis.

69) The following features make a diagnosis of ulcerative colitis more likely than one of Chrohn's disease:

A Perianal lesions. B Crypt abscesses on biopsy. C Granulomata. D Pyoderma gangrenosum. E Presence of a fistula.

(False) (True) (False) (True) (False)

Comments: Perianal lesions T (tags, abscess, and fistulae) and the development of a fistula are highly suggestive of Crohn's disease (terminal ileum). Histological features of ulcerative colitis (colon) include mucosal disease with crypt abscesses, decreased goblet cells and an infiltration of polymorphs. Granulomata are not a feature of ulcerative colitis but pyoderma gangrenosum does occasionally develop. (non-caseating granulomas) Tin the biopsy of chrohns disease. 70) Intra-uterine infection may result in: A Short stature. B Intraventricular haemorrhage. C Retinal vein thrombosis. D Patent ductus arteriosus. E Mental handicap. (True) (False) (False) (True) (True)

Pediatrics

26

Comments: Intrauterine infection results in prematurity, growth failure and cerebral palsy. Intraventricular haemorrhage is inversely proportional to birth weight and gestational age and related to neonatal sepsis. Retinal vein thrombosis is not associated. Patent ductus arteriosus can follow prematurity or rubella infection.

71) Acute osteomyelitis in children: A Is commonly caused by Streptococcus pyogenes infection B Usually commences in the epiphysis C Blood cultures are always positive D A sequestrum and involucrum are classical radiological features E Joint fluid aspiration may be necessary to make a diagnosis (False) (False) (False) (False) (True)

Comments: Acute osteomyelitis is commonly caused by Staphyloccus aureus infection. It may be acquired by haematogenous route, direct skin puncture following injury or from infection spreading from adjacent soft tissues. It usually commences in the metaphyseal region of long bones, but at later stages could affect the growth plate and epiphyseal cartilage. The child with acute osteomyelitis usually presents after several hours of pain, malaise and fever. The child refuses to walk or to move an affected limb. There is invariably local tenderness over the inflammation; local redness, swelling and oedema are late signs, but joint movement is often painful. Blood cultures may be positive in about 60% of cases. A sequestrum and involucrum are features of chronic osteomyelitis; a sequestrum is a necrotic nidus of bone within a focus of chronic osteomyelitis while an involucrum is a cloak of new bone produced by the periosteum around the infection. If the child is refusing to move the limb/joint, then a joint fluid aspiration may be necessary to distinguish acute osteomyelitis from septic arthritis. 72) The following statements regarding sudden infant death syndrome (SIDS) are correct: A The incidence is equal for infants born permaturely and at term. B It occurs more commonly in lower socio-economic groups. C If parents smoke, there is a two-fold increased risk of a baby dying as a result of SIDS. D SIDS does not occur after 6 months of age. E Siblings of children who suffered SIDS have a five-fold increased chance of dying from SIDS. (False) (True) (True) (False) (True)

Comments: If babies are left to sleep in the prone position there is an increased risk of SIDS. There is an increased incidence of SIDS in premature infants, low-birth-weight infants and siblings of SIDS victims. It is more common in lower socio-economic groups. Parental smoking (maternal>paternal) is also a risk factor. Peak incidence is around 3-4 months and most cases occur before 6 months of age although it has been reported up to 1 year.

Pediatrics

27

73) Small-for-gestational-age infants are more likely than appropriate-for-gestationalage infants to have: A Hypoglycaemia. B Hypocalcaemia. C Respiratory distress syndrome. D Congenital malformations. E Persistent fetal circulation. (True) (True) (False) (True) (True)

Comments: Small for gestational age fetuses are more likely than normal newborn infants to encounter several problems in the neonatal period. These include: hypothermia, hypoglycaemia due to reduced glycogen stores, impaired gluconeogenesis and relative hyperinsulinism, hyperglycaemia, respiratory difficulties (meconium aspiration and chest infections), polycythaemia and persistent fetal circulation.

74) The following are features of premature thelarche: A Peak onset between 6 months and 2 years. B Association with Turner Syndrome C Asymmetrical breast enlargement. D Associated growth spurt. E Areola pigmentation. (True) (False) (True) (False) (False)

Comments: Premature thelarche is common, with a peak incidence between 6 months and 2 years. It may be asymmetrical, and is differentiated from true precocious puberty by the absence of pubic hair development and a growth spurt. It is usually self-limiting. 75) Left axis deviation is seen on the ECG in the following conditions: A Patent ductus arteriosus. B Ebstein's anomaly. C Atrioventricular canal defects. D Large ventricular septal defect. E Tetralogy of Fallot. (False) (False) (True) (False) (False)

Comments: Left axis deviation J is also seen in tricuspid atresia, Atrioventricular canal defects.

Pediatrics

28

76) Hypersecretion of gastric acid is associated with: A Duodenal ulcer. B Vagotomy. C Short bowel syndrome. D Zollinger-Ellison syndrome. E Ranitidine. (True) (False) (True) (True) (False)

Comments: Gastric acid secretion is facilitated by parasympathetic stimulation via the vagus nerve in addition to local hormonal mechanisms mediated by gastrin. Duodenal ulceration is usually associated with hypersecretion of gastric acid whereas gastric ulceration often occurs with subnormal levels. Ranitidine inhibits gastric acid secretion by H2 receptor antagonism. It is often useful in short bowel syndrome as hypergastrinaemia occurs in many cases leading to excessive gastric acid secretion. In Zollinger-Ellison syndrome there is hypersecretion of the gastrin-secreting cells of the antrum of the stomach. This leads to increased gastric acid secretion causing multiple peptic ulcers. 77) A six month old baby presents with a generalized rash, affecting her trunk and extremities virtually equally. The diagnosis includes: A Measles in mild form. B Meningococcal septicaemia. C Pseudomonas septicaemia. D Haemophilia. E Giant urticaria. (False) (True) (True) (False) (False)

Comments: The measles rash starts about 14 days after exposure and appears as a generalized macular eruption on face, neck and spreads over 3 days. Meningococcal septicaemia causes a generalized non-blanching purpuric eruption. Pseudomonas and other gram-negative septicaemia can cause a rash, eccymoses and petechiae. Haemophilia is not associated with generalized rash. Giant urticaria.is characterized by recurring attacks of transient edema suddenly appearing in areas of the skin or mucous membranes and occasionally of the viscera, often associated with dermatographism, erythema, and purpura.

Pediatrics

29

78) Stiff neck in a young child occurs in: A acute poliomyelitis B retropharyngeal abscess C rickets D Stills disease E Vitamin A deficiency (True) (True) (False) (True) (False)

Comments: 1.due to the meningitic component in acute poliomyelitis 2.a well recognized cause. The neck is hyperextended. 4.due to the involvement of the cervical spine in the arthritis of Stills disease

79) The following conditions are associated with bilateral pulmonary hypoplasia: A Preterm prelabour rupture of membranes. B Diaphragmatic hernia. C Maternal smoking. D Myotonic dystrophy. E Renal agenesis. (True) (True) (False) (True) (True)

Comments: The following are associated with pulmonary hypoplasia: Oligohydramnios - renal aplasia/severe dysplasia (Potter's syndrome), - premature rupture of membranes, - postamniocentesis. Compression of lung - congenital diaphragmatic hernia, - cystic adenomatoid malformation, - pleural effusion, - small chest syndromes. Neuromuscular disease - spinal muscular atrophy, - myotonic dystrophy. 80) Regarding osteosarcomas A Affects the epiphyses of long bones B Are most commonly seen around the knee and in the proximal humerus C Haematogenous spread can result in pulmonary metastases D Is exclusively a disease of adolescence and early adult life E X-ray shows a 'sunburst' appearance due to soft tissue involvement (False) (True) (True) (False) (True)

Comments: Osteosarcomas affect the metaphyses of long bones. They are most commonly seen around the knee and in the proximal humerus. They often occur in young adults but are also seen in the elderly in association with Paget's disease. They usually present as bone pain and a palpable lump. X-ray shows periosteal elevation (Codman's triangle) and a 'sunburst' appearance due to soft tissue involvement. Early haematogenous spread occurs and the 5year survival rate is approximately 50%.

Pediatrics

30

81) Koplik's spots appear: A two days before the rash of measles B opposite incisor teeth C only when fever is over 39C D as red papules on the dorsum of the hands E fluorescent under Wood's light (True) (False) (False) (False) (False)

Comments: Classically Koplik's spots are pathognomonic of measles being found opposite the premolars two days prior to the development of the rash. 82) Causes of chronic rhinitis include: A Tetanus B Dysmotile cilia syndrome C Foreign body D Henoch-SchX nlein purpura E Wegener granulomatosis (False) (True) (True) (False) (True)

Comments: Chronic nasal discharge with or without acute exacerbations may reflect an underlying disturbance such as:
Local: Nasal polyps, chronic sinusitis, chronic infected adenoids, deviated septum, foreign bodies, nasal diphtheria. Generalised: Cystic fibrosis, dismotile cilia syndrome, allergy, syphilis, Wegener granulomatosis, immune deficiency, hypothyroidism.

The commonest symptom is chronic nasal discharge with foul odour. A bloody discharge suggests a foreign body, nose picking, syphilis, or diphtheria.

83) Congenital hypothyroidism: A Has an incidence of some 1:1000 births B Is screened for at birth by testing Thyroxine levels in the blood C If untreated may be a cause of precocious puberty D May present as short stature E Is associated with prolonged jaundice in the neonatal period (False) (False) (True) (True) (True)

Pediatrics

31

Comments: Congenital hypothyroidism has an incidence of 1 in 4000 births. It is screened for by measuring TSH levels. If untreated precocious puberty may occur, however growth and skeletal maturation is impaired. It may present as growth failure and short stature. It is rarely associated with prolonged conjugated hyperbilirubinaemia in neonates.

84) Truncal (sensory) ataxia is recognized in: A Extrapyramidial diseases B Lesions of the cerebellar vermis C Benzodiazepine overdose D Middle cerebral artery occlusion E Phenytoin (False) (True) (True) (False) (True)

Comments: The smooth accurate performance of purposeful movements require intact sensory and motor functions as well as efficient control by higher centres. Any lesion causing weakness can be accompanied by clumsiness, but incoordination is particularly prominent in sensory and cerebellar ataxia. Sensory ataxia results from defective propreception, and can be mitigated by visual control. It is, therefore, exacerbated when the eyes are closed. Cerebellar ataxia - peripheral propreceptive information is fed to the posterior lobe of the cerebellum, which is also connected to the motor cortex. This incoordination is, therefore, not susceptible to visual compensation. Tests of coordination include:
1. 2. 3. 4. 5. Finger/nose test. Finger to finger test. Intention tremor. Heel to knee test. Dysdiadochokinesis.

Phenytoin gives rise to nystagmus as well as ataxia, and this is, therefore, central in origin. The effects of Diazepam are also centrally mediated with associated drowsiness.

85) Regarding osteosarcomas A Affects the epiphyses of long bones B Are most commonly seen around the knee and in the proximal humerus C Haematogenous spread can result in pulmonary metastases D Is exclusively a disease of adolescence and early adult life E X-ray shows a 'sunburst' appearance due to soft tissue involvement (False) (True) (True) (False) (True)

Pediatrics

32

Comments: Osteosarcomas affect the metaphyses of long bones. They are most commonly seen around the knee and in the proximal humerus. They often occur in young adults but are also seen in the elderly in association with Paget's disease. They usually present as bone pain and a palpable lump. X-ray shows periosteal elevation (Codman's triangle) and a 'sunburst' appearance due to soft tissue involvement. Early haematogenous spread occurs and the 5year survival rate is approximately 50%.

86) Hypertension in childhood typically complicates A Post-streptococcal glomerulonephritis B Coarctation of the aorta C Minimal change nephrotic syndrome D Appendicitis E Peptic ulceration (True) (True) (False) (False) (False)

Comments: Hypertension in childhood is commonly secondary due to renal disease (Glomerulonephritis), heart disease (coarctation) or endocrine disease (Phaeochromocytoma). Minimal change disease is not complicated by hypertension and typically resolves with no sequelae.

87) Regarding Salter-Harris classification for bone injuries in children: A This classification is for fractures through the metaphysis and the diaphysis B Growth arrest is common in Salter-Harris type I injury C Salter-Harris type IV is the commonest form of growth plate injury D In Salter-Harris type V, there is compression of the epiphysis leading to growth arrest E Accounts for <5% of all fractures in children

(False) (False) (False) (False) (True)

Comments:
This classification is for fractures through the growth plate or the epiphysis (in Salter-Harris types II & IV the metaphyseal fragment is also involved but the diaphysis is never affected). Salter-Harris types I & II does not involve the germinal layer and therefore growth disturbance is uncommon. Salter-Harris types II & I are the commonest forms of epiphyseal injuries. In Salter-Harris types III & IV, the germinal layer is breached and growth disturbance is likely, although its incidence could be minimised by precise reduction of the fracture. Although not originally described, Salter-Harris type V fracture is recognised as a crushing injury of the epiphysis following which growth arrest is common. This fracture is often diagnosed retrospectively, when disturbance of physeal growth is apparent as a limb deformity. Because of the weakness of the growth plate, these injuries are relatively common, accounting for approximately onethird of all fractures in children.

Pediatrics

33

88) The following are recognized complications in a post term baby: A Birth asphyxia. B Hypoglycaemia. C Hyaline membrane disease. D Pulmonary haemorrhage. E Prolonged jaundice. (True) (True) (False) (True) (False)

Comments: Birth asphyxia secondary to traumatic delivery and meconium aspiration occurs and hypoglycaemia may occur from depleted glycogen stores. Hyaline membrane disease and prolonged jaundice are seen in preterm infants.

89) The following statements are true: A Secretion of parathyroid hormone (PTH) is stimulated by increased ionized calcium levels in extracellular fluid. B Vitamin D increases calcium and phosphate mobilization from bone. C Vitamin D increases calcium and phosphate absorption from the intestine. D PTH increases reabsorption of phosphate from the proximal renal tubule. E Calcitonin is secreted by thyroid C-cells.

(False) (True) (True) (False) (True)

Comments: PTH is stimulated by low levels of ionized calcium in extracellular fluid. It leads to increased mobilization of calcium and phosphate from bone and increases calcium from the distal renal tubule. PTH also inhibits reabsorption of phosphate from the proximal renal tubule. The biologically active form of vitamin D (1, 25-hydroxycholecalciferol) increases mobilization of calcium and phosphate from bone and increases gut absorption of calcium and phosphate. Calcitonin is secreted by thyroid C-cells and inhibits bone resorption in response to increased calcium.

Pediatrics

34

90) Atopic eczema: A Usually starts within the first two weeks of life. B Will have cleared by puberty in at least 50% of children. C Is best managed with an egg and milk free diet. D Predisposes to cutaneous viral infections. E May be exacerbated by a viral infection. (False) (True) (False) (True) (True)

Comments: Cows milk intolerance is commonly associated and soy milk may be substituted. Atopic eczema commonly starts within the first year of life. It predisposes to eczema herpeticum, infection with herpes simplex virus. Infection and stress as well as environmental exposure to allergens such as dust mite will exacerbate symptoms.

91) Regarding dysmorphic syndromes: A A malformation is a primary structural defect occurring during the development of a tissue or organ. B A deformation implies an abnormal physical influence in the intrauterine environment. C An association refers to a group of malformations occurring together more often than expected by chance. D A sequence is pattern of multiple anomalies that occurs when a single primary defect in early morphogenesis E A disruption involves destruction of a fetal part which initially formed normally.

(True) (True) (True) (False) (True)

Comments: Although singly, most syndromes are rare, recognition of a dysmorphic syndrome may give information regarding of risk of recurrence, prognosis, complications, unnecessary investigations which may be avoided, and allow the parents to liaise with self-help groups. Dysmorphology is the study of malformations during embryogenesis. All the definitions are correct apart from that for a sequence, which refers to a series of events occurring after one initiating defect. An example is Potter's Syndrome in which all the abnormalities may be traced to the single malformation of renal agenesis.

Pediatrics

35

92) Unconjugated hyperbilirubinaemia is typical of: A Dubin-Johnson syndrome B hereditary spherocytosis C Rotor's syndrome D Gilbert's disease E biliary atresia (False) (True) (False) (True) (False)

Comments: Dubin Johnson syndrome is associated with a conjugated hyperbilirubinaemia as well as mental retardation. Rotor's syndrome is a rare autosomal recessive condition associated with the inability to excrete organic anions into bile and is associated with a conjugated hyperbilirubinaemia. Biliary atresia is also associated with a conjugated jaundice. On the other hand, haemolysis is associated with unconjugated hyperbilirubinaemia as is Gilbert's disease.

Rotor's syndrome Rotor's syndrome is an autosomal recessive condition characterized by a chronic, predominantly conjugated hyperbilirubinaemia. The mechanism is similar to that seen in the Dubin-Johnson syndrome, with defective excretion of organic anions into bile. The liver is a normal colour on biopsy, unlike Dubin-Johnson syndrome. The prognosis is excellent. 93) The following are features of acute intermittent porphyria: A Positive Hoesch test. B Photosensitivity. C Autosomal recessive inheritance. D Diagnosed by the presence of gamma aminolaevulinic acid and porphobilinogen in the urine. E A lack of symptoms until after puberty. (True) (False) (False) (True) (False)

Comments: The porphyrias are a group of illnesses caused by a deficiency of one of the seven enzymes involved in haem biosynthesis. Acute intermittent porphyria is caused by porphobilinogen deaminase deficiency. It is diagnosed by the presence of gamma aminolaevulinic acid and porphobilinogen in the urine. The Hoesch test detects porphobilinogen in the urine by turning a cherry-red colour. Symptoms of the illness may start prior to puberty.

Pediatrics

36

94) in human neurons: A Myelin sheaths extend across the nodes of Ranvier. B Unmyelinated fibres have faster conduction. C Sodium ion influx occurs during the action potential. D The action potential increases with increased stimulation. E Increased extracellular calcium leads to increased neuronal excitability. (False) (False) (True) (False) (False)

Comments: Myelin sheaths are interrupted by the nodes of Ranvier allowing depolarization to jump from one node to another and increase conduction velocity. This is called saltatory conduction and it allows a more than 50-fold increase in conduction speed in myelinated fibres. The action potential occurs as a result of sodium ion influx and potassium efflux from the neuron and is an 'all or none' phenomenon. Decreased extracellular calcium concentration leads to a lowering of the threshold for neuronal depolarization and thereby increases nerve cell excitability.

95) Deficiency of cell mediated immunity occurs in the following: A Agammaglobulinaemia. B Asthma. C Human immunodeficiency virus (HIV) infection. D Malnutrition. E Di George syndrome. (False) (False) (True) (True) (True)

Comments: Causes of deficient cell emdiated immunity include: severe combined immune deficiency (SCID) defects of neutrophil mobility/chemotaxis Di George sydnrome malnutrition postinfection (e.g. measles) cancer chemotherapy HIV infection. 96) In the newborn term infant: A The Apgar score at 5 minutes is more predictive of later neurodevelopment prognosis than Apgar score at 1 minute. B Anaemia may be caused by a cephalhaematoma. C Ballotable kidneys are always abnormal. D The findings of bilateral single palmar creases in an otherwise normalappearing baby demands a chromosome analysis. E Oedema of the feet and hands suggest Turners syndrome.

(True) (True) (False) (False) (True)

Comments: A low 1-minute Apgar score does not correlate with the infant's future outcome. The 5-minute Apgar score, and particularly the change in the score between 1 and 5 minutes, is a useful

Pediatrics

37

index of the effectiveness of resuscitation efforts. An apgar score of 0-3 at 5 minutes is associated with increased risk of cerebral palsy. Ballotable kidneys may indicate the presence of polycystic disease, but not in all cases. Single palmar creases appear normally in approximately 1 out of 30 people. Cephalhaematoma can cause jaundice and anemia. Oedema of the feet and hands is a very rare clinical finding and would prompt an investigation for Turners syndrome.

97) Obstructive sleep apnoea (OSA) in children A is more pronounced Z[\]^ in rapid eye movement (REM) sleep B results in reduced synthesis of neurotransmitters C causes bradycardia and other arrhythmias D does not lead to permanent changes of pulmonary vasculature E treatment of choice is adenoidectomy (True) (True) (True) (False) (False)

Comments: There are two factors that are important in the development of obstructive sleep apnoea (OSA): firstly, tonsil and adenoid hypertrophy; and secondly the loss of tone in the tongue and pharyngeal wall in sleep. The latter is more pronounced in REM sleep. OSA causes a reduction in neurotransmitter levels, which together with sleep fragmentation accounts for daytime somnolence. A range of vagally mediated arrhythmiias, most commonly bradycardia, result from hypoxia. Permanent changes in the pulmonary arterial vasculature and corpulmonale are well described. Treatment of choice is adeno-tonsillectomy because adenoidectomy alone does not consistently cure OSA.

98) Erbs palsy: A Is the commonest form of birth trauma. B May lead to wasting of the arm. C Affects the small muscles of the hand. D Is common after breech delivery. E May be due to injury of the upper brachial plexus. (False) (True) (False) (True) (True)

Comments: Minor soft tissue injuries to the head for example following scalp electrode placement or caput succedaneum occur more commonly. Erbs palsy involves injury to the upper brachial plexus (C5-6), leads to wasting of arm, but not the instrinsic muscles of the hand, which would be the result of lower brachial plexus injury. Erbs palsy is more common in large for dates infants and breech deliveries. Klumpke described the clinical picture of lower brachial plexus injury (C7T1).

Pediatrics

38

99) The following statements regarding messenger RNA (mRNA) are correct:

A mRNA never contains introns. B mRNA is translated into proteins in the nucleus. C mRNA contains the bases cytosine and thymine. D Reverse transcriptase uses mRNA as a template to produce complementary DNA. E mRNA is used in the Southern blotting technique.

(False) (False) (False) (True) (False)

Comments: The structure of mRNA is similar to DNA except that uracil replaces thymine as one of the bases. Both coding (exons) and non-coding regions of DNA are initially transcribed into mRNA. Splicing is required for mature mRNA to be produced only consisting of introns. Translation occurs in the cytoplasm. Southern blotting is a technique that uses denatured fragments of DNA in a gel to bind to DNA probes in order to detect the presence of particular genes or sequences of DNA. The enzyme reverse transcriptase can be used by viruses to insert viral mRNA into the host genome. 100) Which of the following are true of eponymous fractures? A Bennett's fractures involves the distal ulna B Colle's fracture involves the distal radius C Galeazzi' s fracture involves the radial shaft & dislocation of the proximal radioulnar joint D Monteggia' s fracture involves the proximal ulna & anterior dislocation of the head of the radius E Pott's fracture is a general term applied to fractures around the knee (False) (True) (False) (True) (False)

Comments: Bennett's fracture Tis an intra-articular fracture of the base of the first metacarpal. Galeazzi's fractureJ involves the radial shaft with dislocation of the distal radioulnar joint. Monteggia's fractureT is angulated fracture at the junction of the proximal and middle third of ulna accompained by ANTERIOR dislocation of the radial head. Pott's fractureJ is a general term applied to fractures around the ankle. Colle's fractureJ involves the distal radius.

Pediatrics

39

101) The following features suggest a diagnosis of toxoplasma as a cause of congenital infection: A Growth retardation B Petechiae C Focal chorioretinitis D Hydrocephalus E Intracranial calcification Comments: Please refer to the excellent table figure 7.11 on page 73 of Illustrated Paediatrics. Acute infection with toxoplasma results from eating raw or undercooked meat. The incidence is greatly increased in France, and has resulted in a massive education programme there. Cats can become infected and mothers may come into contact with cat faeces. During primary infection, about 40% of fetus' are infected. Most are asymptomatic. The 10% who are symptomatic may have hydrocephalus, cerebral calcification, retinopathy (focal) and long term neurological disability. The chorioretinitis may progress gradually through childhood and cause later problems. Serial IgG antibody tests are reasonably sensitive, but obviously take time to distinguish between passively acquired and genuine fetal infection. Unfortunately IgM, though produced specifically by the fetus, is very insensitive. Treatment is with Spiromycin antenatally or Pirimethamine and Sulpherdiazine with Folinic acid supplements alternating with Spiromycin for the first year. No screening programme exists in the UK, although they do screen in France. Most cases in the UK are imported from abroad. 102) Cogenital adrenal hyperplasia: A Has presenting features that result from excessive adrenocorticotrophic hormone (ACTH) secretion. B Has an incidence of about 1 in 10,000. C Is most commonly due to 17-hydroxylase (17-OH) deficiency. D May present with hyponatraemia, hypochloridaemia and hypokalaemia. E Can be diagnosed by increased plasma 17-OH progesterone levels. (True) (True) (True) (True) (True)

(True) (False) (False) (False) (True)

Comments: Cogenital adrenal hyperplasia (CAH) is a variable clinical syndrome (Autosomal recessive) caused by an ACTH-driven adrenocortical hypersecretion secondary to an enzyme deficiency. Decreased cortisol production results in a loss of negative feedback to the hypothalamicpituitary axis and thus increased ACTH secretion. Incidence of the condition is about 1 in 5000 births. 21-hydroxylase deficiency is the common variant of CAH accounting for up to 95% of cases. A variable proportion (approximately 50%) of the 21-hydroxylase deficient type have salt-losing crises. The biochemical features of a salt-losing crisis are: metabolic acidosis, hyponatraemia, hypochloridaemia, hyperkalaemia, hypoglycaemia and hyperuraemia. Treatment includes glucocorticoid and mineralocorticoid replacement. Increased plasma 17OH progesterone is a feature of 21-hydroxylase deficiency along with increased urine free cortisol, increased urinary 17-oxosteroids and urinary pregnantriol.

Pediatrics

40

103) The following statements regarding coarctation of the aorta are correct: A Coarctation of the aorta is associated with a bicuspid aortic valve in more than 50% of cases. B When preductal, coarctation of the aorta is frequently associated with other cardiac defects. C Coarctation of the aorta is suggested by a pressure difference of 1015 mmHg between the upper and lower limbs. D Coarctation of the aorta may present in the neonatal period with acute collapse, acidosis and renal failure. E Hypertension may occur following surgical repair.

(True) (True) (False) (True) (True)

Comments: Coarctation of the aorta (CoA) has a slight male preponderance and accounts for about 8% of all congenital heart disease. The most commonly affected part of the aorta is in the upper thoracic section. Preductal CoA is associated with other cardiac defects in about 40% of cases (ventricular septal defect, patent ductus arteriosus, transposition of the great arteries). These children become symptomatic in very early life. Postductal CoA is usually an isolated defect and usually does not produce symptoms in infancy. A blood pressure gradient of >20 mmHg suggests CoA. Severe cases classically present at around 10 days of age with acute collapse, acidosis and renal failure. Prompt diagnosis and aggressive treatment is necessary. Hypertension may persist long term, even after successful repair of the CoA.

104) The following drugs are paired with recognized side-effects: A Salbutamol - hyperkalaemia. B Indomethacin - decreased urine output C Aminophylline - increased urine output. D Frusemide - decreased urinary calcium excretion. E Hydrochlorothiazide - hypoglycaemia. (False) (True) (True) (False) (False)

Comments: Salbutamol lowers serum potassium and may lead to hypokalaemia. Indomethacin is a powerful renal vasoconstrictive agent and therefore decreases urine output by decreasing glomerular filtration rate. Theophyllines have weak diuretic properties. Frusemide increases urinary calcium excretion and may be useful in severe hypercalcaemia. Thiazide diurectics inhibit pancreatic insulin release and may therefore cause hyperglycaemia.

Pediatrics

41

105) The following are features of Keloids: A Found only in humans B They do not extend beyond the margins of the original scars C More common in genitalia, eyelids and palms D Usually develops within weeks of initial injury E Surgical excision and primary closure is usually curative (True) (False) (False) (False) (False)

Comments: Keloids are dermo-proliferative disorders unique to humans of unknown aetiology. It is thought to have a familial tendency and is seen 5-15 times more frequently in nonwhites. Hypertrophic scars are confined to the borders of the original wound, but keloids extend beyond the original scar margins. Whilst anatomical locations such as genitalia, eyelids, palms and soles are usually spared, keloids are more common in wounds that cross tension lines and in areas such as the earlobe, presternal and deltoid regions. Hypertrophic scars generally begin to develop in the weeks after injury, whereas keloids can develop up to one year later. Various treatment modalities including surgical excision have been tried, but no treatment, at present, seems to be curative for keloids. Intralesional steroid therapy and pressure therapy are useful to keep the lesion under control.

106) Which of the following is/are true concerning atopic eczema? A Usually starts in the first year of life. B Never benefits from dietary measures. C Does not have a genetic basis. D Is a generalized rash over the whole body. E Should be treated in its early stages with topical corticosteroids. (True) (False) (False) (False) (False)

Comments: Atopic eczema often presents in childhood, affecting 15-20% of children, and there is a strong genetic basis. Atopy is the term that describes a family predisposition to a variety of allergic conditions, including asthma, hay fever and eczema. Dietary measures, eg exclusion of cows milk from the diet may have a major effect on the disease course, especially in infants. Atopic eczema usually affects the face, trunk, the backs of the knees and the front of the elbows. The main treatments are aimed at controlling and preventing inflammation and itching and include avoiding triggers which can be environmental, such as house dust mites or common detergents, food based, such as an allergy to dairy product, or genetic. Frequent bathing and hydrating of the skin avoiding strong soaps, and the liberal use of moisturizers and lubricants are first line treatments. Sterroids should be used in short bursts, for acute flare-ups, and potent topical steroids are not generally recommended for long-term use, especially in children and through puberty.

Pediatrics

42

107) Which of the following statements is/are true of pertussis (whooping cough)? A immunization is effective in preventing the disease in over 95% of immunized people B for well premature infants the immunization should be carried out 2 months after birth C erythromycin has been shown to inhibit the growth of the aetiological agent in vitro D children under the age of 3 months are not at risk from the disease E the incidence of permanent neurological complications from immunization is less than 1:150,000

(False) (True) (True) (False) (True)

Comments: Pertussis is caused by Bordettela pertussis and young children/infants are particularly susceptible. The vaccination is a suspension of killed organisms and confers immunity in less than 90%. Recommendation for the triple DTP vaccination are for a series of 5 vaccinations beginning at two months of age. Effective treatment includes macrolides. Neurological complications associated with the vaccine are rare.

108) Which of the following is true regarding rheumatoid arthritis A Primarily affects the articular cartilage B Is associated with the HLA antigens DR4 and DW4 C Occurs more often in women D The cervical spine is commonly involved E Extra-articular manifestations occur in 20% of patients (True) (False) (True) (False) (True)

109) The following are recognized risk factors for neonatal group B streptococcus (GBS) infection: A Preterm labour. B Prolonged rupture of membranes greater than 24 hours. C Chorioamnionitis. D Sibling with previous neonatal GBS infection. E Maternal GBS urinary tract infection. (True) (True) (True) (True) (True)

Comments: Risk factors for neonatal GBS sepsis are: preterm labour rupture of membranes>18 hours prior to birth maternal chorioamnionitis maternal colonization with GBS previous baby with GBS sepsis maternal GBS urinary tract infection.

Pediatrics

43

110) A boy of 2 years has congenital adrenal hyperplasia (21 hydroxylase deficiency) and is being treated with Cortisone Acetate and Fludrocortisone: A A greater than expected growth rate points to inadequate treatment with cortisone. B After 3 years of age there will be no need to continue Fludrocortisone C There is no risk to any later born female children D The ideal method of control is to regularly measure plasma 17 hydroxyprogesterone E Even though control is satisfactory throughout childhood he is still likely to end up shorter than expected Comments: In poorly controlled or untreated patients, increased androgen production leads to the early appearance of pubic hair, phallic enlargement, increased muscular development, rapid growth and skeletal maturation. Classical CAH causes impaired cortisol and aldosterone synthesis. Fludrocortisone is required to prevent salt losing crisis. The disease is inherited in an autosomal recessive fashion. Impaired cortisol synthesis leads to a feedback increase in ACTH, which increases the production of cortisol precursors and androgen precursors proximal to the block including serum 17 hydroxyprogesterone , androstenedione, and testosterone. The striking elevation of 17 hydroxy progesterone is a distinguishing feature of the condition. Long term follow up studies on the effects of glucocorticoid and mineralocorticoid replacement indicate that the mean adult height of males and females is less than that of unaffected siblings and less than normal mean adult height. 111) A ventricular septal defect: A Is likely to cause heart failure in the first week of life. B Is associated with plethoric lung fields on chest x-ray in a 10 week old infant. C Requires surgical correction in all but a small minority of cases. D Requires surgical correction if central cyanosis occurs. E Causes fixed splitting of the second heart sound. (False) (True) (False) (False) (False)

(True) (False) (False) (True) (True)

Comments: VSD is unlikely to present till after the first month of life and associated with pulmonary plethora. The majority of cases will resolve spontaneously. Central cyanosis indicates shunt reversal and pulmonary hypertension, which implies poor prognosis and response to operative repair of the VSD. The second heart sound is normally spilt.

Pediatrics

44

112) With regard to congenital heart disease A Occurs in 3% of children under the age of 5 years B Only 50% of cases present in the neonatal period C Most babies with cyanotic congenital heart disease are small for gestational age D Detection of pulmonary stenosis in a female child should be followed by karyotyping E Heart failure is a common presentation in Fallots tetralogy (False) (True) (False) (False) (False)

Comments: Congenital heart disease occurs in 3-12 per thousand live births and many conditions such as ventricular septal defect and tetralogy of Fallot may present in infancy, with 50% presenting in infancy and the rest presenting later in life (sometimes in adulthood eg ASD). Growth failure is masked by fluid retention and reduced urine volume which leads to inappropriate weight gain. Pulmonary stenosis T is associated with Noonans syndrome, congenital Rubella, connective tissue disorders and Williamss syndrome-not chromosomal abnormalities. In females of XO karyotype coarctation and bicuspid aortic valve may occur. Fallots tetralogy typically presents with hypercyanotic spells characterised by anxiety, air hunger and respiratory distress

113) Which of the following are true concerning paediatric abdominal trauma? A diagnostic peritoneal lavage (DPL) is a commonly performed investigation B a rectal examination (PR) is contraindicated C in the presence of hypovolaemia, 4.5% Albumin is preferable to crystalloid D a plain abdominal CT scan is indicated in the absence of cardiovascular instability E a double contrast CT scan is of limited value Comments: A diagnostic peritoneal lavage (DPL) should rarely be used in children as the presence of intraperitoneal blood per se is not necessarily an indication for laparotomy. A DPL is 3 considered positive if the red cell count is over 100 000 /mm , the white cell count over 500 3 /mm or if enteric contents or bacteria are seen. Rectal examinations should only be performed on children by the operating surgeon and even then it should only be done if the results of the examination will alter the management. Fluid resuscitation is based on boluses of 20 ml/kg of crystalloid not albumin. A double contrast CT scan of the abdomen (with intravenous and intragastric contrast) is the radiological investigation of choice in children but should only be performed in cardiovascularly stable patients.

(False) (False) (False) (False) (False)

Pediatrics

45

114) The following drugs are appropriately linked to substances that reverse their action: A Morphine - naloxone. B Warfarin - protamine. C Pancuronium - neostigmine. D Propanolol - isoprenaline. E Ipratropium bromide - atropine. (True) (False) (True) (True) (False)

Comments: Naloxone is a narcotic antagonist that competes for central nervous system narcotic receptor sites. Protamine combines with heparin to form a complex without anticoagulant activity. Neostigmine inhibits acetylcholinesterase at the neuromuscular junction, thereby reversing the action of non-depolarizing muscle relaxants such as pancuronium, which competitively block cholinergic receptors. Propanalol is a non-selective beta-blocker and isoprenaline is a beta agonist. Ipratropium bromide (Atrovent) is an atropine derivative.

115) Ulcerative colitis (UC) is associated with: A Ankylosing spondylitis. B Renal tubular acidosis. C Episcleritis. D Sub-total villous atrophy. E Psoriasis. (True) (False) (True) (False) (False)

Comments: Ulcerative colitis only affects the colon, except for the rare backwash ileitis. Thrombocytosis may be a non-specific sign of inflammation. Extra-intestinal manifestations occur more commonly with ulcerative colitis than Crohn's disease. Growth retardation is present in 25% at diagnosis. Of the extra-intestinal manifestations that occur with IBD, joint, skin, eye, mouth, and hepato-biliary involvement tend to be associated with colitis whether ulcerative or Crohn's colitis. Correlations with bowel disease: erythema nodosum,anaemia, pyoderma gangrenosum, sclerosing cholangitis, peripheral arthritis, ankylosing spondylitis, and sacroiliitis, uveitis, erythema nodosum.

Pediatrics

46

116) Regarding developmental (congenital) dislocation of the hip: A Instability of the hip occurs in less than 1 in 20,000 live births B It is a recognized complication of breech presentation C One of the possible causes of this condition is subclinical neuromuscular abnormality D It is characterised by an anterior lump sign E If identified at birth, conservative treatment is usually effective Comments: Congenital dislocation of the hip (CDH), now termed developmental dysplasia, is of uncertain aetiology, and occurs in some 5 cases per 1000 live births. In many instances, there is a strong family history and predominance in females, although this is less marked in late presentations. Breech presentation, caesarean section and being the first-born child are other recognized factors. The causes of hip instability are thought to be capsular laxity, acetabular anteversion and possibly subclinical neuromuscular abnormality. The diagnostic sign is elicited at birth by separating the infants flexed thighs and noting the jump or thud as the femoral head slips back into the socket in the mid range of abduction. Once the child starts to walk, the restricted hip movement causes gait abnormalities, the limb appears shortened with pelvic asymmetry and there is an anterior groin lump sign. The limb may be extremely rotated and abduction is limited. Splintage with the hips 90 degrees abducted and 90 degrees flexed produces a successful outcome in about 98% of the cases if the instability is identified at birth. 117) In children with a ventricular septal defect: A it is the commonest congenital heart lesion B closes spontaneously in up to 50% of cases C of Maladie de Roger type is the most severe form D is associated with pulmonary oligaemia E with a large lesion is associated with biventricular hypertrophy (True) (True) (False) (False) (True) (False) (True) (True) (True) (True)

Comments: VSDs are the commonest congenital heart defect with the vast majority being small often spontaneously closing and of no haemodynamic significance. Small residual VSDs may be picked up due to the loud murmur (Maladie de Roger) and VSDs are associated with increased pulmonary blood flow. Larger VSDs would be associated with biventricular hypertrophy associated with volume overload.

Pediatrics

47

118) The causes of hepatosplenomegaly in a 10-year old child include A Gaucher's disease B metastatic spread from a medulloblastoma C leukaemia D primary biliary cirrhosis E congenital rubella syndrome (True) (False) (True) (False) (True)

Comments: Causes of HS-megaly include leukaemia, Gaucher's (glycogen storage disease), and infections. PBC would not even be expected in a 10 year old and Medulloblastoma is a cerebral tumour where no HS-megaly would be expected. Rubella may itself cause HS-megaly as may congenital rubella in the newborn. The latter is associated with a poor prognosis. If the child were to survive until 10 years of age the inital HSM may be expected to have subsided. 119) Complications of measles include: A common occurrence of severe corneal ulceration B cancrum oris C higher incidence of acute appendicitis D post-measles encephalopathy one month later E subacute sclerosing panencephalitis occurs years after an attack of measles (False) (True) (True) (False) (True)

Comments: Corneal ulceration is rarely associated with measles but cancrum oris is a feature. The associated lymphadenopathy may cause a presntation with acute appendicitis. Measles encephalopathy occurs 1-14 days after the onset of infection but SSPE, in which measles is implicated, is a particularly rare disease and may occur many years after the inital attack of measles. 120) A lesion of the prefrontal cortex can present with: A Disinhibition B Loss of appreciation of smell C Spatial orientation disruption D Speech problems E Abnormalities of micturition Comments: Lesions of the cerebral cortex result in dementia, dysphagia, apraxia, astereognosis and other forms of agnosia. Because of the diffuse radiation of nerves beyond the internal capsule, (True) (True) (False) (False) (False)

Pediatrics

48

upper motor neurone signs tend to be more localized. A paresis confined to one limb or to one side of the face is likely than a hemiplegia. Specific visual field defects may assist in localizing the lesion within the cerebral hemisphere. 121) A newborn infant with a fracture of the femur may have: A Been born by Caesarean section. B An absent Moro reflex in the affected limb. C Evidence of disease elsewhere. D An associated sciatic nerve palsy. E Need of treatment by bone plating. (False) (True) (True) (False) (False)

Comments: Twin pregnancies, breech presentation and prematurity are associated with fractures. Skeletal deformities such as osteogenesis imperfecta may be the cause. Splinting or casting is the treatment. 122) Target cells appear in the peripheral blood film in the following conditions: A Chronic liver disease. B Beta-thalassaemia. C Vitamin B12 deficiency. D Haemoglobin C disease. E Sickle cell disease. (True) (True) (False) (True) (True)

Comments: Target-shaped red blood cells with normal haemoglobin content are present in homozygous haemoglobin C disease. This is a relatively benign condition that results in splenomegaly and red blood cells with increased osmotic fragility. The most common cause of target cells is liver disease. Hypochromic target cells appear in the thalassaemias and in sickle cell disease. 123) The following are features of neonatal heroin withdrawal A Sneezing B Wakefulness C Hiccoughs D Temperature instability E High-pitched cry (True) (True) (True) (True) (True)

Pediatrics

49

Comments: Physiologic addiction to narcotics occur in most infants born to actively addicted mothers, since opiates cross the placenta. Withdrawal may be manifest even before birth by increased activity of the foetus when the mother feels the need for the drug or develops withdrawal symptoms. Heroin and methadone are the drugs most frequently associated with withdrawal syndromes, but these syndromes may also occur with alcohol, phenobarbital, pentazocine, diazepam, etc. Pregnancy in women who abuse illegal drugs/alcohol is high risk. Prenatal care is usually inadequate, and there is a higher incidence of sexually transmitted disease, premature rupture of the membranes, breech presentations and prolapsed cords. Heroin addiction results in a 50% incidence of low birth weight infants, half of whom are small for gestational age. Infections, maternal under-nutrition, and a direct foetal growth inhibiting effect are associated abnormalities. The rate of stillbirths is increased, but not the incidence of congenital anomalies. Clinical manifestations of withdrawal occur in 50-75% of infants, usually within the first 48 hours, depend on the daily maternal dose (<6 mg/24 hr is associated with no/mild symptoms); duration of addiction (>1 year has a greater than 70% incidence of withdrawal); time of last maternal dose (there is a higher incidence if the last dose was taken within 24 hr of birth). Symptoms rarely appear as late as 4-6 weeks of age. Tremors and hyperirritability are the most prominent symptoms. The tremors may be fine or jittery and indistinguishable from those of hypoglycaemia but are more often coarse, "flapping," and bilateral; the limbs are often rigid, and hyper-reflexic. Hyperactivity are generally marked and may lead to skin abrasions. Other signs include tachypnoea, diarrhoea, vomiting, highpitched cry, fist sucking, poor feeding, and fever. Sneezing, yawning, convulsions, abnormal sleep cycles, nasal stuffiness, apnoea, flushing alternating rapidly with pallor, and lacrimation are less common. The diagnosis is established by the history and clinical presentation. Examining the urine for opiates may reveal only low levels during withdrawal, but quinine, which is often mixed with heroin, may be present in higher concentrations. Hypoglycaemia and hypocalcaemia should be excluded.

124) The following conditions show a predisposition to developing hepatocellular carcinoma: A Tyrosinaemia. B Galactosaemia. C alpha-1 antitrypsin deficiency D hepatitis A. E Beta-thalaessaemia. (True) (True) (True) (False) (True)

Comments: Heptocellular carcinoma is found frequently in conditions where liver cirrhosis is an important feature. Serum alpha-fetoprotein is raised in up to 80% of cases. Prognosis for children with this condition is poor.

Pediatrics

50

125) A 15 year old girl suffers from weight loss and amenorrhoea. The following findings suggest anorexia nervosa: A high plasma LH concentrations B buccal pigmentation C hypokalaemia D bouts of overeating E Increased exercise (False) (False) (True) (True) (False)

Comments: Typically anorexia nervosa is associated with hypogonadotrophic hypogonadism and elevated LH and FSH concentrations would suggest primary ovarian failure. Similarly, increased exercise is itself a cause for amenorrhoea and weight loss and would therefore not suggest a diagnosis of anorexia nervosa per se. Most female athletes for instance are amenorrhoeic but certainly don't have AN. Buccal pigmentation suggests Addison's disease, but a hypokalaemic could imply vomiting or use of laxatives. The DSM criteria reveal that the associated features of anorexia nervosa include "Some people with this disorder cannot exert continuous control over their intended voluntary restriction of food intake and have bulimic episodes (eating binges), often followed by vomiting.

126) Children with eczema: A Are more often found in social class V families. B Have involvement of the face in most cases. C Should not use bubble baths. D Often have a family history of psoriasis. E May develop adrenal suppression from topical steroid application. (False) (True) (True) (False) (True)

Comments: Eczema is related to the presence of a family history of atopic condtions rather than social class or psoriasis occurring in family members. The common sites of occurrence are in the face, ears,elbows and knees. Irritant chemicals may exacebate the rash. Topical steroids should be used only if symptoms do not resolve as they may lead to adrenal suppression.

Pediatrics

51

127) The following is correct concerning a patent ductus arteriosus: A Physiological closure in the normal infant born at term occurs within the first 24 hours. B Interruption of the aortic arch and pulmonary atresia are ductdependent malformations. C In the prematurely born baby it is possible to keep the ductus open with indomethacin. D The incidence of patent ductus arteriosus is higher in small-for-dates babies compared with those normally grown. E The incidence of patent ductus arteriosus is lower in children in highaltitude communities.

(True) (True) (False) (False) (False)

Comments: The physiological function of the ductus arteriosus in the fetus is to divert blood away from lungs to the placenta, constriction occurs in the first 24 hours of life in response to increased environmental oxygen. In order to maintain life some cardiac malformations such as pulmonary atresia ,hypoplastic left ventricle and interrupted aortic arch require the patency of a wide PDA after birth. Indomethacin causes duct closure through an anti- prostaglandin effect. PDA occurs in premature, rather than small for dates infants. Birth at a high altitude location increases the risk of PDA. 128) Erythema nodosum can be found in association with: A Miliary tuberculosis. B Leprosy. C Streptococcal infection. D Pneumococcal infection. E Aspirin therapy. (False) (True) (True) (False) (False)

Comments: Other causes of Erythema nodosum include yersiniosis, ulcerative colitis and sarcoidosis. Drugs causing the condition include the oral contraceptive pill and sulphur containing antibiotics. Non steroidal anti-inflammatory drugs are used in treatment. 129) In suspected acute rheumatic fever the following indicate carditis: A An ESR of 120mm in one hour. B Short apical soft systolic bruit. C Strong cardiac impulse at the apex, which is displaced laterally. D Sinus arrhythmia. E Erythema nodosum. (False) (False) (True) (False) (False)

Pediatrics

52

Comments: The confirmation of rheumatic carditis solely depends on the findings of mitral and or aortic incompetence. 130) Antibodies cross the placenta causing disease in the fetus/neonate in the following conditions: A Dystrophica myotonica. B Diabetes insipidus. C Hyperthyroidism. D Myasthenia gravis. E Idiopathic thrombocytopenic purpura. (False) (False) (True) (True) (True)

Comments: Infants born to mothers with dystrophica myotonica are likely to be affected with the condition but not as a result of antibody transfer. Mothers who have been thyrotoxic will have circulating IgG antibodies that may affect the neonate. About 10% of infants born to mothers with myasthenia gravis are affected by an IgG antibody. Autoimmune thrombocytopenia occurs in neonates of mothers who suffer from idiopathic thrombocytopenic purpura. Mothers who are Zwb antigen negative can produce an IgG antibody to Zwa antigen (present in 98% population). This can cause an isoimmune neonatal thrombocytopenia. 131) Hypoglycaemia is a feature of the following disorders: A Nesidioblastosis B Congenital adrenal hyperplasia C Reyes syndrome D Maple syrup urine disease E Phenylketonuria (True) (True) (True) (True) (False)

Comments: Nesidioblastosis is histological pattern of hyperplasia of pancreatic beta cells, which results in endogenous hyperinsulinaemia. Congenital adrenal hyperplasia causes hypoglycaemia as a consequence of failure to synthesise cortisol. Reyes syndrome results in hepatic failure and defective glycogen synthesis. Maple syrup urine disease results in the accumulation of branched chain amino acids and causes defective gluconeogenesis and hence hypoglycaemia. Phenylketonuria is not associated with hypoglycaemia, as gluconeogenesis is not compromised.

Pediatrics

53

132) The following are characteristic features of infantile autism: A Failure to develop social relationships. B Presence of echolaliaT abcdeZ fghi^ jkg lmckgZ [^ncogZ C Normal intelligence. D Ritualistic or compulsive phenomena. E More common in girls. (True) (True) (False) (True) (False)

Comments: Autism is characterised by failure of communication, interaction, repetitive or ritualistic behaviour and developmental delays and learning disabilities. The ratio of males: females affected is 3-4:1. 133) Idiopathic growth hormone deficiency: A If diagnosed and treated with human growth hormone before the age of 9 years will result in the attainment of normal stature. B Shows a male preponderance of 2:1 C Results in retarded bone maturation D Is associated with tendency to central obesity E Is a cause of delayed puberty

(False) (True) (True) (True) (True)

Comments: Most treated children have an adult height significantly better than expected before therapy and reach the third centile on the growth curve, however most do not reach their predicted genetic potential. Some forms of familial growth hormone deficiency are x linked and therefore present more often in males. Skeletal maturity is retarded. Central obesity, abnormal facial bone growth, small penis, delayed puberty and poor muscle bulk are clinical features. 134) The following would lead you to consider a murmur to be innocent in a four and a half year old child: A Pansystolic timing. B Association with a thrill. C Marked variation of loudness with change of posture. D Radiation towards the left axilla. E Association with fixed splitting of the second heart sound. (False) (False) (True) (False) (False)

Comments: Innocent murmurs are of short duration and vary with posture. A thrill implies significant turbulence and hence valvular disease. Radiation to the axilla implies mitral incompetence and fixed splitting implies an ASD.

Pediatrics

54

135) Absent ankle reflexes and extensor plantar reflexes are seen in the following conditions: A Abeta-lipoproteinaemia. B Cord compression at L3/L4. C Metachromatic leukodystrophy. D Friedreichs ataxia. E Vitamin B12 deficiency. (True) (False) (True) (True) (True)

Comments: The clinical presentation of absent ankle reflexes and extensor plantar reflexes suggests a combination of upper and lower motor neuron signs. In abetalipoproteinaemia, metachromatic leukodystrophy, Friedreichs ataxia and vitamin B12 deficiency there are combinations of corticospinal tract dysfunction and peripheral neuropathy which result in the mixed upper and lower motor neuron signs. In spinal cord compression at L3/L4 there will be a loss of the knee jerk reflex. The ankle jerk reflex will be intact. 136) Patent ductus arteriosus: A May be treated with prostaglandin infusion. B Is commonly encountered in very low birthweight infants. C Causes a wide pulse pressure. D May present with cyanosis. E Should be suspected when femoral pulses are weak or impalpable. (False) (True) (True) (False) (False)

Comments: Prostaglandins will maintain ductal patency. PDA is frequent in very low birthweight and premature babies and causes a wide pulse pressure. Cyanosis would occur if the shunt reverses. Weak femoral pulses suggest Aortic coarctation. 137) A parietal lobe lesion is associated with: A Asterognosis B Finger agnosia C Sensory inattention D Dressing apraxia E Superior quadrant homonymous anopsia (True) (True) (True) (True) (False)

Comments: Astereognosis is the inability to discriminate shapes, and may indicate a problem of peripheral sensation or cortical/ memory interpretation. Agnosia is the loss of the ability to recognise sensory inputs, and can be auditory,

Pediatrics

55

visual, olfactory, gustatory or tactile. Apraxia is the inability to carry out familiar purposive movements (especially using an object), in the absence of motor or sensory impairment. Anopsia is the suppression of vision in one eye, as in heterotropia. Heterotropia is the failure of the visual axes to remain parallel when fusion is a possibility. Sensory inattention is the lack of conscious awareness of a sensory input. A lesion in the non-dominant parietal lobe may give rise to ipsilateral dyspraxia. It is often associated with other signs of cortical disturbance, such as agnosia or dysphasia.
138) Gynaecomastia can be found in the following conditions: A Two-thirds of pubescent boys. B Hypothyroidism. C Klinefelter syndrome. D Liver cirrhosis. E Ketoconazole therapy. (True) (True) (True) (True) (True)

Comments: Gynaecomastia can be a transient phenomenon in up to two-thirds of male adolescents. This is associated with a decreased testosterone to oestriol levels. Gynaecomastia in liver cirrhosis occurs as a result of decreased metabolism of oestrogens. Ketoconazole therapy directly inhibits testosterone synthesis resulting in gynaecomastia. Gynaecomastia occurs in hypoand hyperthyroidism. The mechanism behind this is not known. 139) Which of the following developmental attainments is/are appropriate for a child of 10 months? A Has good finger-thumb apposition with the left hand but uses a mild palmar grasp (False) on the right B Crawls symmetrically by dragging his extended legs behind, using his forearms C Has a symmetrical forward parachute reaction D responds to noise but cannot localize the source E is mobile by shuffling along on his bottom in a sitting position (False) (True) (False) True

Pediatrics

56

140) Benign Rolandic epilepsy: A Occurs mainly in pre-school children. B Is commoner than petit mal epilepsy. C Mainly occurs during sleep. D Is commonly resistant to anticonvulsant treatment. E Shows centrotemporal spikes on the electroencephalogram. Comments: Mainly school age. Petit Mal accounts for 1-2% of childhood epilepsy, while Rolandic is the commonest benign epilepsy in childhood. Seizures usually stop by mid-teens and may not need treatment. Fits are often nocturnal generalized tonic-clonic, and start with unilateral face or arm distortion. Centrotemporal spikes are seen. Carbamazepine or valproate may be indicated. 141) A study of the intellectually handicapped was performed. The 112 subjects, put through program A, showed an increase in their mean IQ score of 6 points. The 115 subjects, put through program B, showed an increase in their mean IQ score of 4. The p value was >0.05 . The following are true: A the numbers are too large for a Student t-test B the study demonstrates the usefulness of program A C the distribution of individual values is not important D even though the difference between the means is not significant it would be appropriate to calculate confidence intervals E the above results would be found by chance in less than 1:20 (False) (False) (False) (True) (False) (False) (True) (True) (False) (True)

Comments: a-The t-test is designed for small samples but as n increases it approximates the normal distribution. However the t-test should be used in cases where the population standard deviation is not known. (This question might be remembered incorrectly because an IQ test is designed in such a way that the population standard deviation is known! So maybe a z test would be best - using probabilities from the normal distribution). b-A is no more usefull than B or even simply repeating an IQ test? c+d This gives us an idea of the distribution of the data. e-more
142) in patent ductus arteriosus in children: A A continuous murmur must be present to make the diagnosis B Antibiotic prophylaxis is needed for dental extraction. C The femoral pulses are difficult to feel. D Prostaglandin infusions are used for pharmacological closure in the newborn. E Surgical management requires cardio-pulmonary bypass. (False) (True) (False) (False) (False)

Pediatrics

57

Comments: The continuous murmur is present in the absence of severe pulmonary hypertension, but the child may present following shunt reversal. Antibiotic prophylaxis is mandatory. Coarctation of the aorta is associated with reduced femoral pulses. Prostaglandins maintain ductal patency, indomethacin can be used. Surgical correction can be performed by trans-catheter placement of a mental clip.

141) A boy has recently had a sore throat and now complains of pain in the left knee and the right wrist. On examination the knee is swollen, warm and tender: the wrist appears normal. His temperature is 39aC, your suspicion that this might be rheumatic fever would be strengthened if you found: A Erythema marginatum B PR interval on the ECG of 0.25 seconds C Eosinophil count 400/mm3 D Low ASO titre E Pericardial friction rub (True) (True) (False) (False) (True)

Comments: Rheumatic fever is diagnosed on the Duckett Jones criteria with minor and major criteria. Major criteria include the characteristic skin rash, erythema marginatum, a prolonged PR interval, cardiac involvement and Sydenhams chorea. An elevated ASO titre is expected as the disorder is a consequence of an immune reaction to Lancefield group A beta haemolytic Streptococcus. Minor criteria include a raised ESR The eosinophil count is not included in this criteria.

142) Which of the following congenital heart lesions have characteristic ECG patterns: A Ventricular septal defect (VSD) B Patent ductus arteriosus (PDA) C Endocardial cushion defect D Tricuspid atresia E Mild pulmonary stenosis (False) (False) (True) (True) (False)

Comments: VSD and PDA have similar ECG patterns which may be normal or show Left ventricular hypertrophy , biventricular hypertrophy, or right ventricular hypertrophy if the shunt reverses. Tricuspid atresia is associated with small right ventricular complexes. Endocardial cushion defect involves fusion of the tricuspid and mitral valve with an ASD. The ECG will show right atrial enlargement, rightward axis, right ventricular hypertrophy and rSR1- v1. In tricuspid atresia may show left ventricular hypertrophy. In mild pulmonary stenosis the ECG will usually be normal.

Pediatrics

58

143) The following are recognized features of Rubella: A Koplik's spots B conjunctivitis C suboccipital lymphnode enlargement D thrombocytopenia E rashes on the 5th day of illness (False) (True) (True) (True) (True)

Comments: Koplik's spots seen in measles on the buccal mucosa Forchheimer spots (petechiae on the soft palate) may be seen in Rubella but are nonspecific. B-characteristic. C-characteristically suboccipital, postauricular and posterior cervical and may persist long after the rash has faded. D-very rare. Commoner complication is arthritis resembling rheumatoid. Encephalitis also rare. E-The rash has a similar distribution (face and behind ears spreading to trunk) as measles but is paler. RASH + INFECTION (MNEMONIC)

Very Sick People Must Take Early Retirement


Varicella day 1 Scarlet fever 2 Pox (small) 3 Measles 4 Typhus 5 Enteric fever (typhoid) 6 Rubella 1-7

(Thanks to Dr PM Dubey - http://www.geocites.com/Athens/Oracle/1632/medical/useful.htm)

144) Regarding chest deformities: A Sprengel's deformity is found in Klippel-Feil Syndrome. B Pectus excavatum is often associated with a displaced apex beat. C Severe kyphoscoliosis may be complicated by cor pulmonale. D Pectus carinatum is associated with Marfan's Syndrome. E Harrison's sulci are associated with an increased risk of sudden infant death syndrome. (True) (True) (True) (True) (False)

Comments: Pectus carinatum (pigeon chest) is found in a number of skeletal dysplasias including Marfan's Syndrome, Noonan's Syndrome, and hyperphosphotasia. Pectus excavatum is found with upper airways obstruction, in severe rickets, following repair of a diaphragmatic hernia, and also Marfan's Syndrome and homocystineurea. Mild or moderate scoliosis does not usually causes a serious restriction of chest

Pediatrics

59

movement. Severe scoliosis, particularly associated with kyphosis can dangerously impair lung function, and is associated with respiratory failure, cor pulmonale or both. There is often an additional diffusion abnormality that aggravates hypoxaemia, and minor respiratory infections may become life- threatening. Klippel-Feil Syndrome is an association of congenital scoliosis, failure of segmentation of cervical vertebrae, renal anomalies, congenital heart disease, congenital elevation of the scapula (Sprengel deformity), and hearing impairment. Harrison's sulci, an anterolateral depression of the thorax at the insertion of the diaphragm, are usually caused by children with recurrent severe retractions because of asthma. A Harrison's groove can also be found in rickets.
145) Human breast milk: A has a sodium concentration of >20 mmol/l. B Is associated with prolonged neonatal jaundice and if so breast feeding must be discontinued. C In infant feeding decreases the likelihood of developing atopic disease in childhood. D Has lower carbohydrate content than cow's milk. E Has lower casein to lactalbumin ratio than cow's milk. (False) (False) (True) (False) (True)

Comments: It is important to know about the basic composition of milks and in particular the main differences between human breast milk and unmodified cow's milk. In addition it is useful to know the composition of a common formula milk and a special preterm formula. Unmodified cow's milk has a high sodium content (>20 mmol/l), whereas in breast and standard formula milks this is low (approx. 6 mmol/l). Cows milk also has a high concentration of potassium, calcium and phosphate. The preterm formulae have a higher sodium content (approx. 14 mmol/l) and more calories, reflecting the increased sodium losses of preterm infants and their greater calorific requirements. Breast, standard formula and preterm formula milks have a higher carbohydrate and lower protein content than unmodified cow's milk. Breast milk is associated with neonatal jaundice since it contains oestrogens (3-alpha, 20beta-pregnanediol) and non-esterified long chain fatty acids which inhibit glucuronyl-1transferase. There is increasing evidence that breast feeding decreases the likelihood of developing atopic disease, particularly if the mother avoids consuming dairy products or other likely allergens.

Pediatrics

60

146) Which of the following statements is/are true? A Babies are able to respond to sounds in utero B Full term babies are unable to follow a large object at birth with their eyes C A 6-week old infant would be able to follow a large object through an arc of 135 degrees D Growth velocity of head decreases with age E A 12-month old infant who keeps falling when starting to walk is likely to have cerebral palsy (True) (False) (True) (True) (False)

Comments: Evidence would suggest that fetuses can respond to sounds in utero. Term babies are able to follow moving objects and by about six weeks this is well developed. Falling when endeavouring to walk at 12 months is perfectly normal.

147) A newborn female baby is noticed to have imperforate anus but to be passing meconium from an opening in the vestibule. A This is a high variety of anorectal anomaly. B It is important to x-ray the spine and sacrum. C A cutback operation without colostomy is probably the appropriate operation. D An intravenous urogram and micturating cystogram should be performed in the neonatal period. E The parents should be told that she would eventually be incontinent of faeces. (False) (True) (True) (True) (False)

Comments: There is a recto vaginal fistula and the site may be low or high. Children who have imperforate anus may also have other congenital anomalies. The acronym VACTERL describes the associated problems that infants with imperforate anus may have: Vertebral defects, Anal atresia, Cardiac anomalies, Tracheoesophageal fistula, Esophageal atresia, Renal anomalies, and Limb anomalies. A perineal anoplasty is the appropriate procedure.The most important prognostic feature is the severitythe imperforate anus and the presence or absence of associated spinal abnormalities. Children with a low lesion, especially those who require only a perineal anoplasty, have a very good chance of having normal stool patterns. Children with spinal abnormalities of the lower sacrum and a high imperforate anus have a poorer chance of obtaining normal bowel function.

Pediatrics

61

148)The following statements regarding systemic lupus erythematosis (SLE) are correct: A When disease is active the levels of complements C3 and C4 are raised. B When evidence of mild nephritis is present, a renal biopsy is unnecessary. C There is a female preponderance of 8:1. D First manifestation of the disease may be idiopathic thrombocytopenia purpura. E There is neurological involvement in about 10% of cases.

(False) (False) (False) (True) (False)

Comments: When SLE is active the serum complement is depressed. C3 and C4 levels can be used to monitor response to treatment. A poor correlation exists between the clinical manifestations and severity of renal involvement. A biopsy is essential in guiding treatment when renal involvement exists. Neurological involvement is common in SLE. Nearly 50% have neurological problems including: personality disorder, seizures, cardiovascular accidents, and a peripheral neuritis (mononeuritis mulitplex).

149) Regarding the defence mechanisms of the lung: A Only particles of <10micrometers in size can reach the terminal bronchioles. B The ciliary beat frequency is usually about 1000 beats/min. C Warming and humidification of inspired air occurs before the level of the tracheal bifurcation. D The respiratory tract proximal to the segmental bronchi may contain commensals. E Pulmonary secretory IgA reaches adult levels in the first month of life.

(False) (True) (True) (False) (True)

Comments: The upper airway warms and humidifies the inspired air, and filters off particles larger than 1005 microns. Only particles below about 1 micron can reach the respiratory bronchioles and air spaces. Intermediate size particles are trapped by the mucociliary blanket, with a normal beat frequency of 1000/min. Each ciliated cell has about 275 cilia, and the blanket moves towards the pharynx at 1cm per minute. The cough reflex can generate pressures of up to 300mmHg, with flow rates of up to 5-6L/sec. Secretory IgA makes up the principle antibody and respiratory secretions in health. 2 molecules of IgA combine with a secretory component to yield a dimer which is highly resistant to digestion by protolytic enzymes. When lung inflammation occurs, IgG and IgM can also be found. The phagocytic ability of alveolar macrophages and the mucociliary mechanism are impaired by: - Alcohol - Ccigarettes Hypoxia - Starvation - Chilling - Steroids - Nitrogen dioxide - Ozone - Hyperoxia - Heroin Anaesthetic gases. Antibacterial killing of macrophages is decreased by: acidosis, uraemia, acute viral infections (especially rubeolla and influenza), and gases such as sulphur, nitrogen dioxide, ozone, chlorine, ammonia, cigarettes. Mucociliary clearance is reduced by: hypothermia, hyperthermia, opiates, hypothyroidism.

Pediatrics

62

150) Polyhydramnios is often associated with the following fetal abnormalities: A Renal agenesis. B Tracheo-oesophageal fistula. C Anencephaly. D Coarctation of the aorta. E Harelip and cleft plate. (False) (True) (True) (False) (False)

Comments: Renal agenesis causes absent urination and oligohydramnios. Polyhydramnios can be due to maternal factors such as maternal diabetes and Rh incompatibility. 20% of babies are born with a congenital defect. Duodenal atresia, GI obstruction and tracheooesophageal fistula are all associated. 22% of babies with polyhydramnios have a congenital heart abnormality, 40% have a GI problem and 26% have neural tube defects. Coarctation and cleft plate are not associated conditions.

151) The following public health measures would reduce the incidence of iron deficiency anaemia: A Using doorstep cow's milk from 6 months of age. B Giving young children tea rather than fruit juice. C Delaying the introduction of mixed feeding until 9 months of age. D Giving 0.5mg per day of elemental iron to all preterm babies. E Continuing breast feeding until a year of age. Comments: The following would achieve primary prevention of iron deficiency anaemia: Provision of adequate iron supplements for premature and low birth weight infants in adequate dosage (2mg/kg of elemental iron per day). Not using unmodified doorstep milk in the first year of life. Although breast milk has a low iron concentration, the relative bioavailability is much higher than from modified or unmodified cow's milk. Not giving young children tea (this reduced iron's bioavailability). Use of follow-on or ordinary infant formulae in the second half of the first year of life. Weaning on to mixed feeding by 6 months of age. Iron supplementation for all children in high risk groups. (False) (False) (False) (False) (True)

Copyright 2002 Dr Colin Melville

Pediatrics

63

152) In resuscitating a collapsed neonate: A 10ml/kg of fluid should be given if hypovolaemia is suspected. B Treatment of metabolic acidosis would be with 10mmol/kg of sodium bicarbonate. C Naloxone may be used to stimulate respiration in a child born by caesarean section under general anaesthesia. D In asystole, the first dose of adrenaline should be 10mcg/kg. E Atropine is frequently useful for bradycardia. (True) (False) (False) (True) (False)

Comments: The asphyxiated neonate remains a relatively common problem. The sequence of resuscitation should follow the usual A, B, C, securing of the airway, establishing aeration of the lungs, and then correcting circulatory disturbance. Bradycardia and asystole are usually secondary to respiratory problems. Adrenaline may be needed in asystole, and fluid in hypovolaemia. Bicarbonate is sometimes needed if the child has developed a severe metabolic acidosis. Naloxone is useful only if the child has received indirect opiates via mother, and atropine is rarely useful.
Copyright 2002 Dr Colin Melville

153) Pulmonary plethora on a chest X-ray is seen in: A Tetralogy of Fallot B Atrial septal defect (ASD). C Partial anomalous pulmonary venous drainage (PAPVD). D Tricuspid atresia. E Ebstein's anomaly. (False) (True) (True) (False) (False)

Comments: Pulmonay vascular markings should be assessed on the chest x-ray of any child suspect of having a congenital heart defect. Increased markings are seen when there is increased flow through the pulmonary vasculature as in a left to right shunt. Increased markings may also be seen when there is outflow obstruction to the left side of the heart such as with total anomalous pulmonary venous drainage and coarctation of the aorta.

Pediatrics

64

154) In a 3-week-old baby with a serum bilirubin level of 275 mcmol/l, the following findings support a diagnosis of breast milk jaundice: A Dark urine. B Conjugated bilirubin of 120 mcmol/l. C Hepatomegaly. D Alkaline phosphatase of 400 IU/l. E Poor weight gain. (False) (False) (False) (False) (False)

Comments: Breast milk jaundice can only be diagnosed after the exclusion of other serious causes of prolonged hyperbilirubinaemia. Breast milk contains steroids which may inhibit glucuronyl transferase. Babies with breast milk jaundice are well and have no evidence of obstructive jaundice (dark urine, pale stools). Conjugated bilirubin levels are less than 10% of the total bilirubin. The liver is not enlarged and serum liver enzyme concentrations are not raised.

155) If a characteristic is normally distributed in a population A this means that most of the population is composed of normal individuals B there will be equal numbers who have more or less of the characteristic than the mean C the median value will be greater than the mean D ten percent of individuals will be beyond two standard deviations from the mean E the mode will be equal to the mean (False) (True) (False) (False) (True)

Comments: a-nonsense b-i.e. the values will be symmetrical about the mean c-median = mean in normal distribution d-about 5% e-mode = mean

156) Human breast milk A can be safely used in babies with galactosaemia B contains more protein per millilitre than cow's milk C contains less fat per millilitre than cow's milk D has a relatively high sodium content E contains more carbohydrate per millilitre than cow's milk (False) (False) (True) (False) (True)

Comments: Human breast milk is produced under the influence of prolactin, has less calorific value than cow's milk (less protein and fat, but more carbohydrates) and a low sodium content. Breast milk is not recommended in galactossaemia due to precipitation of hypoglycaemia.

Pediatrics

65

157) Gaucher's disease A causes accumulation of gangliosides B causes massive splenomegaly C is associated with X-linked inheritance D causes bone erosions E is associated with discolouration of the skin (True) (True) (False) (True) (True)

Comments: a-deficiency of beta-glucocerebrosidase leads to accumulation of gangliosides in nervous tissue (children) or liver, spleen and bone (adults). c-autosomal recessive, chromosome 1 dcortices of long bones e-yellow brown skin

158) A boy of 2 years cannot walk yet. Duchenne muscular dystrophy is unlikely if: A There is no family history of Duchenne muscular dystrophy. B The serum creatinine phosphokinase (CPK) is 236 IU/l. C Facial muscle weakness is present. D A muscle biopsy is normal. E Hyperreflexia is present. (False) (True) (True) (True) (True)

Comments: Approximately one-third of cases of Duchenne muscular dystrophy are new mutations with no past family history of the disease. The legs usually have moderate proximal weakness at the time of presentation, but upper limb and facial musculature may appear relatively normal for several more years. The serum CPK is grossly elevated (>1000 IU/l) and muscle biopsy confirms the diagnosis.

159) The following conditions are associated with characteristic skin disorders: A Glycogen storage disease. B Sturge-Weber syndrome. C Tuberose sclerosis. D Cystinosis. E Essential fatty acid deficiency. Comments: Glycogen storage disease typically affects the muscles and the liver. Sturge Weber is associated with capillary haemangiomas. (False) (True) (True) (False) (True)

Pediatrics

66

Adenoma sebaceum, shagreen patches and ash leaf hypopigmentation are the typical skin lesions of tuberous sclerosis. Cystinosis commonly causes renal tubular defects. Essential fatty acid deficiency causes a generalized scaly dermatitis, and alopecia and thrombocytopenia.

160) A 6 month old baby presents with a generalized purpuric rash, affecting her trunk and extremities virtually equally. The diagnosis includes: A Measles. B Meningococcal septicaemia. C Pseudomonas septicaemia. D Haemophilia. E Giant urticaria. (False) (True) (True) (False) (False)

Comments: Measles rash starts about 14 days after exposure and appears as a generalized macular eruption on face, neck and spreads over 3 days. Meningococcal septicaemia causes a generalized non blanching purpuric eruption. Pseudomonas and other gram-negative septicaemia can cause a rash, eccymoses and petechiae. Haemophilia is not associated with generalized rash. Giant urticaria.is characterized by recurring attacks of transient oedema suddenly appearing in areas of the skin or mucous membranes and occasionally of the viscera, often associated with dermatographism, erythema, and purpura

161) If one carries out a Valsalva manoeuvre against a closed glottis the following will occur: A A rise in intrathoracic pressure. B A rise in left ventricular output. C A rise in blood pressure. D A fall in heart rate. E A fall in right ventricular output. (True) (False) (False) (False) (True)

Comments: Owing to increased intrathoracic pressure, there is a fall in right ventricular output because of decreased venous return to the heart. There is therefore a fall in left ventricular output and a concomitant fall in blood pressure.

Pediatrics

67

162) The following agents are paired with their appropriate mechanism of action: A Vigabatrin - GABA agonist. B Lamotrigine - inhibition of glutamate release. C Methyldopa - beta-adrenergic receptor agonist. D Baclofen - GABA beta receptor agonist. E Acetazolamide - carbonic anhydrase inhibitor. (False) (True) (False) (True) (True)

Comments: Vigabatrin is a recently developed anti-epileptic agent. It acts by selectively and irreversibly inhibiting the enzyme GABA transaminase. This increases the levels of GABA which is the major inhibitory neurotransmitter in the brain. Lamotrigine is another anti-epileptic agent and acts on voltage-sensitive sodium channels. This stabilizes neuronal membranes and glutamate release is inhibited. Glutamate is thought to have a key excitatory role in the generation of epileptic seizures. Methyldopa is a centrally acting antihypertensive agent. It is metabolized to alphamethyladrenaline and stimulates central inhibitory alpha-adrenergic receptors. Baclofen stimulates GABA beta receptors, which inhibit glutamate and aspartate release thus depressing synaptic transmission and relieving spasticity of voluntary muscles.

163) Indications for tonsillectomy include: A Persistently enlarged tonsils B Snoring C Peritonsillar abscess D Recurrent glue ear requiring grommet insertion E More than 3 episodes of tonsillitis requiring antibiotics (False) (False) (False) (False) (True)

Comments: The indications for tonsillectomy are controversial. Undoubtedly, upper airway obstruction, particularly if associated with hypoxaemia or apnoea, are a clear indication. In most cases, recurrent tonsillitis with pus requiring antibiotic therapy would encourage the surgeon to remove the tonsils. This is particularly the case if there has been a previous abscess. However, if there is no history of chronic tonsillitis, the chance of a recurrence of quinsy is only about 10%, so isolated incision and drainage with antibiotics is all that is required.

Pediatrics

68

164) Regarding raised intracranial pressure after head injury: A The pressure usually rises immediately after severe head injury B Causes dilatation of ipsilateral pupil C Causes hemiparesis of the contralateral side D Could lead to irregular respiration E Results in rise in pulse rate and fall in blood pressure (False) (True) (True) (True) (False)

Comments: In the early stages of head injury there is a non-linear relationship between an expanding haematoma and the elevation of the intracranial pressure (ICP) that is a haematoma may expand without any significant rise in pressure. Once this early compliance is lost, the pressure will rapidly rise. This severely jeopardises cerebral perfusion (cerebral perfusion pressure is equal to mean arterial blood pressure minus intracranial pressure). As ICP raises, cerebrospinal fluid is driven out of the intracranial compartment the first stage in compensation. As the ICP continues to rise, brain shifts occur within the cranial cavity. The most important of these brain shifts is uncal transtentorial herniation or coning. This causes impairment of conscious level, development of an ipsilateral fixed dilated pupil (due to third nerve compression), hemiparesis of the contralateral side (due to compression of the cerebral peduncle), and later cardiovascular and respiratory abnormalities due to brain stem compression. The agonal event is often accompanied by a rise in blood pressure and a fall in the pulse rate Cushings reflex.

165) Ventricular septal defects A Usually occur in the muscular part of the septum B Close spontaneously in more than 50% of affected children C Have a benign outcome if accompanied by accentuation of the second heart sound D Causing a large left to right shunt result in the presence of a middiastolic murmur E No longer requires antibiotic prophylaxis with dental extraction (False) (True) (False) (True) (False)

Comments: 80% of VSD s occur in the membranous part of the septum.50% of VSDS are small and over 75% of these close spontaneously, of the remaining moderate and large VSDS 10% can be expected to close spontaneously. 90% Of those destined to close will have done so before the age of 10. A loud second heart sound may indicate the onset of pulmonary hypertension. A large left to right shunt may increase flow across mitral valve and cause a relative mitral stenosis. Unclosed ventricular septal defects always require antibiotic prophylaxis with dental extraction. Following successful operative repair antibiotic prophylaxis is usually required for the first 6 months.

Pediatrics

69

166) Infants of diabetic mothers: A Have a higher than average rate of insulin secretion B Have a higher than expected incidence of the respiratory distress syndrome C Have a higher than average incidence of congenital malformations D Develop hypocalcaemia E Are usually delivered after the expected date of delivery (True) (True) (True) (True) (False)

167) Regarding the incidence and aetiology of cleft lip and palate: A The incidence of cleft lip and palate is one in 8,000 live births B Has a higher incidence among the black population C Isolated cleft palate is more common than isolated cleft lip D Genetic influence is more significant in combined cleft lip and palate E Pierre Robin syndrome remains the most common syndrome associated with clefting (False) (False) (True) (True) (True)

Comments: The incidence of cleft lip and palate is one in 600 live births, and 1:1000 live births for isolated cleft palate. The incidence increases in the Oriental groups (1:500) and decreases in the black population (1:2000). The highest incidence reported for cleft lip and palate occurs in the Native American tribes of Montana, USA (1:276). The typical distribution of cleft types is: cleft lip alone 15%; cleft lip and palate 45%; isolated cleft palate 40%. Genetic influence is more significant in combined cleft lip and palate than cleft palate along where environmental factors such as maternal epilepsy and drugs (e.g. steroids, diazepam, phenytoin) exert a greater influence. Although most clefts of the lip and palate occur as an isolated deformity, the Pierre Robin sequence remains the most common syndrome. This syndrome comprises isolated cleft palate, retrognathia and a posteriorly displaced tongue (glossoptosis), which is associated with respiratory and feeding difficulties.

168) Perthe's disease: A Commonly seen in the 15-20 age group B Up to 50% of patients are obese C Bilateral in more than 30% of cases D There is displacement through the growth plate with the epiphysis always slipping down and back E The x-rays are usually normal Comments: (False) (False) (False) (False) (False)

Pediatrics

70

Perthes disease is osteochondritis of the femoral head and affects those aged 3-11 years [commonest 4-7 years]. It is bilateral in 10% with a M : F ratio of 4:1. Patients (children) present with pain in hip or knees, and with a limp. All movements of the hip are restricted. It is not associated with obesity and obesity is a feature in patients with slipped upper femoral epiphysis. Early x-rays show widening of the joint space; later there is decrease in the size of the nuclear femoral head with patchy density. Normal x-ray is a feature of irritable hip [also called transient synovitis]. The important differential diagnoses for this condition are: Slipped upper femoral epiphysis, Irritable hip and Tubercular arthritis.

169) Sudden cardiac death in an adoloscent can occur in: A Wolf-Parkinson-White Syndrome B severe Aortic Stenosis C Long QT syndrome D atrial septal defect E familial hypercholestolemia (True) (True) (True) (False) (False)

Comments: In WPW fast AF may rapidly deteriorate into VF. In severe AS and Long QT there may be VT.

170) Chikenpox is associated with the following:A pnemonitis B pancreatitis C subacute sclerosing panencephalitis D erythema marginatum E cerebellar encephalopathy (True) (False) (False) (False) (True)

Comments: a-Commoner in adults, resolves with the rash however impaired pulmonary function may last months. B-Mumps. C-Measles. D-Rheumatic fever. E-acute cerebellar ataxia - commonest extracutaneous site in children. Appears 21 days after rash.

Pediatrics

71

171) Tetralogy of Fallot is typically associated with: A Cyanosis in the newborn period. B Right ventricular impulse. C Oligaemic lung fields on chest x-ray. D Turners syndrome. E A loud systolic murmur over the left sternal border. (False) (True) (True) (False) (True)

Comments: Cyanosis develops 3-6 months after birth, rather than in the immediate newborn period. Right ventricular impulse is the result of hypertrophy. Oligaemic lung fields are the result of pulmonary stenosis. Turners syndrome is typically associated with bicuspid aortic valve and coarctation of the aorta. There is a systolic ejection murmur due to pulmonary stenosis.

172) The following are common signs of heart failure in infancy A Intercostal insuction B Basal crepitations C Tachycardia D Raised JVP E Enlarged liver (True) (False) (True) (False) (True)

Comments: Intercostals induction and tachypnoea (60-100 respiration/minute), tachycardia >120-140 bpm, are common. Hepatomegaly is a reliable indicator of effectiveness of therapy and is common. Frank pulmonary oedema is uncommon and raised JVP, though useful in older children is not a reliable clinical sign in infants.

173) Features of Bartter's syndrome are: A Hypertension. B Low urinary chloride. C Hyperreninaemia. D Hyperaldosteronism. E Juxtaglomerular hypoplasia. (False) (False) (True) (True) (False)

Comments: Bartter's syndrome presents with failure to thrive, muscle weakness, polyuria and normal blood pressure. Biochemical features include hypokalaemia (usually <2.5 mmol/l), hypochloridaemia, metabolic alkalosis, high urinary chloride and potassium. Hyperreninaemia, hyperaldosteronism and raised prostaglandin E2 may also be present. Bartter's syndrome can

Pediatrics

72

be inherited as an autosomal condition. The syndrome is thought to be due to a defect in chloride reabsorption in the ascending limb of the loop of Henle.

174) A functional or insignificant heart murmur in a child: A Is diastolic B Is soft and of short duration C Occasionally has an associated thrill D Is usually present along the left sternal edge E Associated finger clubbing would not be expected (False) (True) (False) (True) (True)

Comments: That a murmur is diastolic would imply significance with insignificant murmurs or flow murmurs being systolic of soft intensity and short duration implying insignificant turbulent blood flow and located along the left sternal edge. A thrill implies palpable turbulence, which would be considered significant. Clubbing of the fingers as is found in cyanotic congenital heart disease would not be expected.

175) The mucous membranes may be affected in the following conditions: A Dermatitis herpetiformis. B Lichen planus. C Pityriasis versicolor. D Kawasaki's disease. E Stevens-Johnson syndrome. (True) (True) (False) (True) (True)

Comments: Mucous membranes (MM) may be affected by developmental disorders, infection, acute and chronic skin conditions, genodermatosis, benign and malignant tumours. Dermatitis herpetiformis occasionally has MM involvement, Lichen planus commonly affects the MM. Other important conditions affecting the MM are: Kawasaki's disease, Stevens-Johnson syndrome, toxic shock syndrome, mucocutaneous candidiasis and mucocutaneous leishmaniasis. Some specific conditions of the MM include: Fordyce disease (aberrant sebaceous glands), geographic tongue, cheilitis, oral thrush and aphthous stomatitis.

Pediatrics

73

176) Tardive dyskinesia is a recognized effect of treatment with: A Metoclopromide B Chlormethiazole C Fluphenazine D Propranolol E Temezepam Comments: Neuroleptics can be subdivided into low-potency, midpotency, and high-potency types. Chlorpromazine and thioridazine are both low-potency medicines and usually require a higher dose than the other neuroleptics for symptom remission. They are both rather sedative, producing numerous anticholinergic side effects but causing comparatively fewer extrapyramidal symptoms. Mesoridazine, a midpotency medicine, produces more extrapyramidal symptoms than the low-potency drugs. Thiothixene and haloperidol are highpotency medicines that produce, comparatively, the greatest number of extrapyramidal symptoms. The most worrisome side effect of the neuroleptics is the development of tardive dyskinesia. This is characterized by choreoathetoid movements of trunk, limbs, and facial musculature; these movements develop in approximately 20{endash}-30% of children treated long-term with neuroleptics. Dyskinesia can occur during the treatment with the drug or after it has been discontinued, in which case it is referred to as withdrawal dyskinesia. This latter type of dyskinesia, the symptoms of which can include nausea, vomiting, diaphoresis, ataxia, oral dyskinesia, and various dystonic movements, is reversible in most cases, whereas the dyskinesia developing during drug use may not be reversible. The treatment of tardive dyskinesia involves decreasing or discontinuing the medication if possible, despite the fact that it has been noted that increasing the neuroleptic causes a temporary diminution of dyskinetic symptoms. Prophylactic measures involving drug-free holidays and periodic discontinuation of neuroleptics are also advisable to help mitigate the development of tardive dyskinesia. Metoclopramide can cause TD in addition to extrapyramidal effects (oculogyric crises, especially in the young), and hyperprolactinaemia. Copyright 2002 Dr Colin Melville (True) (False) (True) (False) (False)

177) Characteristic features of lead poisoning in childhood are: A anaemia B blue line at the gingival margin C pica D convulsions E alopecia (True) (True) (False) (True) (False)

Comments: Chronic lead poisoning occurs from environmental ingestion, usually from old layers of paint. Lead prevents iron integration into the porphyrin ring, resulting in hypochromic anaemia and an increase in stainable iron on red cells (sideroblasts). Pica is often associated with iron-

Pediatrics

74

deficiency anaemia, so the child then eats paint getting lead poisoning. Encephalopathy can occur, resulting in fits.

178) Erythema nodosum: A Is characterized by red, tender nodules over the forearm. B Is a recognized feature of tuberculosis. C Commonly occurs in inflammatory bowel disease. D May be associated with viral infections. E Is characterized by target lesions. (True) (True) (False) (True) (False)

Comments: Erythema nodosum is characterized by red tender nodules over the tibia but may also appear on the thighs and forearms. Erythema nodosum is shiny and hot and can be surrounded by bruising. Causes include idiopathic, viral infections, streptococcal infections, ulcerative colitis and Crohn's disease. Erythema nodosum rarely occurs in inflammatory bowel disease. Target lesions refer to the rash seen with erythema multiforme.

179) The following statements about immunodeficiency are correct: A Early onset (<2 years) hypogammaglobulinaemia affects males and females equally. B Selective IgA deficiency is present in about 1 in 500 of the population. C Frequent infections with fungal and viral agents suggest a T-cell deficiency. D Chronic granulomatous disease (CGD) is usually an X-linked condition. E in AIDS (acquired immunodeficiency syndrome) the CD4 to CD8 ratio increases

(False) (True) (True) (True) (False)

Comments: Early hypogammaglobulinaemia is almost always the male sex-linked type (Bruton's). IgA deficiency is one of the most common immunodeficiencies. Abnormalities of cell-mediated immunity are less common than deficiencies of immunoglobulin. Frequent and severe viral and fungal infections or Pneumocystis carinii suggest a T-cell defect. A T-cell deficiency is found in Di George syndrome, Wiskott-Aldrich syndrome, ataxia telangiectasia and chronic mucocutaneous candidiasis. The CD4 to CD8 ratio decreases in AIDS.

Pediatrics

75

180) Atopic eczema: A Usually starts in the first year of life. B Never benefits from dietary measures. C Does not have a genetic basis. D Is a generalized rash over the whole body. E Should be treated in its early stages with topical corticosteroids. (True) (False) (False) (False) (False)

Comments: The typical distribution is face, ears, elbows and knees. Cows milk is a common cause and switching to soya milk for instance may assist. It is more common in those with a family history of asthma, hayfever and eczema. Topical steroids should be applied sparingly only if symptoms cannot be controlled.

181) In acute bronchiolitis A ribavirin is the treatment of choice for hospitalized cases B lung volume is usually decreased C bronchodilators are usually effective D feeding difficulties are common E upper airway obstruction is a common feature (False) (False) (False) (True) (False)

Comments: Acute bronchiolitis presents in infancy with signs of respiratory distress, hyperinflation, inspiratory crackles and often wheezing. Feeding difficulties are common and most hospitalized cases require nasogastrics feeds or intravenous fluids. Bronchodilators are often ineffective and ribavirin treatment is reserved for cases where underlying cardiopulmonary disease or immunological deficiency presents and increased risk of mortality from bronchiolitis.

182) Conditions associated with an increased risk of malignant neoplasia during childhood include: A Aniridia. B Ataxia telangiectasia. C Fanconi's anaemia. D 13q- syndrome. E Peutz-Jegher's syndrome. (True) (True) (True) (True) (False)

Comments: Aniridia (sporadic) along with hemihypertrophy, renal dysplasia and Beckwith-Wiedemann

Pediatrics

76

syndrome are all associated with an increased incidence of Wilms' tumour. Ataxia telangiectasia is associated with an increased incidence of lymphoma and leukaemia. Fanconi's anaemia is also associated with an increased risk of developing leukaemias as well as hepatomas. 13q- syndrome is associated with an increased risk of developing a retinoblastoma. Other important associations include: xeroderma pigmentosa - skin cancer; congenital X-linked immunodeficiency and severe combined immunodeficiency disease lymphoma and leukaemia; familial IgM deficiency and Wiskott-Aldrich syndrome - lymphoma; Down syndrome - leukaemia; 11p- syndrome - Wilms' tumour; von Hippel-Landau syndrome phaeochromocytoma; multiple endocrine adenomatosis I - schwannoma; multiple endocrine adenomatosis II - phaeochromocytoma and thyroid carcinoma; familial polyposis - carcinoma of the colon. In Peutz-Jegher's syndrome the polyps are benign hamartomas but can undergo malignant change in adulthood.

183) Which of the following is true regarding rheumatoid arthritis A Primarily affects the articular cartilage B Is associated with the HLA antigens DR4 and DW4 C Occurs more often in women D The cervical spine is commonly involved E Extra-articular manifestations occur in 20% of patients (True) (False) (True) (False) (True)

Comments: Rheumatoid arthritis is a disease of the synovial membrane. It results in joint destruction, pannus formation and periarticular erosion. DR4 and DW4 are associated with diabetes. The female to male ratio is 3:1. The hand, elbows and knees are the commonest joints involved. Extra-articular manifestations occur in 20% and include keratoconjuctivitis sicca, episcleritis, pulmonary nodules, pleural effusions, pericarditis and subcutaneous rheumatoid nodules.

184) Psoriasis in children is: A Associated with cows milk protein intolerance. B Usually not itchy. C Best treated initially with steroid creams. D A common cause of alopecia. E Sometimes a sequel to staphylococcal infection. (False) (True) (False) (False) (False)

Comments: Eczema is associated with cows milk intolerance and itchy, rather than psoriasis. Coal tar is commonly used initially. Small patches are found on the scalp rather than complete alopecia. There is no known association with infective causes.

Pediatrics

77

185) According to the 1989 Children Act the following principles and provisions are applicable if child abuse is suspected: A An emergency protection order lasts for 8 days. B A child assessment order allows a child to be taken into local authority care. C The welfare of the child is paramount. D The length of an emergency protection order cannot be extended. E A child assessment order can be granted by a local authority. (True) (False) (True) (False) (False)

Comments: In the 1989 Children Act it is stated that the welfare of the child is paramount. An emergency protection order allows a child to be taken into care and lasts for 8 days. It may be extended at the discretion of the court for a further 7 days. After 3 days the parent or the child may challenge the order. A child assessment order allows medical or psychiatric examination to be carried out. A ful court order is necessary. If the parents or carers do not cooperate, the assessment order may be converted to an emergency protection order. From Hannam et al. MRCP (Paediatrics) Part 1 MCQs. page 81 WB Saunders. Reproduced with permission.

186) A term infant has a serum bilirubin of 331 mmol/l (unconjugated 310 mmol/l) and haemoglobin of 8.8 g/dl at 10 days of age. The baby's blood group is A Rhesus negative and the mother's blood group is O Rhesus positive. The direct Coombs test is negative. Phototherapy is started and group O Rhesus negative donor red blood cells are transfused. The bilirubin is found to be 351 mmol/l 6 hours later. The following are diagnostic possibilities: A Physiological jaundice. B Rhesus incompatibility. C ABO incompatibility. D Crigler-Najjar syndrome. E Transfusion mismatch. (True) (False) (True) (True) (False)

Comments: The infant has hyperbilirubinaemia and anaemia, suggesting haemolysis. The mother is Rhesus positive and so Rhesus isoimmunization is not possible. As she is blood group O she has anti-A and anti-B antibodies (IgG) which can cross the placenta and cause haemolysis of the infant's red blood cells. Crigler-Najjar syndrome type I is an autosomal recessive condition leading to absence of uridyl diphosphoglucoronyl transferase (UDPGT). Severe unconjugated hyperbilirubinaemia occurs in the neonatal period. Although haemolysis does not occur as part of this syndrome, this unlikely diagnosis cannot be excluded using the information given in the question. Physiological jaundice presents after the first 24 hours of life, reaches a peak between the 3rd and 5th day at a maximum level of 200-250 mcmol/l and disappears by the 10th to 14th day. Group O blood is considered the universal donor.

Pediatrics

78

187) The following conditions can present with chronic constipation: A Diabetes mellitus. B Hypercalcaemia. C Hypothyroidism. D Coeliac disease. E Abeta-lipoproteinaemia. (True) (True) (True) (False) (False)

Comments: Both diabetes mellitus and hypercalcaemia can present with constipation. This is due to the dehydration that accompanies these conditions. In hypothyroidism there is decreased gut motility. Coeliac disease and abeta-lipoproteinaemia are characteristically associated with diarrhoea.

188) Polyhydramnios is related to: A Duodenal atresia. B Renal agenesis. C Tracheo-oesophageal fistula. D Congenital heart disease. E Neural tube defects. (True) (False) (True) (True) (True)

Comments: Polyhydramnios can be due to maternal factors such as maternal diabetes and Rh incompatibility. 20% of babies are born with a congenital defect. Duodenal atresia, GI obstruction and tracheoesophageal fistula are all associated. 22% of babies with polyhydramnios have a congenital heart abnormality, 40% have a GI problem and 26% have CNS disorders. Renal agenesis will result in absence of urine and oligohydramnios.

Pediatrics

79

189) The following statements regarding a retinoblastoma are correct: A Patients can present with strabismus. B Tumours spontaneously regress in 10% of cases. C All bilateral cases of retinoblastoma are hereditary. D Patients with a family history of retinoblastoma have a 25% chance of transmitting the disease to their children. E Other types of malignancies develop in 15% of children. (True) (False) (True) (False) (True)

Comments: The most common presentation of a Retinoblastoma is with a white reflex. Tumours regress in about 1% of cases and for this reason the parents of a child with the disease should have their eyes examined for signs of retinal scarring. This is important as the tumour is inherited in up to 15% of unilateral tumours. The gene mutation that results in the tumour developing is on chromosome 13q. The inheritance is autosomal dominant with incomplete penetrance and around 80% of children with the abnormal gene develop a retinoblastoma. The most common malignancy is Osteogenic sarcoma of the femur.

190) A diagnosis of Fallots tetralogy is supported by: A Cyanosis at birth. B Left ventricular hypertrophy on ECG. C Episode of squatting. D Pulmonary plethora on chest x-ray. E A murmur loudest at the lower left sternal edge. (False) (False) (True) (False) (False)

Comments: Cyanosis usually presents at 3-6 months of age. The ECG should show Rt. ventricular hypertrophy in this condition. Squatting is a characteristic clinical sign. Lung fields are typically oligaemic and the systolic ejection murmur of pulmonary stenosis would be at the upper rather than lower left sternal edge.

Pediatrics

80

191) The following features are suggestive of supra-ventricular tachycardia rather than sinus tachycardia: A Presence of beat to beat variation. B Known presence of a ventricular septal defect. C Pulse rate of 208/min. D Pulse rate decreasing on carotid massage. E Pulse rate decreasing with D/C shock. (False) (False) (True) (True) (True)

Comments: Variable R-R with a constant P-R interval occurs in sinus tachycardia. Pulse rate of greater than 160 is suggestive of SVT, with greater than 200bpm suggestive of an accessory pathway. Carotid sinus massage and other vagal manouvres may assist in treatment of SVT as will D/C cardioversion. 192) A light-for-dates full-term baby is at particular risk from the following conditions: A Hyaline membrane disease. B Physiological jaundice. C Milk aspiration. D Hypoglycaemia. E Apnoeic attacks. (False) (False) (False) (True) (False)

Comments: Physiological jaundice occurs in 90% of infants. Hyaline membrane disease, apnoeic attacks and aspiration are due to prematurity. Hypoglycaemia (<2 mmol/L) is often accompanied by ketosis and due to depletion of, or failure to utilize, glycogen stores in infants of low birth wt and those born to DM mothers, premature infants and sepsis.

Pediatrics

81

193) The following statements regarding lymphomas in childhood are correct: A Hodgkin's disease is more common than non-Hodgkin's under the age of 5 years. B Hodgkin's disease has equal sex incidence. C Lymphocyte-predominant Hodgkin's disease has the worse prognosis. D The nodular sclerosing variety is the most common form of Hodgkin's disease. E The most common presenting clinical sign is splenomegaly. (False) (False) (False) (True) (False)

Comments: Hodgkin's lymphoma occurs in four forms: (1) Lymphocyte-predominant (10-20%) with the best prognosis. (2) Nodular sclerosing (50%) which is the most common form. (3) Mixed cellularity (40-50%) which is most likely to have extranodular disease at presentation. (4) Lymphocyte depleted (<10%) which is the rarest type with the worst prognosis. Hodgkin's disease is rarely found in children aged less than 5 years old (male: female ratio=2:1) and peaks at between 15 and 34 years. Non-Hodgkin's disease is more common in younger children (male: femalratio=3:1). The most common presenting clinical sign is enlarged cervical lymph nodes.

194) A neonate presents with floppiness. The following results of investigations suggest the diagnoses indicated: A Markedly elevated CPK - Duchenne or Becker Muscular Dystrophy B Myopathic electromyogram - Guillain-Barr syndrome C Decromental response on electromyogram - myasthenia gravis D Degenerating and regenerating myofibres on muscle biopsy - Duchenne Muscular Dystrophy E Branch chain ketoaciduria - isovaleric acidemia (True) (False) (True) (True) (False)

Comments: Serum CPK tends to be markedly elevated in Duchenne and Becker Muscular Dystrophies. These conditions also show focal death and regeneration of muscle cells, resulting in the hypertrophy characteristic of the later child. Electromyography may be useful in diagnosing myasthenia gravis, as repeated stimulation results in progressive adduction in the response. Guillain Barr Syndrome is associated with greatly reduced motor nerve conduction velocities, and sensory nerve conduction is also often slow. The electromyogram shows evidence of acute denevation of muscle. Serum CK may be mildly elevated or normal.

Pediatrics

82

195) The fourth cranial nerve: A Enters orbit through superior orbital fissure. B Supplies superior oblique muscle. C Causes a convergent squint. D Traverses the petrous temporal bone. E Runs in the lateral wall of the cavernous sinus. (True) (True) (False) (False) (True)

Comments: The fourth cranial nerve supplies the superior oblique (RL6SO4). Lateral rectus palsy causes a convergent squint.
196) Which of the following statements is/are true regarding atopic eczema. A Often presents in early childhood. B May be familial. C Is commonly associated with asthma. D Commonly requires treatment with oral steroids. E Can be confused with scabies. (True) (True) (True) (False) (True)

Comments: The onset is commonly in infancy, and it is associated with a history of asthma, eczema and hayfever in the family. Topical rather than oral steroids are used sparingly for unresolved symptoms. It is itchy, and can therefore be confused with scabies.

197) Supraventricular tachycardia: A Often responds to Digoxin B May be associated with structural cardiac defects C In infancy is associated with maternal systemic lupus erythematosus D Episodes are often difficult to document on ECG E Can cause feeding problems in infants (True) (True) (True) (False) (True)

Comments: Digoxin may be administered in all forms of supraventricular tachycardia in which the AV node is involved excepting in those with a pre excitation syndrome that are greater than 1 yr old. SVT though most commonly associated with a structurally normal heart with an accessory pathway, may be a feature of congenital heart disease. Transplacental passage of anti Ro SSA and anti La SSB antibodies in maternal SLE can result in neonatal lupus syndrome and associated cardiac rhythm disturbances, most commonly complete heart block. Feeding difficulty is a common manifestation of cardiac insufficiency.

Pediatrics

83

198) Cot death (sudden unexpected death in infancy): A Has a peak incidence at 6 months of age. B Is associated with strong evidence of Mendelian inheritance. C Has no social class association. D Is more common in winter months. E Does not occur in infants born at term. (False) (False) (False) (True) (False)

Comments: The peak incidence of cot death is between 2nd and 4th month of life. The cause is unknown and probably related to neural, cardiorespiratory control mechanisms. Although it is more common in babies with a sibling affected by SIDS, inheritance is unknown. It is more common in cold months, premature infants, lower socio-economic classes, and infants who have had apnoeic episodes requiring resuscitation.

199) A moderately dehydrated baby with acute gastroenteritis: A Has a fluid deficit of about 100ml per kilogram B Is more likely to have hypernatraemia than hyponatraemia C Will have loss of skin turgor D If breast-fed, should stop oral feeding E If formula-fed will need a lactose-free milk on commencing oral feeding (True) (False) (True) (False) (False)

Comments: A moderately dehydrated baby has a loss of 6-9% of body weight and a fluid deficit of around 100mls/kg. Hyponatraemia or a normal serum sodium is more likely to be present than hypernatraemia. Decreased skin tone and tissue turgor will be present, also oliguria, dry mucous membranes and sunken fontanelle. Breast feeding should be continued and supplement oral feeds may be required to maintain adequate volumes. Formula fed infants should not have their formula changed and milk feeding should not be discontinued.

Pediatrics

84

200) The following milk feeds are lactose free: A Formula S. B Wysoy. C Pregestemil. D SMA gold cap. E Cow and Gate premium. (True) (True) (True) (False) (False)

Comments: Lactase seems to be the most vulnerable brush border enzyme. For this reason activity can be reduced after insults to the mucosal lining of the gastrointestinal tract. This can happen after acute gastroenteritis, coeliac disease in relapse, after hypoxic episodes and in protein energy malnutrition. Recovery is the general rule in such cases after treatment with lactose-free milks. Formula S is soya-based milk, as is Wysoy. Pregestemil is whole-protein-free milk and contains medium chain triglycerides for ease of absorption.

201) Recognised features of functional (innocent) cardiac murmur in childhood include: A Maximum site of intensity is at the apex B Shortness of duration C An accompanying thrill D An accompanying click E An increase in intensity during an intercurrent infection (False) (True) (False) (False) (True)

Comments: Insignificant flow murmurs are systolic, of short duration and located along the LSE. Thrills imply significant turbulence and hence diseased valve.An accompanying click is a feature of aortic or pulmonary stenotic lesions and mitral valve prolapse. Innocent murmurs will be accentuated in hyperdynamic states such as fever and exercise when cardiac output increases.

Pediatrics

85

202) Antidiuretic hormone (ADH): A Is stored in the anterior pituitary gland. B Release is stimulated by hypernatraemia. C Release is inhibited by hypotension. D Release is stimulated by mannitol. E Leads to water reabsorption by increasing the water permeability of the ascending loop of Henle. (False) (True) (False) (True) (False)

Comments: ADH is a peptide hormone that is synthesized in the hypothalamus and stored in the posterior pituitary. Release is mainly stimulated by increased plasma osmolality, which may be secondary to hypernatraemia, hyperglycaemia or mannitol therapy. When plasma volume decreases significantly, non-osmotic stimulation of ADH secretion occurs via aortic and left atrial baroreceptors. ADH increases the water permeability of the renal medullary collecting ducts, thus leading to the formation of concentrated urine. The ascending loop of Henle is impermeable to water. 203) Causes of jaundice in the newborn include A hypothyroidism B administration of phenobarbitone C post-maturity D a large cephalhaematoma E congenital adrenal hyperplasia (True) (False) (False) (True) (False)

Comments: Newborn jaundice may be physiological associated with prematurity, due to birth trauma, biliary atresia, ABO/rhesus incompatibility, galactossaemia, hypothyroidism or CMV infection to name but a few. Treatment is usually unnecessary. In all cases, it is important to keep the baby well-hydrated (breast milk and formula are preferable to glucose water) and encourage frequent bowel movements by feeding frequently. This is because broken down bilirubin is carried out of the body by the intestines in the stools (bilirubin is what gives stool their brown color). Sometimes artificial lights are used on infants whose levels are very high, or in premature infants. These lights work by helping to break down bilirubin in the skin. In the most severe cases of jaundice, an exchange transfusion is required.

Pediatrics

86

204) The following findings in the first week of life require investigation: A Erythema toxicum neonatorum. B Subconjunctival haemorrhage. C A bulging fontanelle. D Failure to pass meconium in the first 48 hours in a term infant. E A squint. (False) (False) (True) (True) (False)

Comments: Erythema toxicum is a common rash of unknown aetiology which does not require investigation. Conjunctival haemorrhages commonly result from birth trauma and frequently seen in the first week of life. A bulging fontanelle can be due to raised intracranial pressure, meningitis, intracranial bleeds or hydrocephalus. Most term infants pass meconium in the first 24 hours. Delay beyond this period can occur in cystic fibrosis, bowel atresia and Hirschsprung's disease. Full conjugate eye movement has usually developed by 6 months. In the first weeks of life squints are common and are usually of no consequence.

205) For a group of young men with height 160-190cm the PEFR was measured and the results plotted on a scatter diagram. A the correlation coefficient can't be higher than 1 B if the correlation coefficient is 0 there is is no relationship between height and PEFR C if the correlation coefficient is positive, the curve would have an upward slope D this data can be used to calculate the PEFR for a height of 150cm E height should be on the vertical axis on the graph (True) (True) (True) (True) (False)

Comments: a- -1 to +1 d- if a correlation can be made then figures can be extrapolated and 150cm isn't too far from 160cm e-PEFR is the dependent variable and is usually put on the Y (vertical) axis. (height is fixed and is on the X (horizontal axis).

Pediatrics

87

206) The following laboratory results were returned in a 6 week old boy admitted with 6 days of severe projectile vomiting: pH PO2 PCO2 Blood Urea Sodium Potassium Chloride 7.51 (7.35-7.45) 12 KPa (95 mmHg) 4.7 KPa (35 mmHg) 11 mmol/l 131 mmol/l 3 mmol/l 83 mmol/l (False) (False) (True) (False) (False)

A He has respiratory alkalosis B He is likely to have a bulging anterior fontanelle C He might have pyloric stenosis D He should be commenced immediately on half strength soy protein, low lactose formula E X-ray of abdomen is likely to show dilated loops of small bowel

Comments: He has a metabolic, rather than respiratory alkalosis as CO2 is not reduced. He is likely to have a sunken fontanelle as he is very dehydrated. Pyloric stenosis would highly likely as because of the age and hypochloraemic, hypokalaemic metabolic alkalosis. He should be resuscitated with normal saline first. Dilated bowel would not be expected, as pyloric rather than small bowel obstruction is present.
Cognitive deficit is found in: A alkaptonuria B maple syrup urine disease C homocystinuria D galactosaemia E Wilsons disease (False) (True) (True) (True) (False)

Comments: 1.Symptomless in childhood, produces homogentisic aciduria. An Autosomal Recessive condition, ochronosis develops later in life with or without arthritis. 2.Neurological disturbances appear soon after birth. There are increased amounts of valine, isoleucine, and leucine in blood and urine, due to a defect in the metabolism of these branched chain amino-acids. 3.Mental retardation, fair hair, lens dislocation. 4.Causes irreversible brain damage if the diagnosis is not made early, and the child placed on a lactose free diet. 5.Neurological impairment is associated with involvement of the basal ganglia, and cognitive deficit is not normally observed.

Pediatrics

88

Vitamin E deficiency: A is associated with retinopathy of prematurity. B can cause cerebellar ataxia. C is associated with abetalipoproteinaemia. D may present with haemolytic anaemia. E can be diagnosed by increased red blood cell haemolysis with hydrogen peroxide. (True) (True) (True) (True) (True)

Comments: Vitamin E acts as a biological reducing agent and prevents damage to cell membranes by high levels of oxygen. Premature infants absorb vitamin E poorly and this is thought to be one of the aetiological factors that lead to retinopathy of prematurity. Premature infants are also susceptible to haemolytic anaemia caused by vitamin E deficiency as another role of the vitamin is to stabilize red blood cell membranes. One of the diagnostic tests for vitamin E deficiency is that the red blood cells have an increased susceptibility to haemolysis by hydrogen peroxide. For this reason all premature infants should have vitamin E supplementation. Chronic vitamin E deficiency can lead to progressive areflexia, cerebellar ataxia and ophthalmoplegia. Treatment is with tocopherol at a dose of 10mg/kg/day. Owing to poor absorption this dose has to be increased to 100mg/kg/day in abetalipoproteinaemia. From Hannam et al. MRCP (Paediatrics) Part 1 MCQs. page 117 WB Saunders. Reproduced with permission.

An umbilical hernia is commonly found in: A Preterm babies. B Infants of diabetic mothers. C Downs syndrome. D Congenital hypothyroidism. E Turners syndrome. (True) (False) (True) (True) (False)

Comments: Umbilical hernias occur more frequently in infants of African American descent. ,premature babies and females.The vast majority of umbilical hernias are not related to any disease condition. However, umbilical hernias can be associated with rare diseases, such as mucopolysaccharide storage diseases, Beckwith-Wiedemann syndrome, Down syndrome, and others including congenital hypothyroidism

Retinitis pigmentosa can be found in the following conditions: A Tay-Sachs' disease. B Lawrence-Moon-Biedel syndrome. (False) (True)

Pediatrics

89

C Krabbe disease. D Abetalipoproteinaemia. E Refsum's disease.

(False) (True) (True)

Comments: Retinitis pigmentosa is a progressive degeneration of the retina with characteristic changes in pigment and a degree of optic atrophy. As well as being a disease entity in its own right it is seen in many other conditions leading to increasing visual impairment. Lawrence-Moon-Biedel syndrome is the association of hypothalamic and pituitary dysfunction with mental retardation, post-axial polydactyly, renal problems and retinitis pigmentosa. Refsum's disease is diagnosed by increased phytanic acid in the serum. It is inherited as an autosomal recessive trait and presents with failing vision as a result of retinitis pigmentosa, weakness and unsteady gait. From Hannam et al. MRCP (Paediatrics) Part 1 MCQs. page 181 WB Saunders. Reproduced with permission.

Inability to do which of the following would be of concern in a baby of 9 months? A sit unaided B use words with meaning C use pincer grip D put food in the mouth E change objects from one hand to the other Comments: bc should be attained at 10 months so may not be achieved in a baby of 9 months. (True) (False) (False) (True) (True)

The following are features of premature thelarche: A Peak onset between 6 months and 2 years. B Association with Turner syndrome. C Asymmetrical breast enlargement. D Associated growth spurt. E Areola pigmentation. (True) (False) (True) (True) (True)

Comments: Premature thelarche is common, with a peak incidence between 6 months and 2 years. It may be asymmetrical, and is differentiated from true precocious puberty by the absence of pubic hair development and a growth spurt. It is usually self-limiting.

In congenital heart disease:

Pediatrics

90

A A ventricular septal defect may close spontaneously in less than 20% of cases. B The commonest form of cyanotic heart disease presenting in the newborn period is Fallots tetralogy. C Hepatosplenomegaly may be an important clue to diagnosis. D When congenital heart disease is associated with congenital rubella it is commonly a persistent patent ductus arteriosus. E A persistent patent ductus arteriosus is the commonest cardiology complication of Downs syndrome.

(False) (True) (True) (True) (False)

Comments: The rate of spontaneous closure is in over 50% of cases. The most common form of cyanotic heart disease in the newborn period is Fallot's followed by transposition of the great arteries. Hepatosplenomegaly is a feature of congestive cardiac failure,PDA is a sequelae of congenital rubella as are pulmonary stenotic lesions and ASD. The commonest defect in Downs syndrome is an endocardial cushion defect.

Marfan syndrome is associated with: A pulmonary stenosis B increased upper : lower body ratio C dislocation of the lens D cognitive impairment E autosomal recessive inheritance Comments: b - Upper to lower body ratio (head to symphysis pubis : Symphysis pubis to toes) is decreased in Marfan Syndrome. (Dr Vajira H. W. Dissanayake) (False) (False) (True) (False) (False)

Ghent Criteria (American Journal of Medical Genetics 1996;62:417-426) Diagnostic criteria for Marfan syndrome Criteria Family history Genetics Cardiovascular Major Independent diagnosis in parent, child, None sibling Mutation fbn1 Aortic root dilation; dissection of ascending aorta None Mitral valve prolapse; calcification of the mitral valve (<40 years); Dilation of the pulmonary artery; Dilation/dissection of the descending aorta Two needed - flat cornea, myopia, elongated globe Minor

Ocular

Ectopia lentis

Pediatrics

91

Skeletal

Four needed - pectus excavatum needing surgery, pectus carinatum, pes planus, wrist and thumb sign, scoliosis >20 or spondylolisthesis, arm span-height ratio >1.05, protrusio acetabulae (x-ray, MRI), diminished extension elbows (<170)

Two to three major, or one major and two minor signs Moderate pectus excavatum; High, narrowly arched palate; Typical facies; joint hypermobility

Pulmonary

Spontaneous pneumothorax; apical bulla Unexplained stretch marks (striae); Recurrent or incisional herniae Lumbosacral dural ectasia (CT or MRI)

Skin Central nervous system

CT Computed tomography; MRI Magnetic resonance imaging Lesions in the following may cause weakness of finger movements: A Contralateral motor cortex B Inferior cerebellar peduncle C Thalamus D Inferior parietal lobule E Pre-central area Comments: Weakness of finger movements may result from lesions anywhere in the motor pathway form the cortex to the fingertips. This includes the motor cortex (precentral area), the internal capsule, a path through the brainstem to the decosation of the pyramids, the posterior columns, the anterior horn cells, the lower motor neurone, or the motor N plates in the muscles cells. Copyright 2002 Dr Colin Melville In Turner's syndrome: A most affected fetuses will abort in early pregnancy. B growth hormone has no effect on final adult height. C pulmonary stenosis commonly occurs. D horseshoe kidney occurs in about 50% of cases. E expected IQ is about 80. (True) (False) (False) (True) (False) (True) (False) (False) (False) (True)

Comments: Turner's syndrome (45,X) occurs in about 1 in 3000 live births; 95% of all 45,XO conceptions

Pediatrics

92

are aborted. Recombinant human growth hormone has been given for the short stature and has a modest effect on final height. Coarctation of the aorta occurs in about 15% of cases, isolated non-stenotic bicuspid valve has been detected in about one-third of cases, and horseshoe kidney in 50% of cases. IQ is usually normal. From Hannam et al. MRCP (Paediatrics) Part 1 MCQs. page 183 WB Saunders. Reproduced with permission.

The following are abnormal findings in a 7-month-old infant: A extensor plantar reflex. B Moro reflex. C asymmetrical tonic neck reflex. D obligatory palmar grasp. E crossed adductor spread. (False) (True) (True) (True) (False)

Comments: Plantar reflexes are normally extensor in the newborn and this usually persists for most of the first year of life (found in approximately 75% of normal infants at 12 months of age in one study). The Moro and asymmetrical tonic neck reflexes are abnormal if they persist beyond 6 months of age. The obligatory palmar grasp which is present at birth normally disappears by 2 months of age and is replaced by voluntary grasping from about 4 months of age. Elicitation of the knee tendon jerk at birth usually causes crossed adductor spasm. This usually disappears by 8 months of age. From Hannam et al. MRCP (Paediatrics) Part 1 MCQs. page 63 WB Saunders. Reproduced with permission.

The following statements are true for these minor anomalies of the newborn: A Capillary haemangiomata occur in 30-50% of babies. B Mongolian spots do not occur in Caucasian infants. C Erythema neonatorum toxicum requires no treatment. D Breast enlargement only occurs in female infants. E Epsteins pearls occur on the nose and cheeks. (True) (False) (True) (False) (False)

Comments: Capillary haemangiomata occur quite frequently and are commonly in the periorbital region. Mongolian spots can occur more commonly in children of African or Asian descent, but up to 10% of fair skinned infants. Erythema toxicum is self limiting and benign eruptions occurring in healthy newborns in the neonatal period. Breat enlargement can occur in male infants and is physiological. Epsteins pearls are white epithelial inclusion cysts occurring in the midline of the palate in newborns.

Decreased glucose and high polymorphonuclear cell count in the CSF may be seen in:

Pediatrics

93

A Echo virus meningitis B early tuberculous meningitis C E. coli meningitis D meningococcal meningitis E cryptococcal meningitis

(False) (True) (True) (True) (True)

Comments: a-Though it is seen in 1/4 cases of mumps and herpes virus meningitis. b-Lymphocytes usually predominate except in early disease where there may be 80% polymorphs. c+d Bacterial meningitis. e-The glucose is usually reduced OTM 3e, 14.2.6).

The following are true regarding Marfan's syndrome: A there is genetic anticipation B in about 50% it is the result of spontaneous mutations C affected patients are prone to Aortic dissection D Subluxation of lens is a feature E 10% have hypercalcaemia (False) (False) (True) (True) (False)

Comments: Marfan's syndrome is a connective tissue disease with an autosomal dominant inheritance and an incidence of 4-6 per 100 000. People with Marfan's syndrome used to have a life expectancy reduced by 50% but this is now changing because of improved treatment of cardiovascular abnormalities such as mitral valve prolapse and aortic dissection. b-25%, anterior subluxation of the lens is a feature. (Dr Vajira H. W. Dissanayake)

In tetralogy of Fallot: A There is usually a pansystolic murmur at the left sternal edge B Affected infants are usually cyanosed at birth C Both ventricles are usually hypertrophied D Cyanotic spells may be helped by Propranolol E Chest x-ray characteristically shows pulmonary plethora (False) (False) (False) (True) (False)

Comments: There is systolic ejection murmur with this condition. Cyanosis develops at 3-6 months and may occur in later childhood. The right ventricle is hypertrophied. Cyanotic spells can be relieved by squatting which decreases systemic venous return, oxygen, morphine, intravascular volume expansion and phenylephrine. Propanolol is used to prevent spells. Chest x-ray characteristically shows a boot shaped heart and pulmonary blood flow is reduced due to infundibular stenosis.

Pediatrics

94

In a normal 6 month old child, the following reflexes are present: A Grasp reflex B Ankle reflex C Startle reflex D Parachute reflex E Tonic neck reflex Comments: The primitive reflexes appear and disappear in sequence during specific periods of development. Their absence or persistence beyond a given time frame signifies significant dysfunction of the CNS. Some primitive reflexes such as the rooting reflex, may reappear during old age or with specific degenerative diseases involving the cerebral cortex. The moro reflex is obtained by placing the child in a semi-upright position and allowing the head to fall backward momentarily with immediate resupport by the examiners hand. There is a symmetrical abduction and extension of the arms with flexion of the thumbs followed by a flexion and adduction of the upper extremities. An asymmetric response may signify a fractured clavicle, brachioplexus injury or a hemiparesis. Absence of the moro reflex in a term newborn is ominous suggesting significant CNS dysfunction. The grasp reflex is elicited by placing a finger or an object in the open palm of each hand. The normal infant will grasp the object and with attempted removal, the grasp is reinforced. The tonic neck reflex is produced by manually turning the head to one side while supine. Extension of the arm occurs on that side of the body corresponding to the direction of the face, while flexion develops in the contralateral extremities. An obligatory tonic neck response by which the infant remains locked in the fencer's position, is always abnormal and implies a disorder of the CNS. The parachute reflex is demonstrated by suspending the child by the trunk and suddenly producing forward flexion as if the child were going to fall. The child spontaneously extends the upper extremities as a protective mechanism. This appears before the onset of walking. Most primitive reflexes disappear gradually and are absent by 2-3 months. Copyright 2002 Dr Colin Melville (False) (True) (False) (False) (False)

A baby weighing 4.54kg at birth at term to a well controlled insulin dependent diabetic mother: A Has a greater than expected chance of having a major error of morphogenesis B Will become a fat child C Has a 20% chance of developing diabetes by 20 years D Is still prone to hypoglycaemia E Should not be breast fed

(True) (False) (False) (True) (False)

Comments: Malformations of organ systems are increased two to threefold in infants of diabetic mothers.

Pediatrics

95

The risk of obesity is in 50% of offspring. Foetal macrosomia (birthweight above 4kg) results from excessive foetal insulin, which causes hypoglycaemia. The risk of Type 1 diabetes developing in the child is 6.3% if the father is affected and 1.3% if the mother is affected. There is no contraindication to breast feeding.

In pulmonary atresia A cyanosis is present B convulsions occur C a continuous murmur is characteristic D an ECG can differentiate this condition from Fallot's tetralogy E squatting improves the symptoms (True) (True) (True) (False) (True)

Comments: Pulmonary atresia is associated with cyanosis, heart failure in infancy. Kyphoscoliosis is a frequent accompaniment. Convulsions may arise from associated cerebral ischaemia and polycythaemia. The murmur in pulmonary atresia is a frequently a combination of Tricuspid regurgitation and patent ductus arteriosus.

Convulsions in childhood may be caused by: A fever B teething C hypocalcaemia D phenylketonuria E hypertonic dehydration (True) (False) (True) (True) (True)

Comments: 1.This is the commonest cause of convulsions in childhood 2.Teething does not cause convulsions directly or indirectly 3.A well recognised and important cause 4.Now rarely seen because of effective screening procedures 5.An important cause of convulsions especially in infancy

In the fragile-X syndrome: A Mental retardation is found in most males B Mental retardation is not found in females C Cytogenetic finding of fragile-X is induced in a vitamin B12 deficiency culture medium D Affected children are taller than average (True) (False) (False) (True)

Pediatrics

96

E Diagnosis can now be made in the pre-natal period

(True)

Comments: Affected males are usually mentally retarded and exhibit characteristic physical features. One third of females with the mutation will be mentally retarded. The diagnosis of fragile X syndrome was originally based on the expression of a folate sensitive fragile X site (X q27.3) induced in cell culture under conditions of folate deprivation, rather than B12 deficient. Affected children are taller and may have high arched palate, long ears, a long face and macroorchidism. Diagnosis can be made by detection of the mutant FMR 1 gene by chorionic villus sampling , confirmatory amniocentesis may be required in some cases.

The following statements are true: A adrenaline has alpha-adrenergic activity only. B dopamine has both alpha- and beta-adrenergic activity. C isoprenaline acts by alpha-adrenergic stimulation. D alpha-adrenergic stimulation results in vasodilatation. E salbutamol is a beta 2 agonist. (False) (True) (False) (False) (True)

Comments: Adrenaline has alpha- and beta-adrenergic activity. Dopamine has dopaminergic activity at low doses and alpha- and beta-adrenergic activity at higher doses. Isoprenaline is a non-specific beta agonist, whereas salbutamol is a beta 2 agonist. Alpha-adrenergic stimulation leads to vasoconstriction. From Hannam et al. MRCP (Paediatrics) Part 1 MCQs. page 91 WB Saunders. Reproduced with permission.

A normal baby of 12 months, born at term would be expected to be able to: A get into a sitting position. B build a tower of four bricks. C have hearing reliably assessed using a distraction hearing test. D feed with a spoon. E point to eyes, nose and mouth. (True) (False) (True) (False) (False)

Comments: It is essential to have some knowledge of each of the four areas of child development (i.e. gross motor, fine motor and vision, hearing and speech and social behaviour) for several age groups. We suggest learning two to three normal milestones for each area of development for the following ages: 6 weeks, 6-8 months, 12 months, 18 months, 2.5 years, 3 years, 4 years, and 5 years. From Hannam et al. MRCP (Paediatrics) Part 1 MCQs. page 11 WB Saunders. Reproduced with permission.

Pediatrics

97

Osteolytic foci shown on X-ray in a 2 year old child may be caused by: A rickets B histiocytosis X C infectious mononucleosis D leukaemia E rheumatoid arthritis (False) (True) (False) (True) (False)

Comments: Radiolucent bone lesions in children may be caused by histiocytosis X (eosinophilic granulomas), leukaemia, Wilm's tumour and with osteomyelitis. Osteomalacia causes Looser's zones - pseudofractures. Read more Congenital torticollis A Appears after the third month of birth B Facial asymmetry is a common presentation C Tilt and rotation of the neck to the same side of swelling D The ear is nearer the shoulder on the normal side E Physiotherapy is effective in the early stages (False) (True) (False) (False) (True)

Comments: Congenital torticollis develops as a result of birth injury to the sternocleidomastoid muscle. It can present from the second week of birth as a swelling within the sternocleidomastoid muscle. Once the swelling subsides there is subsequent fibrosis resulting in tilt and rotation of the neck to the opposite side. Therefore the ear on the affected side is nearer the shoulder. Facial asymmetry is a common clinical presentation. In the early stages, physiotherapy to lengthen the muscle is beneficial. If the condition persists, surgical treatment in the form of division and release of the muscle at its lower end may be required.

The following statements are true regarding toxic shock syndrome: A Usually the result of burns of >15-20% total body surface area B Is caused by Steptococcus faecalis C Is associated with a diffuse macular rash and diarrhoea D Could lead to a fall in haemoglobin E Cerebral oedema is a recognised late complication (False) (False) (True) (True) (True)

Comments: This alarming syndrome could result even from small scalds in young children. The onset is sudden with vomiting, diarrhoea, temperature >40 degrees, diffuse macular rash which later desquamates, tachynoea and oliguria. These signs usually develop on the third or fourth day

Pediatrics

98

after sustaining the scald/burn and are associated with a drop in haemoglobin and white cell count. It is caused due to a toxin produced by Staphylococcus aureus phage type 29/52. Late complications include irritability, cerebral oedema, convulsions and coma. Treatment should be prompt with control of temperature by vasodilatation, reduction of cerebral oedema, if necessary by hyperventilation, and the administration of whole blood, immunoglobulins and antibiotics.

Associations have been established between: A Adrenal hyperactivity and excess skin melanin B Black eye and haemosiderin C Calcification around renal tubules and excess of vitamin B D Fibrocystic disease of the pancreas and mucous secretion defects E Renal stones and cystinuria (False) (True) (False) (True) (True)

Comments: Excess skin melanin can be the result of hypoadrenalism and high circulating ACTH and POMC. Haemosiderin deposition occurs causing pigmentation. Excessive B6 may cause symptoms of a peripheral neuropathy, but calcification is not associated. Cystic fibrosis is associated with defective mucous secretion hence complications such as bronchiectasis and meconium ileus. Cystinuria is associated with nephrolithiasis.

Mastocytosis: A is characterized by aggregates of tissue mast cells in the epidermis. B commonly presents with pruritis. C may be exacerbated by aspirin. D takes the form of a solitary mastocytoma in about 50% of cases. E most commonly takes the form of urticaria pigmentosa. (False) (True) (True) (False) (True)

Comments: Mastocytosis encompasses a spectrum of disorders (mastocytoma, urticaria pigmentosa, diffuse mastocytosis and systemic mastocytosis) with aggregates of mast cells in the dermis. Symptoms result from the release of histamine, which produces a flush, tachycardia, headache, diarrhoea, hypotension, syncope and respiratory distress. It is exacerbated by hot baths, vigorous rubbing, aspirin, codeine, morphine, atropine and polymyxin B. About 10% have a solitary lesion, i.e. mastocytoma. From Hannam et al. MRCP (Paediatrics) Part 1 MCQs. page 61 WB Saunders. Reproduced with permission.

The following statements regarding dental development are correct: A calcification of the teeth starts in the seventh month of fetal life. (True)

Pediatrics

99

B in most children, deciduous teeth have started to erupt by 12 months. C the first permanent teeth to erupt are molars. D eruption of deciduous teeth can be delayed in hypothyroidism. E calcification of permanent teeth starts at 2 years.

(True) (False) (True) (False)

Comments: Calcification of teeth begins in the 7th month of fetal life. Initially this is seen in the deciduous teeth until shortly before term when calcification of permanent teeth begins. In most infants eruption of deciduous teeth has started by 6 months although this can be delayed by factors such as malnutrition and hypothyroidism. From Hannam et al. MRCP (Paediatrics) Part 1 MCQs. page 29 WB Saunders. Reproduced with permission.

In a patient with a head injury leading to a six hour coma, the following factors will affect CNS outcome: A Right parietal injury compared to left parietal injury B Age C Presence of chest injury D Hemiplegia E Glasgow coma scale at 24 hours Comments: Major insults to the brain may result from contusion (particularly to the frontal lobes or inferolateral portion of the temporal lobes) a penetrating injury, the presence of an intracerebral haemorrhage, and diffuse axonal injury due to shear forces, all of which may produce cerebral oedema. The incidence of space-occupying intracranial haematoma is less frequent in children than in adults, but the presence of diffuse cerebral swelling due to diffuse axonal injury is more frequent, particularly in a severe paediatric head injury. The clinical findings depend on the site and nature of the injury and on the degree of cerebral oedema. The development of additional neurologic signs following admission to the hospital implies increased cerebral oedema; an expanding intracerebral, subdural, or epidural haematoma; or compromised cerebral blood flow secondary to vasospasm. Initial signs associated with a poor prognosis include fixed and dilated pupils, apnoeic breathing, decorticate posturing, and a Glasgow Coma Scale score revised for children of less than 5. Some children are found to have cardiorespiratory arrest and hypotension at the site of an accident following a head injury, which is unrelated to acute blood loss. Cervical spine radiographs often show an injury to the high cervical spine, and postmortem examination shows intraparenchymal haemorrhage or laceration. Although this type of injury is uniformly fatal, it serves as a reminder to consider a spinal cord injury in every child with a severe head injury until appropriate radiographic studies are completed. The most important determinant of neurologic and intellectual recovery in the head-injured child is the duration of coma. If the child survives the immediate consequences of the head injury and recovers from coma within 14 days, the likelihood of normal or near-normal cognitive and neuromotor function is extremely favourable. The reasons for optimism in the child compared with the adult is the contention that the former's brain is more "plastic" than the (True) (True) (True) (True) (True)

Pediatrics100

adult's and generally recovers more completely in a shorter time. However, infants less than 2 year of age with major brain trauma have a uniformly poor prognosis compared with older children, perhaps as the result of immature autoregulation of the cerebral blood vessels, the greater susceptibility of the incompletely myelinated brain to irreversible injury, and the fact that open cranial sutures permit greater distortion among the meninges, cerebral vessels, and the underlying brain. The most sensitive and specific indicator of neurologic recovery during the acute and early stages of a head injury is an evaluation with somatosensory evoked potentials. Late post-traumatic seizures tend to develop within 2 year of the initial insult. Posttraumatic epilepsy is more likely to occur if the original trauma to the brain was severe and the dura disrupted. Copyright 2002 Dr Colin Melville

Which of the following is true of patent ductus arteriosus: A Functional closure in the normal infant term occurs within the first 24 hours. B Pulmonary atresia is a duct-dependent malformation. C It is possible to keep the ductus open by indomethacin. D Its incidence is lower in preterm babies. E Its incidence is lower in children in high altitude communities.

(True) (True) (False) (False) (False)

Comments: The physiological function of the ductus arteriosus in the fetus is to divert blood away from lungs to the placenta, constriction occurs in the first 24 hours of life in response to increased environmental oxygen. In order to maintain life some cardiac malformations such as pulmonary atresia, hypoplastic left ventricle and interrupted aortic arch require the patency of a wide PDA after birth. Indomethacin causes duct closure through an anti- prostaglandin effect. Incidence of PDA is greater in prefer infants and birth in a high altitude environment is associated with greater risk.

At birth a baby has A stepping reflex B its own IgG C its own IgM D positive Babinski response E nasal respiration (True) (False) (False) (True) (True)

Comments: 2. has maternal IgG which can cross the placenta 3. IgM does not cross the placenta, and does not reach adult levels until the infant is 2-5 months old. 4. upgoing plantars can be normal up to the age of one year

Pediatrics101

A newborn infant has ambiguous genitalia. The following statements are true: A assignment of gender is primarily dependent on karyotype. B if gonads are palpable they are likely to be testes. C the infant should be raised as a boy in all cases of male pseudohermaphroditism. D if the karyotype is 46XX, 21-hydroxylase deficiency is the most likely diagnosis. E gonads that are discordant for the assigned gender should be removed. (False) (True) (False) (True) (True)

Comments: In cases of ambiguous genitalia the determination of sex for rearing should be primarily determined by the feasibility of anatomic reconstruction and appropriate hormonal treatment rather than karyotype. The broad categories of diagnosis are female pseudohermaphroditism (most commonly secondary to 21-hydroxylase deficiency), male pseudohermaphroditism or true hermaphroditism (rare). Male pseudohermaphrodites may have disorders of testosterone synthesis or partial end organ resistance to testosterone (incomplete testicular feminization). The latter should never be raised as males as they never virilize. Gonads that are either discordant for the assigned gender or dysgenetic are surgically removed to reduce the risk of later malignancy. From Hannam et al. MRCP (Paediatrics) Part 1 MCQs. page 55 WB Saunders. Reproduced with permission

Papilloedema in pre-school children can be caused by: A Hyperthyroidism B Pseudohypothyroidism C Nephrotic syndrome D TB meningitis E Posterior fossa tumour Comments: Papilloedema is swelling of the optic nerve head due to raised intracranial pressure. If the same appearance arises from an intrinsic lesion of the optic nerve, it is known as papillitis. The ophthalmoscopic appearances are identical of these 2 conditions. Papilloedema of recent onset usually produces little or no change in visual acuity, and papillitis is associated with a severe decrease in vision (central scitoma). Causes in childhood include: hydrocephalus, brain tumours, cerebral abscess, intracranial bleed. Copyright 2002 Dr Colin Melville (False) (False) (False) (True) (True)

A 5-year-old boy refuses to go to school. The following statements regarding school phobia are correct:

Pediatrics102

A it is more common in boys than girls. B it is associated with a low IQ. C somatic complaints often accompany the phobia. D truancy is common in children who have school refusal. E school phobia has a better prognosis in early childhood than in adolescence.

(False) (False) (True) (False) (True)

Comments: In truancy, children leave home but do not arrive at school or leave early. It is commonly associated with a low IQ and is more common in boys than girls, unlike school phobia which has an equal sex ratio. Often in children suffering from school phobia there are functional symptoms present such as abdominal pain or headaches. The condition carries a better prognosis if treated and recognized at an earlier age. From Hannam et al. MRCP (Paediatrics) Part 1 MCQs. page 175 WB Saunders. Reproduced with permission.

Recognised causes of hypertension in childhood include A chronic glomerulonephritis B renal vein thrombosis C neuroblastoma D coarctation of the aorta E congenital adrenal hyperplasia (True) (True) (True) (True) (True)

Comments: Investigation of hypertension in childhood must always include examination of the urine for albumin, red cells and casts. This will exclude chronic glomerulonephritis. In addition urine should be collected to detect the presence (or absence) of catecholamines. Hypertension can be seen in congenital adrenal hyperplasia in both 11-beta-hydroxylase and 17 hydroxylase deficiencies. From Hannam et al. MRCP (Paediatrics) Part 1 MCQs. page 3 WB Saunders. Reproduced with permission. Which of the following is true of isolated ventricular septal defects in childhood A A loud systolic murmur on Day 10 indicates a large defect. B A loud first heart sound indicates pulmonary hypertension. C Heart failure uncommonly presents in the first month of life. D There is no need to give ampicillin before dental extraction. E Surgical closure is needed if the defect does not close by the age of five. Comments: (False) (False) (True) (False) (False)

Pediatrics103

Small defects may present with loud murmurs. A loud pulmonary component of the second heart sound indicates pulmonary hypertension. The first clinical signs of a VSD are not apparent usually till after 1 month. Prophylactic antibiotics are mandatory before dental extraction. Over 50% of the defects close spontaneously.

Breath holding attacks: A Are commoner over the age of 3 years. B Can be confused with a generalised convulsion. C May be precipitated by a minor injury. D Should be treated with sedatives. E Are never fatal. (False) (True) (True) (False) (True)

Comments: The attacks are commonest in infants and younger children, typically occur during crying and emotional outbursts and there is voluntary holding of breath followed by cyanosis and loss of consciousness. The can be precipitated by crying, minor injuries and convulsive movements develop following cyanosis. The cyanosis always precedes the convulsions. They are self resolving and not fatal.

A baby with left congenital diaphragmatic hernia presented with dyspnoea and cyanosis one hour after birth. Which of the following features would be expected? A The abdomen would also be distended. B Both apex beat and trachea are likely to be deviated to the right. C The first resuscitative measure should be to supply oxygen and assist ventilation with a face mask. D The most common site of the diaphragmatic defect is posterolaterally. E The prognosis would be better if the baby had presented later at 48 hours after birth. (False) (True) (False) (True) (True)

Comments: The displacement of abdominal contents into the thoracic cavity occurs.The stomach and abdominal contents cause displacement of thoracic structures to the right, and lung collapse ensues. A diaphragmatic hernia is a surgical emergency. The abdominal organs must be replaced into the abdominal cavity, and the opening in the diaphragm repaired. Invasive ventillation is necessary immediately after birth until the infant recovers from surgery. Some infants are placed on ECMO (extracorporeal membrane oxygenation) which is a heart/lung bypass machine which enables the lungs to expand after surgery.

Left ventricular hypertrophy can occur in:

Pediatrics104

A Tetralogy of Fallot. B Ventricular septal defect. C Atrial septal defect. D Patent ductus arteriosus. E Coarctation of aorta.

(False) (True) (False) (True) (True)

Comments: Right ventricular hypertrophy occurs with tetralogy of Fallot and left ventricular hypertrophy occurs with a moderate sized VSD, PDA and coarctation. Right atrial enlargement occurs with ASD.

A right sided lesion of the 6th cranial nerve: A Causes diplopia when looking to the right. B Causes a convergent strabismus. C The false image is to the left of the true image.. D The false image is parallel to the true image. E The images separate wider when looking to the left. Comments: Abnormalities of III, IV and VI (RL6SO4)result in defects with: Eyelids, ptosis, palpebral fissures. Pupils, size, shape, symmetry and reflexes. Eye movements, diplopia nystagmus. The abducent (VI) arises from the lower pons anterior to the 4th ventricle. It runs a long course intracranially, and is liable to stretching. It supplies the lateral rectus. It contains both parasympathetic and sympathetic fibres. The median longitudinal bundle connects III, IV and VI with vestibular inputs, and co-ordinates lateral gaze. Copyright 2002 Dr Colin Melville (True) (True) (False) (False) (False)

A neonate is born with a supraventricular tachycardia which had been diagnosed antenatally. It has a blood pressure of 30/10, an enlarged liver and a pulse rate of 220/min. The following statements regarding management of the baby are correct: A vagatonic manoeuvres are not appropriate in this situation. B adenosine is the intravenous treatment of choice. C DC cardioversion at 10 J/kg may be used. D intravenous verapamil and propranolol may be of use in this situation. E in one-third of cases of SVT there is an underlying congenital cardiac lesion. (False) (True) (False) (False) (True)

Pediatrics105

Comments: Although in this situation it is unlikely that vagal stimulation would stop the arrhythmia, it is worth attempting unilateral carotid massage even when there is evidence that the infant is in a haemodynamically compromised state. Adenosine at a starting dose of 50 mcg/kg can be very effective at rapidly converting an SVT thythm to a normal sinus rhythm. It works by slowing conduction through the A-V node which interrupts re-entry circuits. DC cardioversion at 0.5-1 J/kg can be extremely effective in returning the heart to a sinus rhythm, especially when the infant is in cardiac failure. Verapamil and propranolol should never be used together as hypotension and asystole can be precipitated. Verapamil is not recommended in children. From Hannam et al. MRCP (Paediatrics) Part 1 MCQs. page 7 WB Saunders. Reproduced with permission.

Infants of diabetic mothers are at increased risk from: A Hypocalcaemia B Anaemia C Jaundice D Shoulder dystocia E Congenital abnormalities (True) (False) (True) (True) (True)

Comments: Hypoglycaemia, hypocalcaemia and hyperbilirubinaemia are common metabolic derangements. Polycythaemia rather than anaemia is associated. Shoulder dystocia results from the increased birthweight and length of each infant. Malformations of organ systems are increased 2-3 fold. Caudal regression syndrome i.e. hypoplasia of lower segment of the body and congenital heart disease are examples.

The following statements regarding Wilson's disease are correct: A low caeruloplasmin levels are diagnostic of Wilson's disease. B Wilson's disease can present as a deterioration in school performance. C colchicine challenge is a useful diagnostic test. D total serum copper levels are always high. E siblings of a child with this condition have a 1 in 4 chance of having Wilson's disease. (False) (True) (False) (False) (True)

Comments: Wilson's disease is inherited as an autosomal recessive condition. It is essential that asymptomatic siblings and cousins of children with the condition are fully investigated. Although most children with Wilson's disease have a low caeruloplasmin level, levels can be normal in between 4 and 20% of patients. Low values can also be found in heterozygotes for the condition as well as in nephrotic syndrome, severe malabsorption, fulminant hepatitis, chronic active hepatitis and tyrosinaemia. The pathognomonic sign of Wilson's disease is the presence of a Kayser-Fleischer ring which can be seen on slit lamp examination. As well as

Pediatrics106

hepatic manifestations of the disease, Wilson's disease can present with neurological or psychological symptoms. Urinary copper excretion, pre-and post-penicillamine challenge, should be measured in any patient with Kayser-Fleischer rings. Total serum copper can be normal, high or low. From Hannam et al. MRCP (Paediatrics) Part 1 MCQs. page 159 WB Saunders. Reproduced with permission. Surfactant production: A takes place in type 1 pneumocytes. B can be detected at 20 weeks gestation. C is decreased by maternal corticosteroid administration. D is increased by intrauterine growth retardation (IUGR). E is decreased by maternal opiate abuse. (False) (True) (False) (True) (False)

Comments: Surfactant is produced by type 2 pneumocytes. Although small amounts may be detected as early as 20 weeks gestation, physiologically significant quantities may not be produced until 30-32 weeks. Surfactant production may be accelerated by several maternally ingested substances including glucocorticoids and opiates. IUGR and other causes of intrauterine 'stress' may also enhance production of surfactant. From Hannam et al. MRCP (Paediatrics) Part 1 MCQs. page 169 WB Saunders. Reproduced with permission.

The following suggest a diagnosis of epiglottitis rather than croup: A Onset over days B Severe barking cough C Temperature of 38C D A toxic, ill-looking child E Drooling of saliva Comments: Croup tends to have an onset over days, with preceding coryza. The child remains able to drink and is not drooling. The child does not look particularly unwell and the fever is low grade. The symptoms of cough and stridor are relatively marked and the voice is hoarse. With epiglottitis, the onset is rapid with no history of preceding coryza. There may be an absence of Hib immunisation. The child is toxic and ill, is drooling saliva, and is unable to drink, often with a high fever of above 38.5C. The child is reluctant to speak with a soft whispering stridor and absent or slight cough. Copyright 2002 Dr Colin Melville (False) (False) (False) (True) (True)

Most children with Downs syndrome:

Pediatrics107

A Require institutional care B Require special schooling C Have associated epilepsy D Cannot be trained to live independently E Develop autistic symptoms

(False) (True) (False) (True) (False)

Comments: Most children with Downs syndrome can be brought up at home, however due to the high frequency of mental retardation they cannot attend normal classes or be trained to live independently in adulthood. Epilepsy risk is not greater in these individuals. Autism occurs in about 10% of cases.

The following statements regarding the diagnosis and management of iritis are correct: A the pupil of the affected eye will be dilated and irregular. B if the eye is painful a diagnosis of iritis is unlikely. C the most common systemic association with iritis in children is with juvenile rheumatoid arthritis. D atropine can be used in therapy. E retinal detachment is a recognized complication. (False) (False) (True) (True) (True)

Comments: Inflammation of the anterior part of the uveal tract can present either as iritis or cyclitis (ciliary body inflammation). Iritis presents with photophobia, pain, blurred vision and a small and irregular pupil. Iritis is most commonly associated with juvenile rheumatoid arthritis although it is also seen in toxoplasmosis, histoplasmosis, sarcoidosis, tuberculosis, mumps, measles and herpes simplex infection. A cycloplegic such as atropine is given to dilate the pupil and relieve iris spasm. Topical corticosteroids are also indicated as long as a viral infection is not thought to be present. Complications such as retinal detachment and glaucoma are sometimes seen. From Hannam et al. MRCP (Paediatrics) Part 1 MCQs. page 97 WB Saunders. Reproduced with permission.

In the management of burns in a specialised burns unit: A In a fit 34-year old male with full-thickness burns involving 55% total body surface area, late debridement and grafting is recommended B Skin grafting is absolutely contraindicated in the presence of Enterococcus faecalis in the wound C Meshed skin grafts achieve superior cosmetic results compared to unmeshed skin grafts D Autografting (during the first surgery) is always possible in burns involving up to 60% total body surface area

(False) (False) (False) (False)

Pediatrics108

E Scalds always cause more skin damage (in terms of burn depth) than chemical burns

(False)

Comments: In a patient with full-thickness burns, early excision of all burned skin is recommended. Grafting, if not possible, could be done at a later time. Enterococcus faecalis is a common skin contaminant in burns and its colonisation does not preclude to successful skin grafting. Unmeshed skin grafts achieve superior cosmetic results compared to meshed skin grafts. Meshed grafts are recommended in areas where continued oozing is anticipated since meshing prevents development of underlying haematoma. Autografting is not possible in burns involving >40% total body surface area without reusing the donor site again in the next 7-10 days. Chemicals burns cause extensive skin damage. Unlike scalds, chemical burns that appear superficial at presentation soon progress to full-thickness (deep dermal).

In coarctation of the aorta: A congestive failure is usually due to hypertension B rupture of the aorta is a recognised complication C cerebrovascular haemorrhage is a known hazard D infective endocarditis of the bicuspid valves is a very rare occurence E Associated hypertension is most frequently due to isolated systolic hypertension. (False) (True) (True) (False) (False)

Comments: a-Usually secondary to valvular disease. Aortic rupture may occur due to associated medial disease and stroke as a consequnece of hypertension is also a feature. Bicuspid valvular disease is more frequently associated with calcification but prophylaxis to SBE should be offered in reference to certain procedures due to the risk of infective endocarditis. Combined diastolic and systolic hypertension is typical.

The following statements on Down's syndrome are correct: A the overall risk of having a child with Down's syndrome, irrespective of maternal age, is 1 in 650 live births. B the risk of having another baby with trisomy 21 after the birth of a child with Down's syndrome is the same as that seen in the general population. C Down's syndrome is due to a 14;21 translocation in 30% of cases. D hyperthyroidism is commonly found in children with Down's syndrome. E 30% of children with Down's syndrome have congenital cardiac defects.

(True)

(False) (False) (False) (True)

Comments: The risk of having a child with Down's syndrome increases with maternal age, reaching 1 in 40 in women over the age of 44 years. After having had one child with Down's syndrome the risk of having another child with trisomy 21 is 1 in 200. In about 5% of cases, Down's syndrome

Pediatrics109

can be shown to be due to a translocation. In less than half these cases one of the parents can be shown to be carrying a balanced version of the translocation. Hypothyroidism is commonly seen in children with Down's syndrome and thyroid function tests should be performed annually. From Hannam et al. MRCP (Paediatrics) Part 1 MCQs. page 77 WB Saunders. Reproduced with permission.

Physiological jaundice in the full term newborn infant is associated with: A Urobilinogen in the urine. B Large doses of Vitamin K. C Pregnanediol in breast milk. D Reticulocytosis. E Itching in the infant. (False) (False) (False) (False) (False)

Comments: Physiological jaundice in the newborn occurs in 90% of infants and is a result of increased rate of red cell breakdown and hence bilirubin accumulation secondary to impaired hepatic clearance.

A baby girl is considered normal at birth but on the second day of age cyanosis is noted. No murmur is heard and there is no respiratory distress. On the third day, cyanosis is more obvious and the respiratory rate is increased. Which of the following is correct concerning this patient? A Cyanosis is most likely due to congenital heart defect B The babys colour should improve rapidly after giving 100% 02 C Echocardiography can often tell the anatomic state of the newborns circulation D The babys chest x-ray shows a small heart. This is against the diagnosis of congenital heart disease. E The lack of murmur is against the diagnosis of CHD. (True) (False) (True) (False) (False)

Comments: Cyanosis is most likely due to a congenital heart defect and a right to left shunt. 100% Oxygen will not improve the degree of cyanosis in the presence of a right to left shunt where deoxygenated blood is entering the systemic circulation. Echocardiogram has a high diagnostic accuracy rate and increasingly surgery is performed following echo findings without interim catheterisation. Certain congenital heart defects such as tetralogy of fallot are associated with small cardiac size. Cyanosis is often the only clinical sign present in patients with CHD.

Concerning the development of the cardiovascular system:

Pediatrics110

A the main pulmonary artery develops from the left 6th branchial arch. B a right-sided aortic arch is usually present at 10 weeks. C the ductus arteriosus is the dorsal remnant of the right sided aortic arch. D the septum secundum grows up as a limbus of the foramen ovale. E atrioventricular septal defects (AVSD) may involve defects of the septum primum.

(True) (True) (True) (True) (True)

Comments: The blood and cardiovascular system are derived form mesoderm and first appear in the middle of the 3rd week. Bilateral ventral tubes fuse and by 21 days there is a single ventral heart tube that begins to beat by about the 23rd day. By 28 days the definitive chambers of the heart are apparent. In the 5th week, septal ridges grow together to septate the heart. The septum primum appears as a crescent-shaped downward growth of the postero-superior wall of the common atrium. Before the septum primum is complete, small openings appear in the upper portion; these merge to form the ostium secundum. On the right of the septum primum the thicker septum secundum grows down. It becomes complete except for a defect that becomes the oval foramen. From Hannam et al. MRCP (Paediatrics) Part 1 MCQs. page 45 WB Saunders. Reproduced with permission.

Which of the following are true of osteomyelitis? A Is most commonly due to a staphylococcus aureus infection B Can be due to salmonella infection in patients with sickle cell anaemia C Infection usually involves the metaphysis of long bones D Dead bone within the medullary canal is known as the involucrum E New bone forming beneath the periosteum is known a the sequestrum (True) (True) (True) (False) (False)

Comments: Osteomyelitis is usually due to staphylococcus aureus infection. Streptococcus pyogenes, Haemophilis influenza and gram-negative organisms can also infect bone. In children the infection is usually aquired by haematogenous spread to the metaphysis of long bones. The dead bone within the medullary canal is the sequestrum. New subperiosteal bone formation is the involucrum.

In vascular endothelium: A nitric oxide (NO) is produced from citrulline by the action of enzyme nitric oxide synthatase (NOS). B endothelial derived relaxing factor (EDRF) is the molecule NO. C NO produces vasorelaxation by increasing cGMP production. D released endogenous NO has a half-life of about 20 seconds.

(False) (True) (True) (False)

Pediatrics111

E tachyphylaxis in the pulmonary circulation commonly occurs when NO is used as an inhaled therapy.

(False)

Comments: Arginine under the action of NOS gives rise to NO and citrulline. EDRF has been shown to be the molecule NO, which has a free radical structure with an extra electron. For this reason it is short lived with a half-life of about 6-7 seconds. Tachyphylaxis has rarely been reported. From Hannam et al. MRCP (Paediatrics) Part 1 MCQs. page 131 WB Saunders. Reproduced with permission.

Increased gastric emptying that may alter the rate of drug absorption is found in the following conditions: A coeliac disease. B raised intracranial pressure. C migraine. D duodenal ulcer. E gastric ulcer. (True) (False) (False) (True) (False)

Comments: Gastric emptying is an important determinant of rate and sometimes extent of drug absorption. Increased gastric emptying is associated with coeliac disease, duodenal ulcer and gastroenterostomy. Decreased gastric emptying occurs with raised intracranial pressure, migraine, pyloric stenosis, trauma, severe pain, gastric ulcer and intestinal obstruction. From Hannam et al. MRCP (Paediatrics) Part 1 MCQs. page 51 WB Saunders. Reproduced with permission.

Immunoglobulin G (IgG): A crosses the placenta. B is secreted in breast milk in significant quantities. C levels decrease in preterm neonates after exchange transfusion. D levels are lowest 4-6 months post-term. E provides immunity to tuberculosis. (True) (False) (False) (True) (False)

Comments: Immunoglobulin G (IgG)crosses the placenta from as early as 12 weeks gestation. This is initially a slow process and transfer increases rapidly only after about 30 weeks gestation. Postnatal levels of IgG decline, reaching a nadir at approximately 4-6 months, until endogenous production becomes significant. The antibody content of colostrum and breast milk is almost entirely immunoglobulin A. Cell-mediated mechanisms provide immunity to tuberculosis. From Hannam et al. MRCP (Paediatrics) Part 1 MCQs. page 15 WB Saunders. Reproduced with permission

Pediatrics112

The presence of the following are characteristic features of a large ventricular septal defect: A Heart failure. B Diminished second heart sound. C Mid-diastolic murmur at the apex. D A plethoric lung field on the CXR. E Repeated chest infection. (True) (False) (True) (True) (True)

Comments: Heart failure, pulmonary plethora and mid diastolic murmur due to increased flow across the mitral valve imply significant shunting. Repeated chest infections are a manifestation of cardiac failure. A large shunt will case pulmonary hypertension and a loud second heart sound. A three month old infant presents with acute dyspnoea and cyanosis. On examination the pulse rate is 180 per minute, he has a grade pan systolic murmur at the left sternal edge, basal crepitations and a liver measuring 4cm below the right costal margin. A The most likely diagnosis is a ventricular septal defect B He requires a diuretic C He should be digitalised with 50ug/kg of Digoxin given over 24 hours D He should make a spontaneous recovery E Chest x-ray would be likely to show a small cardiac shadow (False) (True) (True) (False) (False)

Comments: Ventricular septal defects cause shunting of oxygenated blood from the left ventricle to the right. Cyanosis is a later occurrence-following the development of Eisenmengers syndrome ie shunt reversal. Diuretics are required to offload pulmonary venous congestion. Digoxin has a positive inotropic effect. Cyanotic congenital heart disease requires surgical correction of vascular or shunt anomalies. The differential diagnosis of cyanosis and congestive cardiac failure in neonates includes transposition of great vessels, total anomalous pulmonary venous drainage, hypoplastic left heart , single ventricle and tricuspid valve abnormalities and each is associated with cardiomegaly.

The following are true for errors of morphogenesis (congenital abnormalities): A Teratogens can cause abnormalities by interfering with organogenesis. B Malformations and deformations are synonymous. C 30% of babies born to mothers with insulin dependent diabetes mellitus present at birth with multiple congenital abnormalities. D Single gene defects account for the majority of congenital heart defects. (True) (False) (False) (False)

Pediatrics113

E In the newborn with multiple congenital abnormalities chromosomal aberrations are always found.

(False)

Comments: Malformations include any structural, functional or biochemical abnormality determined genetically or induced during gestation and not due to birthing events. Deformation is a specific developmental anomaly of one organ or tissue. The incidence of birth defects is approximately 1%. Teratogens, maternal infections such as rubella may be the cause of multiple congenital abnormalities without any chromosomal abnormalities.

The following statements are correct: A following the introduction of the MMR vaccine it is no longer necessary to vaccinate against rubella in 10 to 14-year-old girls. B the Hib vaccine is a capsular conjugated polysaccharide. C immunoglobulin must not be given with the MMR vaccine. D hepatitis B vaccine is contraindicated in pregnancy. E the mumps component of the MMR is a live vaccine.

(False) (True) (True) (False) (True)

Comments: From September 1994 the MR vaccine should be given to all children aged 5-16 years in school. There are two brands of Hib vaccines, both of which are capsular conjugated polysaccharide vaccines. They are not interchangeable since they are conjugated with different proteins. Immunoglobulin must not be given with the MMR since the immune response to rubella and mumps may be inhibited. Live vaccines are not routinely given to pregnant women but if tehre is a significant risk of exposure the benefits of vaccination may outweigh the risks to the fetus. Since the hepatitis B vaccine contains inactivated virus surface antigen (HBsAg) it can be given in pregnancy. All parts of the MMR are live. From Hannam et al. MRCP (Paediatrics) Part 1 MCQs. page 83 WB Saunders. Reproduced with permission. The following statements are correct: A it is recommended that folate supplements should be taken by all pregnant women in order to prevent neural tube defects. B the dose of folate necessary to prevent neural tube defects in a first pregnancy is 5 mg. C pregnant women who are taking phenytoin may need to increase their dose of folate. D the recommended dose of folate in a pregnant woman who has had a previous baby with a neural tube defect is 5 mg. E supplementation with folate should continue throughout pregnancy.

(True) (False) (True) (True) (False)

Comments: It has been proven that supplementing the diet with folate can prevent neural tube defects. Prevention in women in their first pregnancy should start in the preconception period. A dose of 400 mcg is recommended and should be continued until the 12th week of pregnancy. In a

Pediatrics114

woman who has previously had a baby with a neural tube defect a higher dose of 5mg is recommended. Folate levels can be reduced in women on phenytoin. For this reason pregnant women on this medication should have an increased dose of folate. From Hannam et al. MRCP (Paediatrics) Part 1 MCQs. page 15 WB Saunders. Reproduced with permission.

Cyanosis in infancy is a common feature of A Ventricular septal defect B Transposition of the great vessels C Atrial septal defect D Coarctation of the aorta E Tricuspid atresia (False) (True) (False) (False) (True)

Comments: Cyanosis is not a feature of VSD unless associated with Eisenmengers syndrome. Transposition of the great arteries, Fallots tetralogy, tricuspid atresia and single ventricle are associated with cyanosis. ASD is associated with increased pulmonary flow. Coarctation causes obstruction to outflow with ventricular hypertrophy and hypertension (usually in the upper limbs) with radio-femoral delay.

Regarding Ewings sarcoma: A Commonly arises towards the end of long bones in the metaphysis B Is more common in children C Associated with a t11:22 chromosomal translocation D X-rays show bone destruction and new bone formation, often with marked periosteal elevation E Could arise from long-standing Pagets disease (False) (True) (True) (False) (False)

Comments: Ewings sarcoma is a malignant round-cell tumour that occurs in the diaphysis of long bones (Osteosarcoma affects the metaphysis) and limb girdles, usually in children. The x-ray appearance described in the choice of answers is a feature of osteosarcoma; in Ewings sarcoma, x-rays often show a large soft-tissue mass with concentric layers of new bone formation known as onion-peel sign. Pagets disease could lead to Osteosarcoma in approximately 10% of patients affected for >10 years, but not to Ewings sarcoma.

In childhood offensive loose stools may be caused by: A increased secretions of pancreatic enzymes. B Bodian-Schwachmann syndrome. C biliary atresia. (False) (True) (True)

Pediatrics115

D cystic fibrosis. E abetalipoproteinaemia.

(True) (True)

Comments: A decrease or absence of pancreatic enzyme secretion may cause loose, offensive stools. Chest infections or respiratory signs are suggestive of cystic fibrosis. Bodian-Schwachmann syndrome presents with failure to thrive in the absence of respiratory symptoms, persistent neutropenia, thrombocytopenia, raised fetal haemoglobin and minor liver enzyme abnormalities. Growth retardation, skeletal dysostoses and bone marrow hypoplasia may also be present. Abetalipoproteinaemia results from a defective production of apoprotein B by the intestinal cells leading to defective synthesis of low-density lipoproteins, very low-density lipoproteins and chylomicrons (CLM). Absence of CLM results in fat malabsorption, deficiency of fat-soluble vitamins and steatorrhoea. Very low cholesterol (<1.3 mmol/l), low triglyceride and acanthocytes on the blood film are found. From Hannam et al. MRCP (Paediatrics) Part 1 MCQs. page 105 WB Saunders. Reproduced with permission.

In the management of congenital heart disease: A Patients with ventricular septal defects require daily prophylactic ampicillin to prevent bacterial endocarditis B Ductus arteriosus that does not close by 6 months old will require surgical ligation. C All atrial septal defects have to be closed surgically. D Peripheral oedema is an uncommon finding in infants with heart failure. E Squatting is a typical description in patients with transposition of the great vessels.

(False) (True) (False) (True) (False)

Comments: Prophylactic antibiotics are only required prior to dental procedures, surgery and other invasive procedures. If spontaneous closure of a patent ductus arteriosis does not occur, surgical ligation is recommended between 6 months to one year.ASD s may spontaneously close usually if less than 8 mm in size. Oedema and ascites and raised jugular venous pressure are uncommon signs of heart failure in infancy. Squatting is a typical occurrence in an older child with tetralogy of fallot.

Hypoglycaemia is a presenting feature in the following disorders: A Nesidioblastosis B Congenital hypothyroidism C Reyes syndrome D Galactosaemia E Phenylketonuria Comments: (True) (False) (True) (True) (False)

Pediatrics116

Nesidioblastosis is a form of pancreatic beta cell hyperplasia, which causes endogenous hyperinsulinism. Hypoglycaemia is not generally a feature of congenital hypothyroidism. Reyes syndrome results in hepatic failure and reduction in glycogen. Galactosaemia is an inborn error of metabolism associated with fatty infiltration of the liver. Phenylketonuria does not affect gluconeogenesis and therefore does not cause hypoglycamia, brain damage and epilepsy result from disordered neurotransmission.

The following statements regarding glomerular disease are correct: A antigen-antibody immune complexes activate complement via the alternate pathway. B crescent formation can be present in Bowman's space. C Berger's disease has only IgA mesangial deposits. D children aged from 2 to 3 years are most commonly affected by the post streptococcal glomerulonephritis. E diffuse thickening of the glomerular basement membrane is the most common cause of nephrotic syndrome in adults.

(False) (True) (False) (False) (True)

Comments: Antigen-antibody immune complexes may produce glomerular disease but via activation of the classical pathway. Crescents are the result of proliferation of parietal epithelial cells in Bowman's capsule. Berger's disease has predominantly IgA deposits but smaller amounts of IgG, IgM, C3 and properidin can be present. Post-streptococcal nephritis raarely occurs in children aged less than 3 years. From Hannam et al. MRCP (Paediatrics) Part 1 MCQs. page 185 WB Saunders. Reproduced with permission. The following are inherited as autosomal dominants: A Phenylketonuria B Achondroplasia C Haemophilia D Glucose-6-phosphate dehydrogenase deficiency E Tuberose sclerosis (False) (True) (False) (False) (True)

Comments: PKU is autosomal recessive. Achondroplasia and Tuberous sclerosis are autosomal dominant. G6PDH deficiency and Haemophilia A and B are X-linked recessive conditions.

Complications of cystic fibrosis include: A biliary cirrhosis B nasal polyps C constipation (True) (True) (True)

Pediatrics117

D diabetes mellitus E rectal prolapse

(True) (True)

Comments: Symptomatic cirrhosis occurs in 2-3% of patients with cystic fibrosis; 25% have changes at post-mortem in the liver. Nasal polyps are common (15-20%). Constipation can be due to excessive pancreatic supplements, meconium ileus equivalent, intestinal stricture secondary to pancreatic supplements or intussusception. Diabetes mellitus may occur at any age and is not related to the disease severity. It is often mild and may not need insulin therapy. Rectal prolapse occurs most frequently in infants and is related to steatorrhoea, malnutrition and repetitive cough. From Hannam et al. MRCP (Paediatrics) Part 1 MCQs. page 143 WB Saunders. Reproduced with permission.

Systemic causes of avasular necrosis (osteonecrosis) include: A Systemic lupus erythematosus B Thalassaemia C Pernicious anaemia D Sickle-cell disease E Addisons Disease (True) (True) (False) (True) (False)

Comments: The other systemic causes of avasular necrosis are:, scleroderma, infective endocarditis, alcoholism, extensive burns, radiation, diabetes mellitus, steroid therapy (e.g. following renal transplantation), Cushings disease and Gauchers disease. The local causes which predispose to this condition include: Rheumatoid arthritis (affecting that particular bone), trauma (as in fracture neck of femur; fractures of the scaphiod), severe osteoarthritis, and psoriatic arthropathy.

The following statements regarding coeliac disease are correct: A muscle wasting affects proximal and distal muscle groups equally. B strict dietary control prevents associated small bowel lymphomas. C the HLA-B8 antigen is present in about 80% of coeliac patients. D dietary restrictions are only necessary in childhood. E a wheat, barley and rye free diet is essential. (False) (False) (True) (False) (True)

Comments: Muscle wasting is one of the most consistent features along with failure to thrive, irritability, vomiting and diarrhoea, abdominal distension and offensive stools. Muscle wasting predominantly affects the proximal muscle groups. Strict dietary control does not prevent intestinal malignancies. HLA-B8 antigen is present in about 20% of the general population and in 80% of coeliac patients. Dietary restriction should be life-long. Wheat, rye and probably barley should be avoided. From Hannam et al. MRCP (Paediatrics) Part 1 MCQs. page 61

Pediatrics118

WB Saunders. Reproduced with permission.

The following therapies are appropriate: A 25% Benzyl Benzoate for scabies. B 1% Hydrocortisone for infantile eczema. C Coal Tar for psoriasis. D Surgical excision for a cavernous haemangioma 3cm and 4cm on the arm. E Ketoconazole for alopecia areata. (True) (True) (True) (False) (False)

Comments: Systemic or local steroids, sclerosants and laser treatments are used for cavernous haemangioma. Steroid injections and creams are used for alopecia areata.

In paediatric shock, which of the following is/are true: A hypotension is an early sign of hypovolaemia B the initial crystalloid bolus is 20 ml/Kg C a normal capillary refill time is < 1 second D the colloid of choice is 4.5% albumin E hyperglycaemia is a common finding Comments: Childrens cardiovascular systems compensate well initially in shock. Hypotension is a late and often sudden sign of decompensation and if not reversed will be rapidly followed by death. A formula for calculating normal systolic blood pressure is 80 + (2 X Age in years). Capillary refill time is a more useful test of perfusion in children than blood pressure measurement. The skin on the sternum or a digit held at the level of the heart should be pressed for 5 seconds. After blanching pressure has been released the time for the colour to return to normal is measured. A normal capillary refill time is < 2 seconds. Hypoglycaemia and shock may coexist as the sick child or infant has poor glucose producing reserves. Urgent blood glucose estimation must always be performed to exclude this common condition. Fluid resuscitation in paediatric shock is based on crystalloid blouses of 20 ml / Kg, which can be repeated up to 3 times. Blood is the colloid of choice although 4.5% albumin may have a role in seticaemia. (False) (True) (False) (False) (False)

Regarding the distraction test of hearing: A It is best carried out at 7 months. B It has a high sensitivity. (True) (False)

Pediatrics119

C It has a low specificity. D It can be improved using specialist teams. E It should not be used to screen babies at high risk. Comments:

(True) (True) (True)

Behavioural tests (distraction tests) requires 2 people, and depends on the infants ability to turn to localise a sound source. 7 months is optimal. Before this age, sitting, balance, head control and sound localisation are imperfect, and beyond 10 months of age, the development of object permanence and increasing sociability make it more difficult to carry out. The room must be quite and proper equipment, adequate sound level monitoring and careful technique are important. More definitive tests should be used to screen babies with high risk factors such as a positive family history or a long stay in an ICU. In practice the sensitivity of the test is low, missing many cases, and poor technique generates a large number of false positives, so the specificity is also low. Good results can be obtained if initial and regular refresher training courses are provided for a specialist team to ensure that technique is meticulous and standard guidelines are observed. Copyright 2002 Dr Colin Melville

A lesion of the sciatic nerve in the region of the buttock causes: A Weakness of knee flexors B Weakness of knee extensors C Complete loss of sensation below the knee D Weakness of dorsiflexion of the foot E Loss of the ankle jerk Comments: The sciatic nerve is the largest nerve in the body, and arises from the sacral plexus (L4, 5, S13). It passes through the greater sciatic foramen into the buttock and then descends through the posterior aspect of the thigh supplying the hamstrings. At the level of the mid-femur it divides to form the tibial and common peroneal nerves. The tibial nerve passes through the popliteal fossa to the posterior aspect of the leg where it supplies the muscles and the skin. The common peroneal nerve winds around the neck of the fibula and divides into the deep and superficial perineal nerves, which supply the skin and muscles of the anterior aspect of the leg and the dorsum of the foot and toes. The saphenous nerve, arising from the femoral nerve, supplies the antromedial aspect of the thigh and calf. Lesions of the sciatic nerve therefore cause weakness of knee flexors, but not extensors, and there is incomplete loss of sensation below the knee, particularly affecting the lateral and posterior parts of it. Plantar flexion is weak, but dorsal flexion is unaffected. The ankle jerk is absent (L5, S1). Copyright 2002 Dr Colin Melville (True) (False) (False) (False) (True)

Which of the following has/have an increased incidence in

Pediatrics120

adolescence? A Thyrotoxicosis B acne vulgaris C peptic ulceration D anorexia nervosa E scoliosis (True) (True) (False) (True) (True)

Comments: Scoliosis, AN, Thyroid disease, acne, osteosarcoma and lymphoma all have an increased adolescent prevalence.

It is considered safe to breast feed when the mother is on the following drugs: A amiodarone B warfarin C ciprofloxacin D rifampicin E sodium valproate (False) (True) (False) (True) (True)

Comments: Amiodarone is present in the milk in significant amounts and should be avoided. Warfarin is safe but phenindione should be avoided. High amounts of ciprofloxacin appear in the milk so should be avoided. Rifampicin and sodium valproate are excreted in breast milk in too small quantities to be harmful. From Hannam et al. MRCP (Paediatrics) Part 1 MCQs. page 157 WB Saunders. Reproduced with permission.

The following are effective treatments for glue ear: A Grommet insertion B Low power hearing aid C Decongestant therapy D Conservative management E Prophylactic nocturnal antibiotics Comments: The majority of cases of glue ear will resolve spontaneously with time, and in those children with occasional self-limiting episodes this probably has minimal influence on speech and language. Persistent severe otitis media may result in significant delay in language acquisition, (True) (True) (False) (True) (False)

Pediatrics121

behavioural disturbances, and sometimes disturbances of balance. Grommet insertion is effective while the grommets remain patent. Long term benefits are much less certain. The majority of patients will recover with conservative treatment. Decongestants, antibiotics and steroids are ineffective. A low power hearing aid may be helpful for the most severely affected children. Usually, once the parents have had the problem explained to them they make allowances for the decreased hearing and can communicate more effectively. Copyright 2002 Dr Colin Melville Haematuria in children may be due to: A vitamin A deficiency B nephroblastoma C meatal ulcer D measles E schistosomiasis Comments: 1. this causes keratomalacia and xerophthalmia 2. Wilm's tumour, haematuria usually a late feature. 3. due to bleeding from the meatal ulcer 5. due to bladder involvement in Schistosoma haematobium infection (False) (True) (True) (False) (True)

The following are normal findings in children: A standing unaided at 13 months B first lower molars erupting at 6 months C extensor plantar response at age 26 months D can say four words at 24 months E control of bowels by age 13 months Comments: b-incisors yes (True) (False) (False) (True) (False)

Which one of the following is/are true of sudden infant death syndrome? A It is the commonest cause of death in infants aged 1-2 months in this country. B Post mortem shows asphyxia in over 80% of cases. C There is a 10 fold increased risk of sudden infant death to siblings of children with SIDS.

(True) (False) (False)

Pediatrics122

D It is more common in low birth weight infants. E It is more common in low socio-economic groups.

(True) (True)

Comments: SIDS is the commonest cause of death in this age group. Post mortem fails to reveal a cause of death. There is a twice normal risk of SIDS in siblings and low birthweight/premature infants.

The following conditions have autosomal dominant inheritance: A achondroplasia B Huntington's chorea. C Christmas disease D myotonia dystrophica E tuberous sclerosis (True) (True) (False) (True) (True)

Comments: Autosomal dominant diseases include: achondroplasia adult polycystic kidney disease Gilbert's disease Hereditary spherocytosis Huntington's chorea hyperlipidaemia (type IV) Marfan's syndrome myotonic dystrophy neurofibromatosis retinoblastoma tuberous sclerosis von Willebrand's disease. From Hannam et al. MRCP (Paediatrics) Part 1 MCQs. page 5 WB Saunders. Reproduced with permission. Regarding associations with congenital heart defects: A Turners syndrome is well recognised to be associated with coarctation of aorta or aortic stenosis. B Trisomy D or E often associated with pulmonary stenosis. C Tetralogy of Fallot is the commonest congenital cyanotic heart disease. D Noonans syndrome is often associated with aortic stenosis. E The cause of a heart malformation can be found in the majority of cases with congenital heart disease.

(True) (False) (True) (False) (False)

Comments: Coarctation occurs in 10% of patients with Turners syndrome and 15% of patients with this condition have a bicuspid aortic valve. Pulmonary steno tic lesions are not associated with chromosomal defects. Tetralogy of Fallot is the commonest cause of cyanotic congenital heart disease in all age groups. Noonans syndrome is associated with pulmonary rather than aortic stenosis. In the majority of cases of congenital hear disease the cause of the defect is unknown.

In a 2 year old child further assessment is necessary if he is unable to: A Build a tower of 3 bricks (True)

Pediatrics123

B Put his shoes on C Feed himself with a spoon D Balance on one leg E Use two word phrases Comments: By 2 years of age the average mobile toddler can:

(False) (True) (False) (True)

GROSS MOTOR: Kick a ball FINE MOTOR/VISION: Build a tower of 6 cubes HEARING/SPEECH: Combine two words meaningfully (gimme dat) SOCIAL: Feed with spoon, point to a body part, show symbolic play. Balancing on one leg is, on average, achieved at 42 months, and putting shoes on at around the same time.

Copyright 2002 Dr Colin Melville

Which of the following is/are true regarding Perthe's disease? A Usually presents before 10 years of age B Is due to avascular necrosis of the distal femoral epiphysis C Is more common in girls D Plain x-ray may show the capital femoral epiphysis to be smaller, denser and flatter E May require surgical containment with a subtrochanteric osteotomy (True) (False) (False) (True) (True)

Comments: Perthe's disease is due to avascular necrosis of the proximal femoral epiphysis. It presents with hip pain and a limp usually before the age of 10 years. The male to female ratio is 4:1. Xrays shows the capital femoral eiphysis to be small, dense and flattened. A bone scan shows a cold spot within the epiphysis.

Infants with gastroenteritis: A In most instances require treatment with an antibiotic which is not absorbed from the gut B if formula-fed, should be given a lactose-free formula C Should be admitted to hospital if they are unable to tolerate fluid orally D Always develop lactose intolerance E Should have a barium meal if the initial refeeding with milk is unsuccessful

(False) (True) (True) (False) (False)

Pediatrics124

Comments: The majority of children do not require antibiotics as the cause is viral. Haemorrhagic diarrhoea may be seen with E.coli. Intravenous fluid therapy will be required if they are unable to tolerate oral fluids. Lactose intolerance is common, but not inevitable. Barium meals are not useful in the investigation of gastroenteritis.

Which of the following is/are true of scabies? A Typically it affects the face. B It causes itchiness in the skin even where there is no obvious lesion to be seen. C It is caused by Staph aureus. D Is best treated by salicylate emulsion. E It can be spread by a droplet infection. (False) (True) (False) (False) (False)

Comments: Scabies is an infestation of the skin with the microscopic mite Sarcoptes scabei. Infestation is common, found worldwide, and affects people of all races and social classes. Scabies spreads rapidly under crowded conditions where there is frequent skin-to-skin contact between people, such as in hospitals, institutions, child-care facilities, and nursing homes. Spread is by direct, prolonged, skin-to-skin contact with a person already infested with scabies. Contact must be prolonged (a quick handshake or hug will usually not spread infestation). Infestation is easily spread to sexual partners and household members. Infestation may also occur by sharing clothing, towels, and bedding. Scabies is characterised by Pimple-like irritations, burrows or rash of the skin, especially the webbing between the fingers; the skin folds on the wrist, elbow, or knee; the penis, the breast, or shoulder blades. A number of treatments are available for the treatment of scabies, including permethrin ointment, benzyl benzoate, and oral ivermectin for resistant cases. Antihistamines and calamine lotion may be used to alleviate itching.

The following associations are correct: A congenital rubella infection - pulmonary stenosis. B maternal diabetes - congenital heart disease. C congenital heart block - seronegative rheumatoid arthritis. D long QT syndrome - atrial tachyarrhythmias. E Noonan,s syndrome - pulmonary stenosis. (True) (True) (False) (False) (True)

Comments: Congenital rubella is associated with both pulmonary stenosis and patent ductus arteriosus (PDA). Infants of diabetics have an increased incidence of congenital heart disease, specifically transposition of the great arteries, ventricular septal defect and PDA. A cardiomyopathy has also been described. Congenital heart block is associated with maternal systemic lupus erythematosus. Long QT syndrome is associated with ventricular tachycardia.

Pediatrics125

From Hannam et al. MRCP (Paediatrics) Part 1 MCQs. page 65 WB Saunders. Reproduced with permission. The pupillary light reflex depends on the integrity of: A The optic nerve B The lateral geniculate nuclei C The occipital cortex D The medial longitudinal fasciculus E The 3rd cranial nerve Comments: There are 3 pupillary reflexes: Direct light reflex: abnormal in acute retrobulbar neuritis, Argyll-Robinson pupil. Consensual light reflex: not lost alone. Accommodation: occasionally absent in brainstem encephalitis. (True) (False) (False) (False) (True)

Shining a light into an eye tests the optic nerve, and both afferent and efferent pathways of III, while the consensual reflex tests only the efferent. Complete III palsy will remove all pupil reactions. The LGN is involved only in accommodation, not the light reflex. Copyright 2002 Dr Colin Melville Regarding bronchopulmonary dysplasia: A Children have higher than average weight gain because of fluid retention. B Children are susceptible to recurrent wheezing and chest infections. C Cor pulmonale is a recognised complication. D Doxopram therapy may be required. E It is associated with an increased incidence of congenital heart disease. Comments: Bronchopulmonary dysplasia is thought to result from a combination of oxygen toxicity, pressure (alveolar stress and volume trauma), hypocapnic surponification, absorption atelectasis, and subsequent inflammation. This results in persistent ventilatory and oxygen requirement with hypoxia, hypocarbia, oxygen dependency, and the development of right sided heart failure. The chest x-ray changes from complete opacification with air bronchogram to interstitial emphysema. Histologically, there is alveolar coalescence with atelectasis, interstitial oedema, and focal thickening of the basement membrane with bronchial and bronchiolar mucosal metaplasia and hyperplasia. There is severe maldistribution of ventilation. It is defined by oxygen dependency at 36 weeks gestational age. Some patients require prolonged hospitalisation, but often they can be discharged on home oxygen therapy. Right heart failure and viral bronchiolitis are the major causes of death. The children also fail to

(False) (True) (True) (False) (False)

Pediatrics126

thrive and require nutritional supplementation. Treatment includes nebulised bronchodilators, Theophylline, diuretics, fluid restriction, and aggressive treatment of infection, and food supplementation. CPAP may be required for tracheomalacia and Dexamethasone may be helpful in reducing oxygen requirement and getting children off the ventilator. Complications include: growth failure, transient psychomotor retardation, parental stress, and nephrolithiasis, osteopenia, subglottic stenosis. Since new alveoli continue to develop up to the age of about 7 or 8, children often improve with age. Prognosis is good for children who have been weaned off oxygen prior to discharge from the NICU, but prolonged ventilation, associated IVH, pulmonary hypertension, cor pulmonale and oxygen dependence beyond 1 year of life are poor prognostic signs. Long term bronchoreactivity may be a problem. Copyright 2002 Dr Colin Melville

Which of the following therapies are appropriate for the condition indicated? A 25% Benzyl Benzoate for scabies. B 1% Hydrocortisone for infantile eczema. C Coal Tar for psoriasis. D Surgical excision for a cavernous haemangioma 3cm and 4cm on the arm. E Ketoconazole for alopecia areata. (True) (True) (True) (False) (False)

Comments: Cavernous haemangiomas are usually not present at birth but appear in the first two weeks of life. Lesions are usually on the face, neck or trunk and are well-circumscribed and lobulated. Treatment options do not include surgical excision. Treatment may be indicated if there is inhibition of normal development - for example impairing normal binocular visual development by obstructing the vision from one eye. It may involve systemic or local steroids, sclerosants, interferon, or laser treatment. Alopecia areata is an autoimmune condition causing discrete areas of hair loss. Treatment options include cortisone injections into the affected areas, and the use of topical cortisone creams.

Polyhydramnios is associated with: A Renal agenesis. B Oesophageal atresia. C Rectal atresia. D Congenital club foot. E Anencephaly. (False) (True) (False) (False) (True)

Comments: Renal agenesis causes absent urination and oligohydramnios. Duodenal atresia and

Pediatrics127

oesophageal atresia are associated rather than rectal atresia. 26% of cases of polyhydramnios have neural tube defects.

The following are recognized causes of acute interstitial nephritis in children: A phenytoin. B cytomegalovirus (CMV). C sarcoidosis. D transplant rejection. E captopril. (True) (True) (True) (True) (False)

Comments: Interstitial nephritis is defined as inflammation between glomeruli in areas surrounding the tubules. There are many causes, which can be divided into acute or chronic: Acute: Drugs penicillins, sulphonamides, cotrimoxazole, rifampicin, phenytoin, thiazides, frusemide, allopurinol, cimetidine. Infections - streptococcal, pyelonephritis, toxoplasmosis, diphtheria, brucellosis, leptospirosis, infectious mononucleosis, CMV. Dieseases - sarcoidosis, glomerulonephritis, transplant rejection. Chronic: Drugs - analgesics, lithium. Infections pyelonephritis. Diseases - vesico-ureteric reflux, nephrocalcinosis, prolonged hypokalaemia, oxalate nephropathy, heavy metals, radiation, obstructive uropathy, medullary cystic disease. From Hannam et al. MRCP (Paediatrics) Part 1 MCQs. page 135 WB Saunders. Reproduced with permission.

In human skin: A apocrine sweat glands produce secretions with a characteristic odour. B apocrine glands are innervated by cholinergic sympathetic fibres. C chondroitin sulphate is the main glycosaminoglycan (GAG) in the dermis. D sebum consists of approximately equal amounts of free fatty acids, cholesterol and glycerides. E a comedo is a blocked pilosebaceous gland. (False) (False) (True) (False) (True)

Comments: Apocrine and eccrine glands are innervated by sympathetic fibres, the apocrine by adrenergic and the eccrine by cholinergic fibres. Apocrine secretions are odourless but the action of bacteria on the skin produces the foul smell of body odour. Chondroitin sulphate is the main GAG of the dermis. Sebum consists mainly of free fatty acids and glycerides; cholesterol is almost absent. From Hannam et al. MRCP (Paediatrics) Part 1 MCQs. page 111 WB Saunders. Reproduced with permission.

A 6-year-old boy has a 2-week history of being generally unwell. He presents with abdominal pain, arthritis and a widespread rash over his

Pediatrics128

legs and buttocks. A diagnosis of Henoch-Schonlein purpura (HSP) would be supported by the following findings: A a low platelet count. B the rash distribution being centrifugal. C a vasculitis affecting large muscular arteries being present. D the rash being purpuric in nature. E the presence of haematuria (False) (True) (False) (True) (True)

Comments: HSP is the most common form of non-thrombocytopenic purpura, the rash having a centrifugal distribution on the limbs rather than the trunk. The rash in HSP is vasculitic. The vasculitis of HSP afffects the small non-muscular arterioles, capillaries and venules. A vasculitis of the large muscular arteries suggests polyarteritis nodosa. Renal involvement occurs in 25-50% of HSP. From Hannam et al. MRCP (Paediatrics) Part 1 MCQs. page 149 WB Saunders. Reproduced with permission.

Hypermobility is associated with A hyperlysinaemia B tabes dorsalis C homocystinuria D rheumatoid arthritis E osteogenesis imperfecta (True) (True) (True) (True) (True)

Comments: Two conditions that definitely have joint hypermobility and are connective tissue disorders are Ehlers-Danlos and Marfan syndrome. These two conditions exhibit hyperelastic skin, hernias, lenticular abnormalities and abnormal body proportions. Other conditions exhibiting hypermobile joints are rheumatoid arthritis, osteogenesis imperfecta, systemic lupus erythematosus, poliomyelitis, myotonia congenita, and some neurological conditions.

The following may cause a downbeat nystagmus: A Chiari type I malformation B Unilateral medial longitudinal fasciculus lesion C Central cerebellar lesion D Wernicke's encephalopathy E Aqueduct stenosis Comments: (True) (False) (False) (False) (False)

Pediatrics129

Nystagmus is defined as involuntary oscillations of the eyes. This may be pendular when the oscillations are equal in rate and amplitude; jerking - when there are quick and slow phases (the quicker phase is used to define the direction. Nystagmus may be caused by visual disturbances, lesions of the labyrinth or the central vestibular connections, or by brain stem or cerebellar lesions. Pendular nystagmus is usually due to loss of macular vision, but may be seen in diffuse brain stem lesions. Jerking nystagmus, which is of constant direction regardless of the direction of gaze, suggests a labyrinthine or cerebellar lesion. Nystagmus which changes with the direction of gaze suggests a widespread central involvement of vestibular nuclei. Jerking nystagmus present only on lateral gaze, and who's fast component is in the direction of gaze, indicates a lesion of the brain stem or cerebellum. Nystagmus confined to one eye suggests a peripheral lesion of the nerve or muscle, or a lesion of the medial longitudinal bundle. Nystagmus restricted to the abducting eye on lateral gaze (ataxic nystagmus) is due to a lesion of the medial longitudinal bundle between the ponds and midbrain as in MS. Nystagmus occurring on upward gaze with the fast component upwards (upbeat nystagmus) may be due to a lesion in the mid-brain at the level of the superior colliculus. Downbeat nystagmus (fast phase downwards) suggests a lesion in the lower part of the medulla. It is therefore, typical of the Arnold Chiari malformation. Copyright 2002 Dr Colin Melville Features of pyloric stenosis include: A hypercholoraemic hypokalaemic metabolic alkalosis. B bile-stained vomiting. C a low urinary pH. D a greater incidence in boys than girls. E an association with Down's syndrome. (False) (False) (True) (True) (False)

Comments: The metabolic abnormality in pyloric stenosis is a hypochloraemic hypokalaemic metabolic alkalosis caused by loss of potassium, chloride and sodium. The urine is paradoxically acidic as the kidneys attempt to reabsorb potassium in the urine in exchange for hydrogen ions in the blood. From Hannam et al. MRCP (Paediatrics) Part 1 MCQs. page 169 WB Saunders. Reproduced with permission Babies born to diabetic mothers when compared with babies born to normal non-diabetic mothers are more likely to suffer from: A Hypoglycaemia B Respiratory distress C Cardiac arrhythmia D Jaundice E Diabetes later in life (True) (True) (False) (True) (True)

Comments: Hypoglycaemia results from excessive foetal insulin secretion in response to maternal glucose.

Pediatrics130

Foetal hyperglycaemia results in delayed lung maturation and increased incidence of respiratory distress. Asphyxia may result from being large for dates. Cardiac arrhythmias per se are not observed more frequently, but congenital heart disease is more common in offspring. Hyperbilirubinaemia occurs and infants of mothers with type 2 diabetes, type 1 diabetes and gestational diabetes are all at risk of subsequent diabetes.

The following are recognised complications of glue ear: A Mastoiditis B Delayed speech development C Learning difficulties D Behavioural problems E Hyperacusis Comments: Chronic secretory otitis media (glue ear) is the commonest cause of conductive hearing loss in children. It is very common, and usually transient. When it persists for many months, it can interfere with speech development, and result in learning and behavioural difficulties. There may be earache because of pressure changes due to the obstructed eustachian tube. Copyright 2002 Dr Colin Melville (False) (True) (True) (True) (False)

At a one-month check-up in clinic a murmur is heard for the first time in an acyanotic, well baby. A diagnosis of a ventricular septal defect was made. A The family is justified for being upset that the murmur was not heard in the newborn period. B It is possible that congestive heart failure will develop a month later. C The baby may present with recurrent pulmonary infection later. D The murmur most likely is diastolic. E A chest x-ray is likely to show decreased pulmonary vascularity.

(False) (True) (True) (False) (False)

Comments: Ventricular septal defects commonly present after 1 month of life. Recurrent pulmonary infection is a presentation of congenital heart disease. The murmur is most likely to be holosystolic and chest x-ray will show plethoric lung fields.

The following apply to recommendations for surgery in children with congenital cardiac diseases: A Surgical correction for a large ventricular septal defect is best between (False)

Pediatrics131

eight and ten years. B Closure of an atrial septal defect is indicated when the child shows signs of marked pulmonary hypertension. C Ligation of patent ductus arteriosus should be postponed until school age because spontaneous closure may occur after one year of age. D Correction of tetralogy of Fallot should be carried out as soon as the diagnosis is confirmed. E Surgical closure of PDA may be life-saving in premature newborn babies with heart failure not responding to medical treatment. (False) (False) (False) (True)

Comments: Closure of an ASD is recommended before the onset of pulmonary hypertension. And necessitates the need for heart lung transplant. Ligation of a PDA is advised if the defect has not corrected spontaneously after 6 months. Immediate surgical closure of a PDA is recommended in neonates with heart failure not responding to medical management. Correction of Tetralogy of Fallot is carried out at 1-5 years of age; palliative shunting can be performed prior. VSD closure depends on the degree of heart failure and location of the VSD. Muscular VSDs often close, so a medical approach will be tried for the first few months of life at least and closure monitored on echo. Large haemodynamically significant perimembranous VSDs may require closure in the first year.

Which of the following statements are correct concerning Hypertrophic cardiomyopathy? A is familial B is associated with Friedrich's ataxia C may be usefully treated with nitrates D treated with beta adrenergic blockers has a lower risk of sudden death E is best screened for by a 12-lead electrocardiogram (True) (True) (False) (False) (False)

Comments: Hypertrophic cardiomyopathy has a well recognised familial often dominant form of ineritance. a-The penetrance is higher in males. b-It is also associated with Pompe's disease and familial lentiginosis. c-Nitrates and other vasodilators are best avoided. d-Beta blockers reduce the symptoms but not the risk of sudden death. e-ECHO is the best screening tool. The ECG has low sensitivity but high specificity for ventricular hypertrophy.

The following are typical features of Downs syndrome: A Hypotonia B Webbing of the neck C A mitral diastolic murmur (True) (False) (False)

Pediatrics132

D Recurrent urinary tract infections E Aggressive behaviour

(False) (False)

Comments: Hypotonia is common. Webbing of the neck is a feature of Turners Sydrome. A diastolic murmur is not a typical feature. Congenital heart disease occurs in 25% of cases and the commonest are AV canal defect and a VSD. Urinary tract infections are not associated. The individuals affected by this disorder are usually passive. The following are typical features of Downs syndrome: A Hypotonia B Webbing of the neck C A mitral diastolic murmur D Recurrent urinary tract infections E Aggressive behaviour (True) (False) (False) (False) (False)

Comments: Hypotonia is common. Webbing of the neck is a feature of Turners Sydrome. A diastolic murmur is not a typical feature. Congenital heart disease occurs in 25% of cases and the commonest are AV canal defect and a VSD. Urinary tract infections are not associated. The individuals affected by this disorder are usually passive.

Pediatrics133

You might also like